Download as pdf or txt
Download as pdf or txt
You are on page 1of 193

D:\EG_Mathematics_Standard-10_(Term-1)_(30-07-2021)\Open_Files\0_Prelims\0_Prelims

\ 14-Sep-2021   Bhandari  Proof-3 Reader’s Sign _______________________ Date __________

TERM - 1 10 CBSE
2021-22

MATHEMATICS
(STANDARD)

Dr. Sangeeta Arora


MSc, PhD

Visit https://telegram.me/booksforcbse for more books.

Full Marks Pvt Ltd


(Progressive Educational Publishers)
An ISO : 9001-2015 Company

New Delhi-110002
D:\EG_Mathematics_Standard-10_(Term-1)_(30-07-2021)\Open_Files\0_Prelims\0_Prelims
\ 14-Sep-2021   Bhandari  Proof-3 Reader’s Sign _______________________ Date __________

Published by:

An ISO : 9001-2015 Company

9, Daryaganj, New Delhi-110002


Phone: 011- 40556600 (100 Lines)
Website: www.fullmarks.org
E-mail: info@fmpl.in

© Publishers

All rights reserved. No part of this publication may be reproduced or transmitted, in any form or by any means,
without permission. Any person who does any unauthorised act in relation to this publication may be liable to criminal
prosecution and civil claims for damages.

Branches:
• Chennai • Guwahati

Marketing Offices:
• Ahmedabad • Bengaluru • Bhopal • Dehradun • Hyderabad • Jaipur • Jalandhar • Kochi
• Kolkata • Lucknow • Mumbai • Patna • Ranchi

NEW EDITION

“This book is meant for educational and learning purposes. The author of the book has taken all reasonable care to ensure
that the contents of the book do not violate any existing copyright or other intellectual property rights of any person in any
manner whatsoever. In the event the author has been unable to track any source and if any copyright has been inadvertently
infringed, please notify the publisher in writing for corrective action.”

Printed at:
D:\EG_Mathematics_Standard-10_(Term-1)_(30-07-2021)\Open_Files\0_Prelims\0_Prelims
\ 14-Sep-2021   Bhandari  Proof-3 Reader’s Sign _______________________ Date __________

Note from the Publishers

Mathematics Standard-10 (Term-1) is based on the latest curriculum released by


CBSE in July 2021. It will certainly prove to be a torch-bearer for those who toil hard to achieve
their goal.
This All-in-one Question Bank has been developed keeping in mind all the requirement of the students
for Board Examinations preparations like learning, practicing, revising and assessing.

Salient Features of the Book:


• Each chapter is designed in ‘Topic wise’ manner where each topic is briefly explained with
sufficient Examples and Exercise.
• In all chapters, Topic wise exercises cover MCQs, Assertion-Reason and Case Study Questions
as per the Special Scheme of Assessment suggested by CBSE vide Circular No. Acad-51/2021.
Answers and sufficient hints are also provided separately at the end of exercise.
• Common Errors by the students are provided to make students aware what errors are usually
committed by them unknowingly.
• Chapterwise Important Formulae and Quick Revision Notes have been prepared for Quick
Revision.
• Experts’ Opinion has been provided to suggest the students which type of questions are important
for examination point of view.
• 3 Sample Papers (including latest CBSE Sample Question Paper with Marking Scheme) for mock
test are given with answers and OMR Sheets for self assessment.
• The book has been well prepared to build confidence among students.
Suggestions for further improvement of the book, pointing out printing errors/mistakes which
might have crept in spite of all efforts, will be thankfully received and incorporated in the next
edition.

–Publishers

(iii)
D:\EG_Mathematics_Standard-10_(Term-1)_(30-07-2021)\Open_Files\0_Prelims\0_Prelims
\ 14-Sep-2021   Bhandari  Proof-3 Reader’s Sign _______________________ Date __________

Mathematics 10

E:\Working\EG_Mathematics-10_(Term-1)\Open_Files\Ch-1\Ch-1
\ 30-Jul-2021   Praveen Kumar  Proof-3
GLIMPSE OF A CHAPTER
Reader’s Sign _______________________ Date __________

. Exercise related to each topic dealt separately and


1 Real Numbers Questions included segregated into Multiple Choice
Topics covered
Questions, Assertion-Reason Type Questions and
E:\Working\EG_Mathematics-10_(Term-1)\Open_Files\Ch-1\Ch-1
\ 30-Jul-2021   Praveen Kumar  Proof-3 Reader’s Sign _______________________ Date __________

1. The Fundamental Theorem of Arithmetic


2. Decimal Representation of Rational Numbers
Case Study Based Questions.
IntroductIon
We know that all the rational and irrational numbers together make up the collection of real numbers.
In this chapter, we shall study divisibility of integers and expressing positive integers as a product of Exercise 1.1
prime integers. Let us recall divisibility of integers and product of prime integers.
Divisibility of integers: Any positive integer a can be divided by another positive integer b in such a
way that it leaves a remainder r that is smaller than b.
A. Multiple Choice Questions (MCQs)
Product of prime numbers: Every composite number can be expressed as a product of primes. Choose the correct answer from the given options:
We shall also study some more topics in this chapter such as Fundamental Theorem of Arithmetic,
rational and irrational numbers, etc. 1. If two positive integers a and b are written as a = x3y2 and b = xy3; x, y are prime numbers then HCF
(a, b) is:
1. the Fundamental theorem oF arIthmetIc
(a) xy (b) xy2 (c) x3y3 (d) x2y2
Fundamental Theorem of Arithmetic: Every composite number can be expressed (factorised) as a
product of primes and this expression (factorisation) is unique, apart from the order in which the prime 2. The LCM of smallest two-digit composite number and smallest composite number is:
factors occur. (a) 12 (b) 4 (c) 20 (d) 44
For any two positive integers a and b, HCF (a, b) × LCM (a, b) = a × b.
Example 1. The values of A and B in the following factor tree, respectively are 3. 325 can be expressed as a product of its primes as:
(a) 52 × 7 (b) 52 × 13 (c) 5 × 132 (d) 2 × 32 × 52
A
4. The total number of factors of a prime number is [CBSE Standard 2020]
B
(a) 1 (b) 0 (c) 2 (d) 3
(a) 42, 21 (b) 21, 42 (c) 12, 10 (d) 10, 12
5. The HCF and the LCM of 12, 21, 15 respectively are: [CBSE Standard 2020]
Solution. The factor tree with missing entries filled is given below. (a) 3, 140 (b) 12, 420 (c) 3, 420 (d) 420, 3
B. Assertion-Reason Type Questions
In the following questions, a statement of assertion (A) is followed by a statement reason (R). Choose
the correct choice as:
Hence, option (a) is the correct answer. (a) Both assertion (A) and reason (R) are true and reason (R) is the correct explanation of assertion (A).
Example 2. When 2120 is expressed as the product of its prime factors we get
(a) 2 × 53 × 53 (b) 23 × 5 × 53 (c) 5 × 72 × 31 (d) 52 × 7 × 33 (b) Both assertion (A) and reason (R) are true but reason (R) is not the correct explanation of assertion
(A).
9 (c) Assertion (A) is true but reason (R) is false.
(d) Assertion (A) is false but reason (R) is true.
.E ach chapter is divided into topics and 1. Assertion (A): The number 6n, n being a natural number, ends with the digit 5.
Reason (R): The number 9n cannot end with digit 0 for any natural number n.
explained separately.
E:\Working\EG_Mathematics-10_(Term-1)\Open_Files\Ch-1\Ch-1
2. Assertion (A): If m and n are odd positive integers, then m2 + n2 is even but not divisible by 4.
Reason (R): 3 × 5 × 7 + 7 is a composite number.
\ 30-Jul-2021   Praveen Kumar  Proof-3 Reader’s Sign _______________________ Date __________

Case Study Based Questions


I. To enhance the reading skills of grade X students, the school nominates you and two of your friends
to set up a class library. There are two sections — section A and section B of grade X. There are 32
21. (d) Planes take off from five different runways B. Assertion-Reason Type Questions students in section A and 36 students in section B.
at 3, 4, 8, 12, 15 minutes intervals. To find 1. (d) Assertion (A) is false but reason (R) is true.
when all planes will take off together again
if they take off at 7 : 30 a.m. once, we must 2. (b) Both assertion (A) and reason (R) are true
find the LCM of 3, 4, 8, 12 and 15. but reason (R) is not the correct explanation
Now 3=3×1 of assertion (A).
4 = 22
8 = 23 Case Study Based Questions
12 = 22 × 3 I. 1. (c) 288 2. (b) 4
15 = 3 × 5
\ LCM = 23 × 3 × 5 3. (a) 22 × 32
= 40 × 3 = 120 minutes 4. (b) composite number
= 2 hours 5. (b) a2b2
fi Plane will took off together again after 2 II. 1. (d) 22 × 33 2. (c) 12
hours 1. What is the minimum number of books you will acquire for the class library, so that they can be
i.e. 9:30 a.m. 3. (b) 432 4. (a) 8:27:12 hrs
distributed equally among students of section A or section B?
22. (c) Now, number of rooms required for 5. (d) 5184 (a) 144 (b) 128 (c) 288 (d) 272
60 III. 1. (b) 2 × 7 × 11
3 1 1
2. (c) 2464
participants in Hindi = =5
12 3. (b) 8 4. (d) 8
Number of rooms required for participants Real NumbeRs 11
84 5. (a) 23 × 53
in English = =7 IV. 1. (b) 13915 2. (c) 11
12
Number of rooms required for participants 3. (b) 23
108 4. (a) Composite number
in Mathematics = =9
12 5. (c) 5 × 112 × 23
fi Number of rooms needed = 21.

2. Decimal RepResentation of Rational numbeRs


We know that rational numbers and irrational numbers together are known as real numbers.
Irrational Number
p
A number is called irrational if it cannot be written in the form , where p and q are integers and q ≠ 0.
Examples of irrational numbers are 2 , 3 , 15 , p, etc.
q
. Topic wise concepts are
Theorem 1. Let p be a prime number. If p divides a2, then p divides a, where a is a positive integer.
This theorem is based on Fundamental Theorem of Arithmetic.
presented to remember them
Rational Numbers and their Decimal Expansions
p
easily.
A number is called rational if it can be expressed in the form q , where p and q are both integers and q π 0.
2 10
Examples of rational numbers are , , 9, 3.5, 2.045, etc.
5 15
p
Theorem 2. Let x = , where p and q are co-primes, be a rational number whose decimal expansion terminates.
q
Then, the prime factorisation of q is of the form 2n5m, where n, m are non-negative integers.

Theorem 3. Let x =
p
be a rational number, such that the prime factorisation of q is of the form 2n5m, where . Each topic is well explained
q
n, m are non-negative integers. Then x has a decimal expansion which terminates. with relevant example for better
16 mathematics–10 understanding.

(iv)
D:\EG_Mathematics_Standard-10_(Term-1)_(30-07-2021)\Open_Files\0_Prelims\0_Prelims
\ 30-Jul-2021   Praveen Kumar  Proof-3 Reader’s Sign _______________________ Date __________

\ 14-Sep-2021   Bhandari  Proof-3 Reader’s Sign _______________________ Date __________

Case Study Based Questions


I. 1. (b) Irrational 2. (d) Non-terminating repeating 3. (d) 2 and 5
(
4. (c) 3 + 7 3 − 7 )( )
E:\Working\EG_Mathematics_Basic-10_(Term-1)\Open_Files\Ch-2\Ch-2
\ 01-Aug-2021   Amit  Proof-1
5. (d) four decimal places
Reader’s Sign _______________________ Date __________

II. 1. (d) All of these 2. (c) 15/4 3. (c) 11.10203040...


4. (a) One decimal place 5. (b) 21 × 51

ExpErts’ OpiniOn
2. The number of zeroes
Following typesof the polynomial yare
of questions = p(x) is important for examination point of view. So, students are
very
(a) 1advised to revise(b) 2 thoroughly. (c) 3
them (d) 4
3. The zeroes of the polynomial are
1. Applications of Fundamental Theorem of Arithmetic.
(a) –4, –2, 2, 4 (b) –4, –1, 2, 4 (c) –4, –2, 0, 4 (d) –2, 0, 2, 4
2. Exploring rational numbers and their decimal expansions.
4. The expression of the polynomial is
(a) x4 + 20x2 + 64 (b) x4 – 20x2 + 64 (c) x4 – 20x2 – 64 (d) x4 + 20x2 – 64
5. The value of the polynomial when x = 2 is
QUICK REVISION NOTES
(a) 144
• The Fundamental (b) –128 (c) 0
Theorem of Arithmetic: (d) 32number can be expressed (factorised)
Every composite
Ans. 1. (b) parabola 2. (d) 4 3. (a) apart
–4, –2,from
2, 4 the other in which the
.As these topics find place in the exam most of the
as a product of primes, and this expression (factorisation) is unique,
4. (b) x4prime
– 20x2factors
+ 64 occur. 5. (c) 0
•time it two
For any is positive
suggested
integers athat
and b, student
HCF (a, b) ×should
LCM (a, b)put
= a ×more
b.
•emphasis
Let p
ExpErts’ OpiniOnbe a on
prime these
number. If p topics.
divides a2, then p divides a, where a is a positive integer.
Questions based p
• Let onx =following types
, where p are
andvery
q areimportant for exams.
co-primes, So, students
be a rational are advised
number to revise expansion terminates.
whose decimal
them thoroughly. q
1. Finding zeroes of the quadratic polynomial and the relation betweenn the zeroes and the coefficients.
Then, the prime factorisation of q is of the form 2 5m, where n, m are non-negative integers.
2. Forming a quadratic polynomial with given sum of zeroes and product of zeroes.
3. Finding •
p a polynomial if its some zeroes are given.
all Let x = ofbe
the zeroes a rational number, such that the prime factorisation of q is of the form 2n5m, where n, m
q E:\Working\EG_Mathematics-10_(Term-1)\Open_Files\Ch-1\Ch-1
\ 30-Jul-2021   Praveen Kumar  Proof-3 Reader’s Sign _______________________ Date __________
are non-negative integers. Then, x has a decimal expansion which terminates.
p IMPORTANT FORMULAE
• Let x = be a rational number such that the prime factorisation of q is not of the form 2n5m, where n,
• If a, b are zeroes ofq the quadratic polynomial ax2 + bx + c, then
m are
.Quite effective for a quick revision
−bnon-negative integers. Then, x has a non-terminating
c repeating (recurring) decimal expansion.
(i) a + b =
a
E:\Working\EG_Mathematics-10_(Term-1)\Open_Files\Ch-1\Ch-1
(ii) ab =
a
Case Study Based Questions
before exams. Have the complete
essence of
the chapter.
\ 30-Jul-2021   Praveen Kumar  Proof-3 Reader’s Sign _______________________ Date __________
COMMON
• If a, b are zeroes of the quadratic polynomial p (x), then p (x) = x2 –ERRORS
(a + b)I.x +1.ab (b) Irrational 2. (d) Non-terminating 3. (d)
repeating 2 and 5
Errors 4. (c) Corrections
3+ 7 3− 7 ( )( )
5. (d) four decimal places
(i) Incorrectly using theQUICK REVISION
result for NOTES (i) The correct
three numbers II. 1.results
(d) Allare:
of these 2. (c) 15/4 3. (c) 11.10203040...
HCF (p, q, r) × LCM (p, q, r) = p × q × r 4. (a) One decimal pqrplace
HCF( p, q5., r(b)
) 21 × 51
• A polynomial p(x) in one variable x is an algebraic expression in x ofLCM (p, q, r) =
the form
Case Study Basedn Questions HCF( p, q ) ⋅ HCF(q, r ) ⋅ HCF( p, r )
I. 1. (b) Irrationaln 2. (d) Non-terminating repeating
ExpErtswith
p(x) = a x + an–1xn–1 + ... + a2x2 + a1x + a0 where an, an–1, ..., a0 are real numbers
3. (d) 2 and 5
’ OpiniOn
an ≠ 0.
pqr LCM ( p, q, r )
• The highest power of x in p(x) is called the degree of the polynomial HCF q, r) =
p(x). (p, Following types of questions are very important for examination point of view. So, students are
(
4. (c) 3 + 7 3 − 7 )( )
5. (d) four decimal places 2 advisedLCM ( p, qthem
to revise
• A quadratic polynomial in x with real coefficients is of the form ax + bx + c, where a, b, c are real
) ⋅ LCM (q, r ) ⋅ LCM ( p, r )
thoroughly.
II.numbers
1. (d) (ii)
All Misunderstanding
with ofa πthese
0. 2. (c)the
that 15/4
prime factors of (ii) If any 3.one
(c)
1.of11.10203040...
2 or 5 occurs,
Applications then also it has
of Fundamental terminating
Theorem of Arithmetic.
• Let4.p(x)
(a) beOne decimal place and k5.be(b) × 5number,
any2 both
1 1
a denominator
polynomial in xshould have real 2 and 5then
forthe value
decimal expansion.
obtained2.byExploring
replacingrational
x by k innumbers and their decimal expansions.
p(x) is called the decimal expansion
of p(x) to be
at x = k and terminating.
by p(k).
ExpErts’ the value
OpiniOn is denoted
QUICK REVISION NOTES
• A real number k is said to be a zero of a polynomial p(x) if pp(k) = 0. p view. So, students are
Following types of questions are very important for examination point of
(iii) Deciding
A polynomial may have incorrectly about
more athan one zeroes. (iii) First make
number co-prime and then decide whether it is
• advised to revise themno zero, one or
thoroughly. q • The q Fundamental Theorem of Arithmetic: Every composite number can be expressed (factorised)
• A1.polynomial
Applicationstop(x)of of
have degree n hasTheorem
terminating
Fundamental atmost n zeroes.
or non-terminating
of Arithmetic. terminatingasora product of primes,expansion.
non-terminating and this expression (factorisation) is unique, apart from the other in which the
• The zeroes of a
2. Exploringexpansionpolynomial p(x) are
just by looking
rational numbers precisely
and theiratdecimal the x-coordinates
the denominator
expansions. of the pointsprime
where factors occur.
the graph of
y = p(x) intersects
withoutthe x-axis.
making p and q co-prime. • For any two positive integers a and b, HCF (a, b) × LCM (a, b) = a × b.
Let p be acprime number. If p divides a2, then p divides a, where a is a positive integer.
• −b
QUICK REVISION NOTES
• If a and b are the zeroes of the quadratic polynomial ax2 + bx + c, then a + b = , ab = p .
a x = a, where p and q are co-primes, be a rational number whose decimal expansion terminates.
• Let
• The 22
Fundamental Theorem of Arithmetic:
MatheMatics –10 Every composite number can be expressed q (factorised)
as a product of primes, and this expression (factorisation) is unique, apart Then,
from thethe other
primeinfactorisation
which the of q is of the form 2n5m, where n, m are non-negative integers.
prime factors occur. Polynomials
p 29
• For any two positive integers a and b, HCF (a, b) × LCM (a, b) = a × b. • Let x = be a rational number, such that the prime factorisation of q is of the form 2n5m, where n, m
q
• Let p be a prime number. If p divides a2, then p divides a, where a is a positive integer.
are non-negative integers. Then, x has a decimal expansion which terminates.
p p
• Let x = , where p and q are co-primes, be a rational number whose decimal • Let x =expansion terminates.
be a rational number such that the prime factorisation of q is not of the form 2n5m, where n,
q q
Then, the prime factorisation of q is of the form 2n5m, where n, m are non-negative integers.
m are non-negative integers. Then, x has a non-terminating repeating (recurring) decimal expansion.
p
• Let x = be a rational number, such that the prime factorisation of q is of the form 2n5m, where n, m
q
COMMON ERRORS
are non-negative integers. Then, x has a decimal expansion which terminates.
p Errors Corrections
• Let x = be a rational number such that the prime factorisation of q is not of the formusing
(i) Incorrectly 2 5 the
n m
resultn,for three numbers (i) The correct results are:
, where
q
HCF (p,
m are non-negative integers. Then, x has a non-terminating repeating (recurring) q, r) ×expansion.
decimal LCM (p, q, r) = p × q × r pqr HCF( p, q, r )
LCM (p, q, r) =
HCF( p, q ) ⋅ HCF(q, r ) ⋅ HCF( p, r )
COMMON ERRORS HCF (p, q, r) =
pqr LCM ( p, q, r )
Errors Corrections LCM ( p, q ) ⋅ LCM (q, r ) ⋅ LCM ( p, r )
(i) Incorrectly using the result for three numbers (i) The correct results are:(ii) Misunderstanding that the prime factors of . (ii)CIfommon
any one of 2 errors
or 5 occurs,have been
then also tagged
it has terminating
HCF (p, q, r) × LCM (p, q, r) = p × q × r denominator
pqr HCF( p, should
q, r ) have both 2 and 5 for decimal expansion.
LCM (p, q, r) = the decimal expansion to
HCF( p, q ) ⋅ HCF(q, r ) ⋅ HCF( p, r ) be terminating. to clear confusions with cautions
pqr LCM ( p, q, r ) p and answers p for productive
HCF (p, q, r) = (iii) Deciding incorrectly about a number (iii) First make co-prime and then decide whether it is
LCM ( p, q ) ⋅ LCM (q, r ) ⋅ LCM ( p, r ) q learning.q
(ii) Misunderstanding that the prime factors of (ii) If any one of 2 or 5 occurs,tothen
have
alsoterminating or non-terminating
it has terminating terminating or non-terminating expansion.
denominator should have both 2 and 5 for decimal expansion. expansion just by looking at the denominator
the decimal expansion to be terminating. without making p and q co-prime.

p p
(iii) Deciding incorrectly about a number (iii) First make co-prime and then decide whether it is
q q 22 MatheMatics–10
to have terminating or non-terminating terminating or non-terminating expansion.
expansion just by looking at the denominator
without making p and q co-prime.

22 MatheMatics–10

(v)
D:\EG_Mathematics_Standard-10_(Term-1)_(30-07-2021)\Open_Files\0_Prelims\0_Prelims
\ 14-Sep-2021   Bhandari  Proof-3 Reader’s Sign _______________________ Date __________

SYLLABUS

FIRST TERM
Units Unit Name Marks
I. Number Systems 06
II. Algebra 10
III. Coordinate Geometry 06
IV. Geometry 06
V. Trigonometry 05
VI. Mensuration 04
VII. Statistics & Probability 03
Total 40

UNIT I: NUMBER SYSTEMS


1. Real Number
Fundamental Theorem of Arithmetic - statements after reviewing work done earlier and after
illustrating and motivating through examples, Decimal representation of rational numbers in terms
of terminating/non-terminating recurring decimals.

UNIT II: ALGEBRA


2. Polynomials
Zeros of a polynomial. Relationship between zeros and coefficients of quadratic polynomials only.
3. Pair of Linear Equations in Two Variables
Pair of linear equations in two variables and graphical method of their solution, consistency/
inconsistency.
Algebraic conditions for number of solutions. Solution of a pair of linear equations in two variables
algebraically - by substitution and by elimination. Simple situational problems. Simple problems
on equations reducible to linear equations.

UNIT III: COORDINATE GEOMETRY


4. Lines (In Two-dimensions)
Review: Concepts of coordinate geometry, graphs of linear equations. Distance formula. Section
formula (internal division).

(vi)
D:\EG_Mathematics_Standard-10_(Term-1)_(30-07-2021)\Open_Files\0_Prelims\0_Prelims
\ 14-Sep-2021   Bhandari  Proof-3 Reader’s Sign _______________________ Date __________

UNIT IV: GEOMETRY


5. Triangles
Definitions, examples, counter examples of similar triangles.
1. (Prove) If a line is drawn parallel to one side of a triangle to intersect the other two sides in distinct
points, the other two sides are divided in the same ratio.
2. (Motivate) If a line divides two sides of a triangle in the same ratio, the line is parallel to the third
side.
3. (Motivate) If in two triangles, the corresponding angles are equal, their corresponding sides are
proportional and the triangles are similar.
4. (Motivate) If the corresponding sides of two triangles are proportional, their corresponding angles
are equal and the two triangles are similar.
5. (Motivate) If one angle of a triangle is equal to one angle of another triangle and the sides including
these angles are proportional, the two triangles are similar.
6. (Motivate) If a perpendicular is drawn from the vertex of the right angle of a right triangle to the
hypotenuse, the triangles on each side of the perpendicular are similar to the whole triangle and
to each other.
7. (Prove) The ratio of the areas of two similar triangles is equal to the ratio of the squares of their
corresponding sides.
8. (Prove) In a right triangle, the square on the hypotenuse is equal to the sum of the squares on the
other two sides.
9. (Prove) In a triangle, if the square on one side is equal to sum of the squares on the other two
sides, the angles opposite to the first side is a right angle.

UNIT V: TRIGONOMETRY
6. Introduction to Trigonometry
Trigonometric ratios of an acute angle of a right-angled triangle. Proof of their existence (well defined).
Values of the trigonometric ratios of 30°, 45° and 60°.. Relationships between the ratios.
Trigonometric Identities
Proof and applications of the identity sin2 A + cos2 A = 1. Only simple identities to be given.

UNIT VI: MENSURATION


7. Areas Related to Circles
Motivate the area of a circle; area of sectors and segments of a circle. Problems based on areas and
perimeter / circumference of the above said plane figures. (In calculating area of segment of a circle,
problems should be restricted to central angle of 60° and 90° only. Plane figures involving triangles,
simple quadrilaterals and circle should be taken.)

UNIT VII: STATISTICS AND PROBABILITY


8. Probability
Classical definition of probability. Simple problems on finding the probability of an event.

(vii)
D:\EG_Mathematics_Standard-10_(Term-1)_(30-07-2021)\Open_Files\0_Prelims\0_Prelims
\ 14-Sep-2021   Bhandari  Proof-3 Reader’s Sign _______________________ Date __________

CONTENTS

1. Real Numbers ........................................................................................................................ 9

2. Polynomials ........................................................................................................................... 23

3. Pair of Linear Equations in Two Variables ........................................................................... 42

4. Coordinate Geometry ............................................................................................................ 79

5. Triangles ................................................................................................................................ 93

6. Introduction to Trigonometry ................................................................................................ 115

7. Areas Related to Circles ........................................................................................................ 128

8. Probability ............................................................................................................................. 147

• Sample Paper 1 (Issued by CBSE on 2nd September, 2021) ................................................. 158

Marking Scheme ................................................................................................................... 165

• Sample Paper 2 ..................................................................................................................... 172

• Sample Paper 3 ..................................................................................................................... 179

• OMR Sheets

(viii)
D:\EG_Mathematics_Standard-10_(Term-1)_(30-07-2021)--\Open_Files\Ch-1\Ch-1
\ 31-Jul-2021   Praveen Kumar  Proof-3 Reader’s Sign _______________________ Date __________

1 Real Numbers

Topics Covered
1. The Fundamental Theorem of Arithmetic
2. Decimal Representation of Rational Numbers

Introduction
We know that all the rational and irrational numbers together make up the collection of real numbers.
In this chapter, we shall study divisibility of integers and expressing positive integers as a product of
prime integers. Let us recall divisibility of integers and product of prime integers.
Divisibility of integers: Any positive integer a can be divided by another positive integer b in such a
way that it leaves a remainder r that is smaller than b.
Product of prime numbers: Every composite number can be expressed as a product of primes.
We shall also study some more topics in this chapter such as Fundamental Theorem of Arithmetic,
rational and irrational numbers, etc.

1. The Fundamental Theorem of Arithmetic


Fundamental Theorem of Arithmetic: Every composite number can be expressed (factorised) as a
product of primes and this expression (factorisation) is unique, apart from the order in which the prime
factors occur.
For any two positive integers a and b, HCF (a, b) × LCM (a, b) = a × b.
Example 1. The values of A and B in the following factor tree, respectively are

(a) 42, 21 (b) 21, 42 (c) 12, 10 (d) 10, 12


Solution. The factor tree with missing entries filled is given below.

Hence, option (a) is the correct answer.


Example 2. When 2120 is expressed as the product of its prime factors we get
(a) 2 × 53 × 53 (b) 23 × 5 × 53 (c) 5 × 72 × 31 (d) 52 × 7 × 33

9
D:\EG_Mathematics_Standard-10_(Term-1)_(30-07-2021)--\Open_Files\Ch-1\Ch-1
\ 31-Jul-2021   Praveen Kumar  Proof-3 Reader’s Sign _______________________ Date __________

Solution. Using the factor tree for prime factorisation, we get:

\ 2120 = 2 × 2 × 2 × 5 × 53 = 23 × 5 × 53
Hence, option (b) is the correct answer.
Example 3. [HCF × LCM] for the numbers 100 and 190 is
(a) 190 (b) 1900 (c) 19000 (d) None of these
Solution. HCF × LCM = Product of two numbers = 100 × 190 = 19,000
Hence, option (c) is the correct answer.
Example 4. The HCF of the smallest composite number and the smallest prime number is
(a) 1 (b) 2 (c) 3 (d) 5
Solution. The smallest composite number is 4 and the smallest prime number is 2.
The prime factorisation of 4 = 2 × 2 = 22 and the prime factorisation of 2 = 21
Now, the HCF of 2 and 4 is the product of smallest power of each common prime factor in the numbers.
HCF (2, 4) = 21 = 2
Hence, option (b) is the correct answer.
Example 5. On a morning walk, three persons step off together and their steps measure 40 cm, 42 cm and
45 cm, respectively. The minimum distance each should walk so that each can cover the same distance
in complete steps is
(a) 1260 cm (b) 1920 cm (c) 2242 cm (d) 2520 cm
Solution. Step measures of three persons are 40 cm, 42 cm and 45 cm.
The minimum distance each should walk so that each can cover the same distance in complete steps
is the LCM of 40 cm, 42 cm and 45 cm.
Prime factorisation of 40, 42 and 45 gives
40 = 23 × 5, 42 = 2 × 3 × 7, 45 = 32 × 5
LCM (40, 42, 45) = Product of the greatest power of each prime factor involved in the numbers
= 23 × 32 × 5 × 7 = 8 × 9 × 35 = 72 × 35 = 2520 cm.
Hence, option (d) is the correct answer.
Example 6. The LCM of two numbers is 14 times their HCF. The sum of LCM and HCF is 600. If one
number is 280, then the other number is
(a) 20 (b) 28 (c) 60 (d) 80
Solution. According to the question,
LCM + HCF = 600
Since LCM = 14 × HCF
\ 14 × HCF + HCF = 600  ⇒  15 × HCF = 600  ⇒  HCF = 600 ÷ 15 = 40
\ LCM = 600 – HCF = 600 – 40 = 560

10  Mathematics–10
D:\EG_Mathematics_Standard-10_(Term-1)_(30-07-2021)--\Open_Files\Ch-1\Ch-1
\ 31-Jul-2021   Praveen Kumar  Proof-3 Reader’s Sign _______________________ Date __________

We know that HCF (a, b) × LCM (a, b) = a × b


HCF × LCM = 40 × 560 = 80
⇒ Other number =
Given number 280
Hence, option (d) is the correct answer.
Example 7. Four bells toll at an interval of 8, 12, 15 and 18 seconds respectively. All the four begin to
toll together. The number of times they toll together in one hour excluding the one at the start will be
(a) 5 (b) 8 (c) 10 (d) 12
Solution. Four bells toll at an interval of 8, 12, 15 and 18 seconds. All the four bells begin to toll together.
To find how many times will they toll together in one hour excluding the one at the start, we first find
the LCM of the numbers 8, 12, 15 and 18.
Now, prime factorisation of the given numbers are:
8 = 2 × 2 × 2 = 23
12 = 2 × 2 × 3 = 22 × 31
15 = 3 × 5 = 31 × 51
18 = 2 × 3 × 3 = 21 × 32
LCM (8, 12, 15 and 18) = 23 × 32 × 51 = 8 × 9 × 5 = 360 sec = 6 min
\  Four bells toll together in one hour = 60 ÷ 6 = 10 times.
Hence, option (c) is the correct answer.

Exercise 1.1
A. Multiple Choice Questions (MCQs)
Choose the correct answer from the given options:
1. If two positive integers a and b are written as a = x3y2 and b = xy3; x, y are prime numbers then HCF
(a, b) is:
(a) xy (b) xy2 (c) x3y3 (d) x2y2­
2. The LCM of smallest two-digit composite number and smallest composite number is:
(a) 12 (b) 4 (c) 20 (d) 44
3. 325 can be expressed as a product of its primes as:
(a) 52 × 7 (b) 52 × 13 (c) 5 × 132 (d) 2 × 32 × 52
4. The total number of factors of a prime number is [CBSE Standard 2020]
(a) 1 (b) 0 (c) 2 (d) 3
5. The HCF and the LCM of 12, 21, 15 respectively are: [CBSE Standard 2020]
(a) 3, 140 (b) 12, 420 (c) 3, 420 (d) 420, 3
6. HCF (a, b) × LCM (a, b) equal to
(a) a + b (b) a – b (c) a × b (d) a ÷ b
7. The values of x and y in the given figure respectively are


(a) x = 84, y = 21 (b) x = 21, y = 84 (c) x = 42, y = 24 (d) x = 24, y = 42
8. If a and b are co-prime, then a2 and b2 are
(a) primes (b) composites (c) co-primes (d) None of these

Real Numbers  11
D:\EG_Mathematics_Standard-10_(Term-1)_(30-07-2021)--\Open_Files\Ch-1\Ch-1
\ 31-Jul-2021   Praveen Kumar  Proof-3 Reader’s Sign _______________________ Date __________

9. When 429 is expressed as a product of its prime factors, we get


(a) 2 × 5 × 29 (b) 33 × 13 × 1 (c) 3 × 11 × 9 (d) 3 × 11 × 13
10. The LCM of two numbers is 182 and their HCF is 13. If one of the numbers is 26, the other number is
(a) 31 (b) 71 (c) 61 (d) 91
11. If xy = 180 and HCF(x, y) = 3, then the LCM(x, y) is
(a) 30 (b) 60 (c) 45 (d) 50
12. When 156 is expressed as the product of primes, we get
(a) 22 × 3 × 13 (b) 22 × 3 × 11 (c) 2 × 32 × 13 (d) 2 × 32 × 11
13. The HCF and LCM of 404 and 96 respectively are
(a) 2, 9696 (b) 4, 9696 (c) 8, 3636 (d) 10, 2020
14. The values of p and q such that the prime factorisation of 2520 is expressible as 23 × 3p × q × 7
respectively are
(a) 2, 3 (b) 3, 5 (c) 2, 5 (d) 5, 7
15. If HCF of 65 and 117 is expressible in the form 65n – 117, then the value of n is
(a) 1 (b) 2 (c) 3 (d) 5
16. The LCM of 150 and 200 is
(a) 320 (b) 400 (c) 550 (d) 600
17. 3 bells ring at an interval of 4, 7 and 14 minutes. All three bells rang at 6 am. When the three bells
will ring together next?
(a) 6:20 am (b) 6:24 am (c) 6:28 am (d) 6:30 am
18. Three bells toll at intervals of 9, 12, 15 minutes respectively. If they start tolling together, what time
will they next toll together?
(a) After 1 hour (b) After 2 hours (c) After 2½ hours (d) After 3 hours
19. The LCM and the HCF of 15, 18, 45 respectively are
(a) 3, 30 (b) 4, 40 (c) 5, 50 (d) 3, 90
20. The largest number which on dividing 1251, 9377 and 15628 leaves ramainders 1, 2 and 3 respectively
is
(a) 450 (b) 575 (c) 625 (d) 750
21. At an international airport, planes take off from five different runways at 3, 4, 8, 12 and 15 minutes
intervals. At 7:30 a.m., planes took off from all five runways simultaneously. When will five planes
take off together again?
(a) 7:45 am (b) 8:15 am (c) 9:00 am (d) 9:30 am
22. In a seminar the number of participants in Hindi, English and Mathematics are 60, 84 and 108
respectively. The minimum number of rooms required if in each room the same number of participants
are to be seated and all of them being in the same subject is
(a) 16 (b) 14 (c) 12 (d) 10
B. Assertion-Reason Type Questions
In the following questions, a statement of assertion (A) is followed by a statement reason (R). Choose
the correct choice as:
(a) Both assertion (A) and reason (R) are true and reason (R) is the correct explanation of assertion (A).
(b) Both assertion (A) and reason (R) are true but reason (R) is not the correct explanation of assertion
(A).
(c) Assertion (A) is true but reason (R) is false.
(d) Assertion (A) is false but reason (R) is true.
1. Assertion (A): The number 6n, n being a natural number, ends with the digit 5.
Reason (R): The number 9n cannot end with digit 0 for any natural number n.
2. Assertion (A): If m and n are odd positive integers, then m2 + n2 is even but not divisible by 4.
Reason (R): 3 × 5 × 7 + 7 is a composite number.

12  Mathematics–10
D:\EG_Mathematics_Standard-10_(Term-1)_(30-07-2021)--\Open_Files\Ch-1\Ch-1
\ 31-Jul-2021   Praveen Kumar  Proof-3 Reader’s Sign _______________________ Date __________

Case Study Based Questions


I.
To enhance the reading skills of grade X students, the school nominates you and two of your friends
to set up a class library. There are two sections — section A and section B of grade X. There are 32
students in section A and 36 students in section B.


1. What is the minimum number of books you will acquire for the class library, so that they can be
distributed equally among students of section A or section B?
(a) 144 (b) 128 (c) 288 (d) 272
2. If the product of two positive integers is equal to the product of their HCF and LCM is true then, the
HCF (32 , 36) is:
(a) 2 (b) 4 (c) 6 (d) 8
3. 36 can be expressed as a product of its primes as:
(a) 22 × 32 (b) 21 × 33 (c) 23 × 31 (d) 20 × 30
4. 7 × 11 × 13 × 15 + 15 is a:
(a) Prime number (b) Composite number
(c) Neither prime nor composite (d) None of the above
5. If p and q are positive integers such that p = ab2 and q = a2b, where a, b are prime numbers, then the
LCM (p, q) is
(a) ab (b) a2b2 (c) a3b2 (d) a3b3
II. Traffic Lights (or traffic signals) are lights used to control movement of traffics.
They are installed on roads at intersections and crossings. The different colours
of light tell drivers what to do.
The traffic lights at different road crossings change after every 48 seconds, 72
seconds and 108 seconds respectively.
1. 108 can be expressed as a product of its primes as
(a) 23 × 32 (b) 23 × 33
(c) 2 × 3 2 2
(d) 22 × 33
2. The HCF of 48, 72, 108 is
(a) 18 (b) 16 (c) 12 (d) 10
3. The LCM of 48, 72, 108 is
(a) 520 (b) 432 (c) 396 (d) 420
4. If all the traffic lights change simultaneously at 8:20:00 hrs, they will again change simultaneously at
(a) 8:27:12 hrs (b) 8:32:24 hrs (c) 8:40:08 hrs (d) 8:24:24 hrs
5. The [HCF × LCM] for the numbers 48, 72, 108 is
(a) 2472 (b) 3680 (c) 4090 (d) 5184

Real Numbers  13
D:\EG_Mathematics_Standard-10_(Term-1)_(30-07-2021)--\Open_Files\Ch-1\Ch-1
\ 31-Jul-2021   Praveen Kumar  Proof-3 Reader’s Sign _______________________ Date __________

III. The Army Day is celebrated on 15th January every year in India. The day is celebrated in the form of
parades and other military shows in the national
capital New Delhi as well as in all headquarters
of army.
Parade I: An Army contingent of 616 members
is to march behind an army band of 32 members
in parade. The two groups are to march in the
same number of columns.
Parade II: An Army contingent of 1000
members is to march behind an army band of
56 members in parade. The two groups are to
march in the same number of columns.
Refer to Parade I
1. The expression of 616 as a product of its prime numbers is
(a) 22 × 141 × 111 (b) 23 × 71 × 111 (c) 24 × 71 × 111 (d) None of these.
2. The LCM of 32 and 616 is
(a) 1248 (b) 1836 (c) 2464 (d) 3016
3. What is the maximum number of columns in which they can march?
(a) 6 (b) 8 (c) 10 (d) 12
Refer to Parade II
4. The HCF of numbers 1000 and 56 is
(a) 2 (b) 4 (c) 6 (d) 8
5. Which of the following is the correct expression of the number 1000 as a product of its prime numbers?
(a) 23 × 53 (b) 103 (c) 8 × 125 (d) 8
IV. A Mathematics Exhibition is being conducted in your school and one of your friends is making a model
of a factor tree. He has some difficulty and asks for your help in completing a quiz for the audience.
Observe the given factor tree and answer the following questions:

5 2783

y 253

11 z

1. What will be the value of x?


(a) 15005 (b) 13915 (c) 56920 (d) 17429
2. What will be the value of y?
(a) 23 (b) 22 (c) 11 (d) 19

14  Mathematics–10
D:\EG_Mathematics_Standard-10_(Term-1)_(30-07-2021)--\Open_Files\Ch-1\Ch-1
\ 31-Jul-2021   Praveen Kumar  Proof-3 Reader’s Sign _______________________ Date __________

3. What will be the value of z?


(a) 22 (b) 23 (c) 17 (d) 19
4. According to Fundamental Theorem of Arithmetic, 13915 is a
(a) Composite number (b) Prime number
(c) Neither prime nor composite (d) Even number
5. The prime factorisation of 13915 is
(a) 5 × 112 × 132 (b) 5 × 113 × 232 (c) 5 × 112 × 23 (d) 5 × 112 × 132

Answers and Hints


A. Multiple Choice Questions (MCQs) 16. (d) Given numbers are 150 and 200.
1. (b) xy 2
2. (c) 20 \ 150 = 2 × 3 × 5 × 5 2 150
3. (b) 52 × 13 4. (c) 2 = 2 × 3 × 52
3 75
5. (c) 3, 420 6. (c) a × b 5 25
7. (b) x = 21, y = 84 8. (c) co-primes 5
9. (d) 429 = 3 × 11 × 13  3 429 \ 200 = 2 × 2 × 2 × 5 × 5 2 200
11 143 = 23 × 52 2 100
13 13 2 50
1 5 25
10. (d) For any two positive integers a and b, 5
a × b = HCF (a, b) × LCM (a, b) LCM of 150 and 200 = 23 × 3 × 52
fi 26 × b = 13 × 182 = 8 × 3 × 25 = 600
13 × 182 17. (c) 4 = 2 × 2
fi b = = 91 7 = 7 × 1
26
Hence, the other number is 91. 14 = 2 × 7
11. (b) LCM (x, y) × HCF (x, y) = x × y LCM = 2 × 2 × 7 = 28
fi LCM (x, y) × 3 = 180 The three bells will ring together again at
fi LCM = 60 6 : 28 am
18. (d) After 3 hrs
12. (c) 156 = 22 × 3 × 13
Hint: Solution is same as the example 7. Refer
13. (b) 404 = 2 × 2 × 101 = 22 × 101
to it.
96 = 2 × 2 × 2 × 2 × 2 × 3 = 25 × 3
19. (d) 3, 90
\ HCF of 404 and 96 = 22 = 4
20. (c) It is given that on dividing 1251 by the
LCM of 404 and 96 = 101 × 25 × 3 = 9696
required number, there is a remainder.
14. (c) Prime factorisation of 2520 is given by This means that 1251 – 1 = 1250 is exactly
2520 = 23 × 32 × 5 × 7 ...(i) divisible by the required number.
Given 2520 = 2 × 3 × q × 7
3 p
...(ii) Similarly, required number is a factor of
On comparing (i) and (ii), we get 9377 – 2 = 9375 and 15628 – 3 = 15625.
p = 2 and q = 5 Clearly, required number is the HCF of 1250,
15. (b) Factors of 9375 and 15625.
117 = 32 × 13 Using the factor tree, we get the prime
65 = 5 × 13 factorisation of 1250, 9375 and 15625 as
\ HCF of 65 and 117 is 13. follows
ATQ, 65n – 117 = 13 1250 = 2 × 54, 9375 = 3 × 55 and 15625 = 56
⇒ 65n = 130 ∴ HCF of 1250, 9375 and 15625 is 54 = 625
⇒ n = 2 Hence required number = 625.

Real Numbers  15
D:\EG_Mathematics_Standard-10_(Term-1)_(30-07-2021)--\Open_Files\Ch-1\Ch-1
\ 31-Jul-2021   Praveen Kumar  Proof-3 Reader’s Sign _______________________ Date __________

21. (d) Planes take off from five different runways B. Assertion-Reason Type Questions
at 3, 4, 8, 12, 15 minutes intervals. To find 1. (d) Assertion (A) is false but reason (R) is true.
when all planes will take off together again
if they take off at 7 : 30 a.m. once, we must 2. (b) Both assertion (A) and reason (R) are true
find the LCM of 3, 4, 8, 12 and 15. but reason (R) is not the correct explanation
Now 3 = 3 × 1 of assertion (A).
4 = 22
8 = 23 Case Study Based Questions
12 = 22 × 3 I. 1. (c) 288 2. (b) 4
15 = 3 × 5
\  LCM = 23 × 3 × 5 3. (a) 2 × 3
2 2

= 40 × 3 = 120 minutes 4. (b) composite number


= 2 hours 5. (b) a2b2
 fi Plane will took off together again after 2 II. 1. (d) 22 × 33 2. (c) 12
hours
i.e. 9:30 a.m. 3. (b) 432 4. (a) 8:27:12 hrs
22. (c)  Now, number of rooms required for 5. (d) 5184
60 III. 1. (b) 23 × 71 × 111 2. (c) 2464
participants in Hindi = =5
12 3. (b) 8 4. (d) 8
Number of rooms required for participants
84 5. (a) 2 × 5
3 3

in English = =7 IV. 1. (b) 13915 2. (c) 11


12
Number of rooms required for participants 3. (b) 23
108 4. (a) Composite number
in Mathematics = =9
12 5. (c) 5 × 112 × 23
fi Number of rooms needed = 21.

2. Decimal Representation of Rational Numbers


We know that rational numbers and irrational numbers together are known as real numbers.
Irrational Number
p
A number is called irrational if it cannot be written in the form , where p and q are integers and q ≠ 0.
q
Examples of irrational numbers are 2 , 3 , 15 , p, etc.
Theorem 1. Let p be a prime number. If p divides a2, then p divides a, where a is a positive integer.
This theorem is based on Fundamental Theorem of Arithmetic.
Rational Numbers and their Decimal Expansions
p
A number is called rational if it can be expressed in the form q , where p and q are both integers and q π 0.
2 10
Examples of rational numbers are , , 9, 3.5, 2.045, etc.
5 15
p
Theorem 2. Let x = , where p and q are co-primes, be a rational number whose decimal expansion terminates.
q
Then, the prime factorisation of q is of the form 2n5m, where n, m are non-negative integers.
p
Theorem 3. Let x = be a rational number, such that the prime factorisation of q is of the form 2n5m, where
q
n, m are non-negative integers. Then x has a decimal expansion which terminates.

16  Mathematics–10
D:\EG_Mathematics_Standard-10_(Term-1)_(30-07-2021)--\Open_Files\Ch-1\Ch-1
\ 31-Jul-2021   Praveen Kumar  Proof-3 Reader’s Sign _______________________ Date __________

p
Theorem 4. Let x = be a rational number such that the prime factorisation of q is not of the form 2n5m,
q
where n, m are non-negative integers. Then, x has a non-terminating repeating decimal expansion.
Example 1. The decimal expansions (without actual division) and its nature (terminating or non-
987
terminating) of will be
10500
__
(a) 0.094, non-terminating (b) 0.094, terminating
(c) 0.094, non-terminating (d) 0.049, terminating
987 47 × 3 × 7 47
Solution. We have = =
10500 500 × 3 × 7 500
Here, the prime factorisation of q = 500 is 2 × 2 × 5 × 5 × 5 = 22 × 53 which is of the form 2n5m, where
n, m are non-negative integers.
987
Hence, will have a terminating decimal expansion.
10500
987 47 47 47 × 2 = 94 = 0.094
Now, = = 2 3=
10500 500 2 ×5 (2 × 5)3 1000
987
Hence, decimal expansion of terminates after 3 places.
10500
Hence, option (b) is the correct answer.

43
Example 2. The decimal expansion of the rational number 4 3 will terminate after how many places
25
of decimals?
(a) 2 places (b) 3 places (c) 4 places (d) 5 places
p
Solution. Here we have a rational number of the form and the prime factorisation of q is of the
q
form 2 5 , where n, m are non-negative integers, so decimal expansion of given rational number
n m

terminates.
43 43 × 5 215
Now, = 4 4 = 4 = 0.0215
2453 25 10
So, given rational number will terminate after 4 decimal places.
Hence, option (c) is the correct answer.
Example 3. Rational number between 2 and 3 is [Delhi 2019]
(a) 1.5 (b) 1.6 (c) 1.7 (d) All of these
Solution. We have 2 = 1.4142135.... and 3 = 1.7320508...
Since every terminating decimal or repeating decimal represents a rational number.
So, 1.666666... is a rational number between 2 and 3.
Also 1.5, 1.6, 1.7 are rational numbers between 2 and 3.
Hence, option (d) is the correct answer.

Real Numbers  17
D:\EG_Mathematics_Standard-10_(Term-1)_(30-07-2021)--\Open_Files\Ch-1\Ch-1
\ 31-Jul-2021   Praveen Kumar  Proof-3 Reader’s Sign _______________________ Date __________

Exercise 1.2
A. Multiple Choice Questions (MCQs)
Choose the correct answer from the given options:
1. A rational number can be expressed as a terminating decimal if the denominator has factors
(a) 2, 3 or 5 (b) 2 or 3 (c) 3 or 5 (d) 2 or 5
p
2. Rational number , q ≠ 0, will be terminating decimal if the prime factorisation of q is of the form
q
(m and n are non-negative integers)
(a) 2m × 3n (b) 2m × 5n (c) 3m × 5n (d) 3m × 7n
3. Which of the following is the decimal expansion of an irrational number?
(a) 4.561 (b) 0.12 (c) 5.010010001… (d) 6.03
11
4. The decimal representation of 3 will [CBSE Standard SP 2019-20]
2 ×5
(a) terminate after 1 decimal place (b) terminate after 2 decimal places
(c) terminate after 3 decimal places (d) not terminate
5. 2.35 is
(a) an integer (b) a rational number (c) an irrational number (d) a natural number
2 45 + 3 20
6. on simplification gives [AI]
2 5
(a) 3 (b) 4 (c) 6 (d) 8
7
7. The rational number will have
75
(a) Terminating after 2 place (b) Non-terminating but repeating
(c) Non-terminating and non-repeating (d) Terminating after 4 places
27
8. After how many places of decimal the decimal form of the number 3 4 2 will terminate? [AI]
2 ·5 ·3
(a) 2 places (b) 3 places (c) 4 places (d) 5 places
9. A rational and an irrational number lying between 0.25 and 0.32 are respectively.
(a) 0.30, 0.3010203040... (b) 0.20, 0.2010203040...
(c) 0.33, 0.3510203040... (d) None of these
14587
10. The decimal representation of will terminate after how many decimal places?
(a) One decimal places 21 × 54 (b) Two decimal places
(c) Three decimal places (d) Four decimal places
11. The decimal expansion (without actual division) and its nature (terminating or non-terminating) of
17
will be
8
(a) Terminating after 2 places (b) Non-terminating but repeating
(c) Non-terminating but non-repeating (d) Terminating after 3 places
12. The decimal expansion (without actual division) and its nature (terminating or non-terminating) of
15
will be
1600
(a) Terminating after 6 places (b) Non-terminating but repeating
(c) Non-terminating and non-repeating (d) Terminating after 2 places
13. The decimal expansion (without actual division) and its nature (terminating or non-terminating) of
64
will be
455
(a) Terminating after 2 places (b) Non-terminating but repeating
(c) Non-terminating but non-repeating (d) Terminating after 3 places

18  Mathematics–10
D:\EG_Mathematics_Standard-10_(Term-1)_(30-07-2021)--\Open_Files\Ch-1\Ch-1
\ 31-Jul-2021   Praveen Kumar  Proof-3 Reader’s Sign _______________________ Date __________

3
14. The 2n5m (where n and m are non-negative integers) from of denominator of and its decimal
5
expansion respectively are
(a) 20 × 51, 0.6 (b) 21 × 50, 0.5 (c) 21 × 51, 0.6 (d) 22 × 50, 0.8
13
15. The 2 5 (where n and m are non-negative integers) form of denominator of
n m
and its decimal
80
expansion respectively are
(a) 23 × 52, 0.1248 (b) 22 × 53, 0.1698 (c) 24 × 51, 0.1625 (d) None of these
127
16. The 2 5 (where n and m are non-negative integers) form of denominator of
n m
, respectively are
500
(a) 22 × 52, 0.254 (b) 23 ×____
52, 0.364 (c) 23 × 52, 0.648 (d) 22 × 53, 0.254
a
17. The form of the number 0.3178 is
b
135 985 635
(a) (b) (c) (d) None of these
1449 2163 1998
B. Assertion-Reason Type Questions
In the following questions, a statement of assertion (A) is followed by a statement reason (R). Choose
the correct choice as:
(a) Both assertion (A) and reason (R) are true and reason (R) is the correct explanation of assertion (A).
(b) Both assertion (A) and reason (R) are true but reason (R) is not the correct explanation of assertion
(A).
(c) Assertion (A) is true but reason (R) is false.
(d) Assertion (A) is false but reason (R) is true.
1. Assertion (A): The sum or difference of a rational number and an irrational number is irrational.
Reason (R): Negative of an irrational number is rational.
2. Assertion (A): 5 + 3 is an irrational number.
Reason (R): The sum or difference of a rational and an irrational number is always irrational.

Case Study Based Questions


I.
Richard Paul and Linda Elder state that “Thinking is not driven by answers but by questions”. Keeping
this idea in mind, mathematics teacher of class X of Blue Marble School decided to develop interest
and motivate students to become actively involved in the concept he asked some questions. The teacher
drew a flow chart for real numbers family on the board as shown below. The chart for the set of real
numerals includes all the types of numbers. Observe the chart carefully.

  Real Numbers R

  Rational Numbers Q   Irrational Numbers

Non-Integer  Integers Z
Rational Numbers

Negative Numbers   Whole Numbers W

Zero   Natural Numbers N

Real Numbers  19
D:\EG_Mathematics_Standard-10_(Term-1)_(30-07-2021)--\Open_Files\Ch-1\Ch-1
\ 31-Jul-2021   Praveen Kumar  Proof-3 Reader’s Sign _______________________ Date __________

1. The sum of a rational and irrational number is


(a) Rational (b) Irrational (c) both of the above (d) none of the above
144
2. The decimal expansion of the number 2 3
2 ×5 ×7
(a) Terminating (b) non-terminating
(c) non-terminating non-repeating (d) non-terminating repeating
3. A rational number in its decimal expansion is 327.7081. What would be the prime factors of q when
the number is expressed in the p/q form?
(a) 2 and 3 (b) 3 and 5 (c) 2, 3 and 5 (d) 2 and 5
4. Which of the following is not irrational?
( )
(a) 3 + 7 (b) 3 − 7 (c) 3 + 7 3 − 7 (d) 3 7 ( ) ( )( )
14587
5. The decimal expansion of the rational number will terminate after:
1250
(a) one decimal place (b) two decimal places
(c) three decimal places (d) four decimal places
II. An object when thrown upwards reaches a certain height and then falls downwards. Whenever an object
falls towards the earth, acceleration is involved. This acceleration is due to the earth’s gravitational
force. Therefore, this acceleration is called the acceleration due to the gravitational force of the earth
(or acceleration due to gravity). It is denoted by g. The unit of g is the same as that of acceleration, that
is, m/s2. Its value is 9.8 m/s2 on Earth. When discussing the acceleration of gravity, it was mentioned
that the value of g is dependent upon location. There are slight variations in the value of g on earth’s
surface. The following acceleration of gravity values can be calculated for the various planets.
Planet g (m/s2)
Mercury 3.61
Venus 8.83
Mars 3.75
Jupiter 26.0
Saturn 56/5
Uranus 21/2
Neptune 13.3
1. Which of the following is the decimal expansion of a rational number?
(a) 26.0 (b) 8.83 (c) 3.75 (d) All of these
2. The p/q form of the number 3.75 will be
(a) 11/4 (b) 17/2 (c) 15/4 (d) None of these
3. An irrational number lying between 8.83 and 11.2 is
(a) 8.8010203040... (b) 8.9010203040... (c) 11.10203040... (d) None of these
4. The decimal expansion of the rational number 56/5 will has
(a) One decimal place (b) Two decimal places
(c) Three decimal places (d) Four decimal places
5. The 2 × 5 form of denominator 21/2 and its decimal expansion respectively are
n m

(a) 20 × 51 (b) 21 × 51 (c) 20 × 50 (d) None of these

20  Mathematics–10
D:\EG_Mathematics_Standard-10_(Term-1)_(30-07-2021)--\Open_Files\Ch-1\Ch-1
\ 31-Jul-2021   Praveen Kumar  Proof-3 Reader’s Sign _______________________ Date __________

Answers and Hints


A. Multiple Choice Questions (MCQs) 17

Hence, has a terminating decimal
1. (d) 2 or 5 2. (b) 2m × 5n 8
3. (c) 5.010010001… expansion with 3 decimal places.
4. (c) terminates after 3 decimal places 15 3×5 3
12. (a) = 6 = 6
5. (b) a rational number 1600 2 ×5 2
2 × 51
2 45 + 3 20
6. (c) = 2 9×5 + 3 4×5 Denominator 26 × 51 is of the form 2n5m.
2 5 2 5 3 55
Now, 6 1 × 5
2×3 5 +3×2 5 2 ×5 5
=
2 5 3 × 55 3 × 55
= 6 6
=
6 5+6 5 12 5 2 ×5 106
= =6 = 15
2 5 2 5  Hence has a terminating decimal
1600
7 7 expansion with 6 decimal places.
7. (b) Since = 2
75 5 × 3 64 26
The denominator 52 × 3 which is not of the 13. (b) =
455 5 × 7 × 13
form 2m ◊ 5n.
Clearly, 64 and 455 are co-prime.
7 64

\  will have a non-terminating repeating
75 \  is in its simplest form.
455
decimal expansion. Denominator 455 = 5 × 7 × 13 is not of the
27 27 2
8. (c) 3 4 2 = 3 4 × form 2n5m.
2 $5 $3 2 $5 $9 2
 Hence 64 has a non-terminating repeating
3× 2 6 455
= 4 4
=
2 ⋅5 (2 × 5) 4 decimal expansion.
6 14. (a) 20 × 51, 0.6
= = 0.0006
10 4 15. (c) 24 × 51, 0.1625
27 16. (d) 22 × 53, 0.254

Then, 3 4 2 will terminate after 4
2 $5 $3 17. (c) Let, x = 0.3178
places.
9. (a) Rational number = 0.30 fi 10x = 3.178 = 3.178178... ...(i)
Irrational number = 0.3010203040… fi 10x × 1000 = 3.178178... × 1000
Or any other correct rational and irrational fi 10,000x = 3178.178178... ...(ii)
number Subtracting (i) from (ii), we get
10. (d) Four decimal places 9990x = 3175
17 17 17 3175
11. (d) = = 3 0 fi x =
8 23
2 ×5 9990
Clearly, 17 and 8 are co-prime. 635
fi x =
17 1998
\  is in the simplest form. Denominator B. Assertion-Reason Type Questions
8
1. (b) Both assertion (A) and reason (R) are true
= 23 × 50 is of the form 2n 5m.
but reason (R) is not the correct explanation
17 17 × 125 of assertion (A).
Also, =
8 8 × 125 2. (a) Both assertion (A) and reason (R) are true
2125 and reason (R) is the correct explanation of
= = 2.125
1000 assertion (A)

Real Numbers  21
D:\EG_Mathematics_Standard-10_(Term-1)_(30-07-2021)--\Open_Files\Ch-1\Ch-1
\ 31-Jul-2021   Praveen Kumar  Proof-3 Reader’s Sign _______________________ Date __________

Case Study Based Questions


I. 1. (b) Irrational 2. (d) Non-terminating repeating 3. (d) 2 and 5
(
4. (c) 3 + 7 3 − 7 5.
)( )
(d) four decimal places
II. 1. (d) All of these 2. (c) 15/4 3. (c) 11.10203040...
4. (a) One decimal place 5. (b) 21 × 51

Experts’ Opinion
Following types of questions are very important for examination point of view. So, students are
advised to revise them thoroughly.
1. Applications of Fundamental Theorem of Arithmetic.
2. Exploring rational numbers and their decimal expansions.

QUICK REVISION NOTES


•• The Fundamental Theorem of Arithmetic: Every composite number can be expressed (factorised)
as a product of primes, and this expression (factorisation) is unique, apart from the other in which the
prime factors occur.
•• For any two positive integers a and b, HCF (a, b) × LCM (a, b) = a × b.
•• Let p be a prime number. If p divides a2, then p divides a, where a is a positive integer.
p
•• Let x = , where p and q are co-primes, be a rational number whose decimal expansion terminates.
q
Then, the prime factorisation of q is of the form 2n5m, where n, m are non-negative integers.
p
•• Let x = be a rational number, such that the prime factorisation of q is of the form 2n5m, where n, m
q
are non-negative integers. Then, x has a decimal expansion which terminates.
p
•• Let x = be a rational number such that the prime factorisation of q is not of the form 2n5m, where n,
q
m are non-negative integers. Then, x has a non-terminating repeating (recurring) decimal expansion.

COMMON ERRORS
Errors Corrections
(i) Incorrectly using the result for three numbers (i) The correct results are:
HCF (p, q, r) × LCM (p, q, r) = p × q × r pqr HCF( p, q, r )
LCM (p, q, r) =
HCF( p, q ) ⋅ HCF(q, r ) ⋅ HCF( p, r )
pqr LCM ( p, q, r )
HCF (p, q, r) =
LCM ( p, q ) ⋅ LCM (q, r ) ⋅ LCM ( p, r )
(ii) Misunderstanding that the prime factors of (ii) If any one of 2 or 5 occurs, then also it has terminating
denominator should have both 2 and 5 for decimal expansion.
the decimal expansion to be terminating.
p p
(iii) Deciding incorrectly about a number (iii) First make co-prime and then decide whether it is
q q
to have terminating or non-terminating terminating or non-terminating expansion.
expansion just by looking at the denominator
without making p and q co-prime.

22  Mathematics–10
D:\EG_Mathematics_Standard-10_(Term-1)_(30-07-2021)--\Open_Files\Ch-2\Ch-2
\ 31-Jul-2021   Praveen Kumar  Proof-3 Reader’s Sign _______________________ Date __________

2 Polynomials

Topics Covered
1. Geometrical Meaning of the Zeroes of a Polynomial
2. Relationship between Zeroes and Coefficients of a Polynomial

Introduction
Polynomial: An expression of the form
p(x) = anxn + an – 1xn – 1 + ... + a1x + a0,
where n is a non-negative integer, a1, a2, ..., an are constants (real numbers) and an π 0, is called a
polynomial in x of degree n.
Degree: If p(x) is a polynomial in x, the highest power of x in p(x) is called the degree of the polynomial
p(x).
Type of Polynomials: Polynomials of degree 1, 2 and 3 are called linear, quadratic and cubic polynomials
respectively.

1. Geometrical Meaning of the Zeroes of a Polynomial


Zeroes of a Polynomial: A real number ‘a’ is said to be a zeroes of a polynomial p(x) if p(a) = 0.
Note: A polynomial may have no zero, one or more than one zeroes.
If p(a) ≠ 0, then ‘a’ is not a zero of the polynomial p(x).
Example 1. The value of k for which (–4) is a zero of the polynomial x2 – x – (2k + 2) is
(a) 2 (b) –6 (c) 9 (d) 8
Solution. Given that –4 is a zero of the polynomial p(x) = x2 – x – (2k + 2).
\ p(–4) = 0  ⇒ (–4)2 + 4 – 2k – 2 = 0
⇒ 16 + 4 – 2k – 2 = 0  ⇒  18 = 2k  ⇒  k = 18 ÷ 2 = 9
Thus, for k = 9, –4 is a zero of the given polynomial.
Hence, option (c) is the correct answer.
Example 2. The solution of x2 + 6x + 9 = 0 is
(a) –1 (b) 3 (c) –3 (d) 1
Solution. Let p(x) = x2 + 6x + 9
\ p(– 3) = (– 3)2 + 6 × (– 3) + 9 = 9 + (– 18) + 9 = 18 – 18 = 0
\ x = –3 is a solution of x2 + 6x + 9 = 0.
Hence, option (c) is the correct answer.
Example 3. The number of polynomials having zeroes as –2 and 5 is
(a) 1 (b) 2 (c) 10 (d) infinite
Solution. As –2 and 5 are zeroes of the polynomial p(x), so (x + 2) and (x – 5) are the factors of p(x).
\ p(x) = k(x + 2) (x – 5)  ⇒  p(x) = k(x2 – 3x – 10), where k is a real number.
Now for different values of k, we get different polynomials.
Thus, the number of polynomials having zeroes as –2 and 5 is infinite.
Hence, option (d) is the correct answer.

23
D:\EG_Mathematics_Standard-10_(Term-1)_(30-07-2021)--\Open_Files\Ch-2\Ch-2
\ 31-Jul-2021   Praveen Kumar  Proof-3 Reader’s Sign _______________________ Date __________

Example 4. The graph of y = f (x) is given in figure (i), for some polynomial Y
f (x). The number of zeroes of f (x) is
(a) 1 (b) 2
(c) 3 (d) many X′ X
Solution. Polynomial f (x) has 3 zeroes as graph intersects the x-axis at O

three distinct points.


Y′
Hence, option (c) is the correct answer.
Figure (i)
Example 5. The graph of y = f (x) is given in figure (ii). How many zeroes Y
are there of f (x)?
(a) 0 (b) 1
(c) 2 (d) many X′ X
Solution. We observe that the graph of y = f (x) intersects x-axis in one
point only. Therefore, the number of zeroes of f (x) is one.
Hence, option (b) is the correct answer. Y′
Figure (ii)
Example 6. The graph of y = f (x) is given in figure (iii). How many zeroes Y
are there of f (x)?
(a) 1 (b) 2
(c) 3 (d) None X′ X
Solution. The given graph y = f (x) does not intersect x-axis. So, it has
no zeroes.
Hence, option (d) is the correct answer. Y′
Figure (iii)

Exercise 2.1
A. Multiple Choice Questions (MCQs)
Choose the correct answer from the given options:
1. Which of the following is not the graph of a quadratic polynomial?

(a) (b) (c) (d)

The zeroes of the polynomial x2 – 3x – m(m + 3) are


2.
(a) m, m + 3 (b) –m, m + 3 (c) m, –(m + 3) (d) –m, –(m + 3)
3.
The graph of a polynomial is shown in figure, then the number of its zeroes is
 [CBSE Standard 2020]
(a) 3 (b) 1
(c) 2 (d) 4

The value of p, for which (–4) is a zero of the polynomial x2 – 2x – (7p + 3) is


4.
(a) 0 (b) 2 (c) 3 (d) None of these

24  Mathematics–10
D:\EG_Mathematics_Standard-10_(Term-1)_(30-07-2021)--\Open_Files\Ch-2\Ch-2
\ 31-Jul-2021   Praveen Kumar  Proof-3 Reader’s Sign _______________________ Date __________

5.
The graph of y = p(x), where p(x) is a polynomial in variable x, is as follows:


The number of zeroes of p(x) is
(a) 2 (b) 5 (c) 6 (d) None of these
6. If 2 is a zero of the polynomial ax2 – 2x, then the value of ‘a’ is
(a) 3 (b) 1 (c) 2 (d) 5
7. A solution of 2x2 + 5x – 3 = 0 is
(a) –3 (b) 3 (c) –5 (d) 5
8. If 1 is a zero of polynomial p(x) = ax2 – 3(a – 1) – 1, then the value of a is
(a) 0 (b) 1 (c) 3 (d) None of these
9. If the graph of a polynomial intersects the x-axis at exactly two points, is it necessarily a
(a) quadratic polynomial (b) cubic polynomial
(c) linear polynomial (d) None of these
What number should be added to the polynomial x2 – 5x + 4, so that 3 is the zero of the polynomial?
10.
(a) 0 (b) 1 (c) 2 (d) 3
For what value of k, (– 4) is a zero of p(x) = x2 – x – (2k – 2)?
11.
(a) 8 (b) 11 (c) 13 (d) 15
12.
A teacher asked 10 of his students to write a polynomial in one variable on a paper and then to handover
the paper. The following were the answers given by the students:
5 1
2x + 3, 3x2 + 7x + 2, 4x3 + 3x2 + 2, x3 + 3x + 7, 7x + 7 , 5x3 – 7x + 2, 2x2 + 3 – , 5x – ,
x 2
1
ax3 + bx2 + cx + d, x + .
x
How many of the above ten, are not polynomials?
(a) 1 (b) 2 (c) 3 (d) 4
13.
How many of the above ten (in question 12), are quadratic polynomials?
(a) 0 (b) 1 (c) 2 (d) 3
2
14.
The zeroes of the polynomial p(y) = 5 5 y + 30 y + 8 5 are
−3 −7 −2 −4 3 3 2 4
(a) , (b) , (c) , (d) ,
5 11 5 5 5 11 5 5
15. If the zero of polynomials 3x2 – px + 2 and 4x2 – qx – 10 is 2, the value of 2p –3q is
(a) 3 (b) 5 (c) 7 (d) –5
B. Assertion-Reason Type Questions
In the following questions, a statement of assertion (A) is followed by a statement reason (R). Choose
the correct choice as:
(a) Both assertion (A) and reason (R) are true and reason (R) is the correct explanation of assertion (A).
(b) Both assertion (A) and reason (R) are true but reason (R) is not the correct explanation of assertion (A).
(c) Assertion (A) is true but reason (R) is false.
(d) Assertion (A) is false but reason (R) is true.
1. Assertion (A): x2 + 4x + 5 has two zeroes.
Reason (R): A quadratic polynomial can have at the most two zeroes.
2. Assertion (A): A quadratic polynomial whose zeroes are 5 + 2 and 5 – 2 is x2 – 10 x + 23.
Reason (R): If a and b are zeroes of the quadratic polynomial p(x), then p(x) = x2 – (a + b) x + ab.

Polynomials  25
D:\EG_Mathematics_Standard-10_(Term-1)_(30-07-2021)--\Open_Files\Ch-2\Ch-2
\ 31-Jul-2021   Praveen Kumar  Proof-3 Reader’s Sign _______________________ Date __________

Case Study Based Questions


I.
A highway underpass is parabolic in shape.

w1
w
Parabola: A parabola is the graph that results from
p(x) = ax2 + bx + c. Parabolas are symmetric about a vertical
line known as the Axis of Symmetry. The Axis of Symmetry
runs through the maximum or minimum point of the parabola
which is called the vertex. [CBSE Standard SP 2020-21]
1. If the highway overpass is represented by x2 – 2x – 8, then its
zeroes are
(a) (2, –4) (b) (4, –2) (c) (–2, –2) (d) (–4, –4)
2. The highway overpass is represented graphically. Zeroes of a polynomial can be expressed graphically.
Number of zeroes of polynomial is equal to number of points where the graph of polynomial
(a) intersects x-axis (b) intersects y-axis
(c) intersects y-axis or x-axis (d) None of these
3. Graph of a quadratic polynomial is a
(a) straight line (b) circle (c) parabola (d) ellipse
4. The representation of Highway Underpass whose one zero is 6 and sum of the zeroes is 0, is
(a) x2 – 6x + 2 (b) x2 – 36 (c) x2 – 6 (d) x2 – 3
  2
5. The number of zeroes that polynomial f (x) = (x – 2) + 4 can have is:
(a) 1 (b) 2 (c) 0 (d) 3
II.
Due to heavy storm an electric wire got
Y
bent as shown in the figure. It followed
a mathematical shape. Answer the 6
following questions below: 5
1. Name the shape in which the wire is bent. 4
(a) spiral (b) ellipse 3
(c) linear (d) parabola
2
2. How many zeroes are there for the
1
polynomial (shape of the wire)?
X' X
(a) 2 (b) 3 –6 –5 –4 –3 –2 –1 1 2 3 4 5 6 7
–1
(c) 1 (d) 0
3. The zeroes of the polynomial are –2
(a) –1, 5 (b) –1, 3 –3
(c) 3, 5 (d) –4, 2 –4
4. What will be the expression of the –5
polynomial?
Y'
(a) x2 + 2x – 3 (b) x2 – 2x + 3
(c) x2 – 2x – 3 (d) x2 + 2x + 3

26  Mathematics–10
D:\EG_Mathematics_Standard-10_(Term-1)_(30-07-2021)--\Open_Files\Ch-2\Ch-2
\ 31-Jul-2021   Praveen Kumar  Proof-3 Reader’s Sign _______________________ Date __________

5. What is the value of the polynomial if x = –1?


(a) 6 (b) –18 (c) 18 (d) 0
III.
An asana is a body posture, originally and still a general term for a sitting meditation pose, and later
extended in hatha yoga and modern yoga as exercise, to any type of pose or position, adding reclining,
standing, inverted, twisting, and balancing poses. In the figure, one can observe the poses that can be
related to represent of a quadratic polynomial.

TRIKONASANA

ADHOMUKHA SAVASANA
1.
The shape of the poses shown is
(a) Spiral (b) Ellipse (c) Linear (d) Parabola
2.
The graph of parabola opens downwards, if _______
(a) a ≥ 0 (b) a = 0 (c) a < 0 (d) a > 0
3.
In the graph, how many zeroes are there for the polynomial?

(a) 0 (b) 1 (c) 2 (d) 2


4.
The two zeroes in the above shown graph are
(a) 2, 4 (b) –2, 4 (c) –8, 4 (d) 2, –8
The zeroes of the quadratic polynomial 4 3 x 2 + 5 x − 2 3 are
5.
2 3 2 3 2 3 2 3
(a) , (b) − , (c) , (d) − ,−
3 4 3 4 3 4 3 4

Polynomials  27
D:\EG_Mathematics_Standard-10_(Term-1)_(30-07-2021)--\Open_Files\Ch-2\Ch-2
\ 31-Jul-2021   Praveen Kumar  Proof-3 Reader’s Sign _______________________ Date __________

IV.
Basketball and soccer are played with a spherical ball. Even though an athlete dribbles the ball in
both sports, a basketball player uses his hands and a soccer player uses his feet. Usually, soccer is
played outdoors on a large field and basketball is played indoor on a court made out of wood. The
projectile (path traced) of soccer ball and basketball are in the form of parabola representing quadratic
polynomial.

1.
The shape of the path traced shown is
(a) Spiral (b) Ellipse (c) Linear (d) Parabola
2.
The graph of parabola opens upwards, if _______
(a) a = 0 (b) a < 0 (c) a > 0 (d) a ≥ 0
3.
Observe the following graph and answer

In the above graph, how many zeroes are there for the polynomial?
(a) 0 (b) 1 (c) 2 (d) 3
4.
The three zeroes in the above shown graph are
(a) 2, 3, –1 (b) –2, 3, 1 (c) –3, –1, 2 (d) –2, –3, –1
5.
What will be the expression of the polynomial?
(a) x3 + 2x2 – 5x – 6 (b) x3 + 2x2 – 5x + 6 (c) x3 + 2x2 + 5x – 6 (d) x3 + 2x2 + 5x + 6

28  Mathematics–10
D:\EG_Mathematics_Standard-10_(Term-1)_(30-07-2021)--\Open_Files\Ch-2\Ch-2
\ 31-Jul-2021   Praveen Kumar  Proof-3 Reader’s Sign _______________________ Date __________

Answers and Hints


A. Multiple Choice Questions (MCQs) −2 −4
fi y= , y=
5 5
1. (d) 2. (b) –m, m + 3 are the two zeroes of the polynomial.
3. (a) 3 15. (b) Given p(x) = 3x2 – px + 2
4. (c) 3 and q(x) = 4x2 – qx – 10 have their zero as 2.
5. (b) 5 6. (b) 1 \ p(2) = 0
7. (a) Given p(x) = 2x2 + 5x – 3 fi 12 – 2p + 2 = 0
p(–3) = 2(–3)2 + 5(–3) – 3
fi –2p + 14 = 0
= 18 – 15 – 3
= 18 – 18 = 0 fi p = 7
Since p(–3) = 0 Also q(2) = 0
\  x = –3 is a solution of 2x2 + 5x – 3 = 0 fi 16 – 2q – 10 = 0
8. (b)   1 is a zero of p(x) fi – 2q + 6 = 0
So, p(1) = 0 fi q = 3
fi a – 3a + 3 – 1 = 0  fi  a = 1
Now, 2p – 3q = 2(7) – 3(3) = 14 – 9 = 5
9. (a) Quadratic polynomial
1 0. (c) Let f(x) = x2 – 5x + 4 B. Assertion-Reason Type Questions
Then f(x) = 32 – 5 × 3 + 4 = –2 1. (d) Assertion (A) is false but reason (R) is true.
For f(x) = 0, 2 must be added to 2. (a) Both assertion (A) and reason (R) are true
f(x) and reason (R) is the correct explanation of
11. (b) If – 4 is a zero of p(x) = x2 – x – (2k – 2) assertion (A).
then p(–4) = 0
Case Study Based Questions
fi 16 + 4 – (2k – 2) = 0
I. 1. (b) (4, –2) 2. (a) intersects x-axis
fi 20 – 2k + 2 0
3. (c) parabola 4. (b) x2 – 36
fi –2k = –22  fi  k = 11
5. (c) 0
12. (a) 3
II. 1. (d) parabola 2. (a) 2
5 1
x3 + 3x + 7, 2x2 + 3 – and x + 3. (b) –1, 3 4. (c) x2 – 2x – 3
x x 5. (d) 0
1 3. (b) 1
I II. 1. (d) Parabola 2. (c) a < 0
3x2 + 7x + 2
14. (b)   p(y) = 0
3. (c) 2 4. (b) –2, 4
2 3
\ 5 5 y 2 + 30 y + 8 5 = 0 5. (b) − ,
3 4
fi 5 5 y 2 + 20 y + 10 y + 8 5 = 0
IV. 1. (d) Parabola 2. (c) a > 0
fi 5 y (5 y + 4 5 ) + 2(5 y + 4 5 ) = 0 3. (d) 3 4. (c) –3, –1, 2
fi  ( 5 y + 2)(5 y + 4 5 ) = 0 5. (a) x3 + 2x2 – 5x – 6

2. Relationship between Zeroes and Coefficients of a Polynomial


• If a, b are zeroes of a quadratic polynomial p(x) = ax2 + bx + c, where a ≠ 0, then
−b − (Coefficient of x)
(i) Sum of zeroes = a + b =   ⇒  a + b =
a Coefficient of x 2
c Constant term
(ii) Product of zeroes = ab =   ⇒  ab =
a Coefficient of x 2
• If a, b are zeroes (or roots) of a quadratic polynomial p(x), then p(x) = x2 – (a + b) x + ab
⇒  p(x) = x2 – (Sum of zeroes) x + (Product of zeroes)

Polynomials  29
D:\EG_Mathematics_Standard-10_(Term-1)_(30-07-2021)--\Open_Files\Ch-2\Ch-2
\ 31-Jul-2021   Praveen Kumar  Proof-3 Reader’s Sign _______________________ Date __________

Example 1. A quadratic equation x2 – 2x – 8 is given. The zeroes of it are


(a) –2 and 4 (b) 3 and 5 (c) 1 and 6 (d) None of these
2
Solution. We have p(x) = x – 2x – 8
Now, x2 – 2x – 8 = x2 – 4x + 2x – 8 = x(x – 4) + 2(x – 4) = (x – 4) (x + 2)
So, p(x) = 0
fi (x – 4) (x + 2) = 0 fi x = 4, x = –2 are the zeroes of p(x)
Hence, option (a) is the correct answer.
Example 2. The sum and product of the zeroes of the quadratic equation given in example 1 are respectively
(a) 2, 4 (b) 5, –8 (c) 6, 8 (d) 2, –8
−( −2) −(Coefficient of x)
Solution. sum of its zeroes = 4 + (–2) = 4 – 2 = 2 = =
1 Coefficient of x 2
−8 Constant term
and product of its zeroes = 4 × (–2) = –8 = =
1 Coefficient of x 2
Hence, option (d) is the correct answer.
Example 3. The zeroes of the quadratic equation 4s2 – 4s + 1 are
1 1 1 1 1 1 1 1
(a) , (b) , (c) , (d) ,
2 4 2 2 4 14 3 4
2
Solution. We have p(s) = 4s – 4s + 1
Since 4s2 – 4s + 1 = 4s2 – 2s – 2s + 1 = 2s(2s – 1) – 1(2s – 1)
= (2s – 1) (2s – 1)
So, p(s) = 0
1 1
fi (2s – 1) (2s – 1) = 0 fi s = , are the zeroes of p(s)
2 2
Hence, option (b) is the correct answer.
Example 4. The sum and product of zeroes of the quadratic equation given in example 3 are respectively
3 1 1
(a) 2, (b) 0, (c) 1, (d) None of these
4 8 4
1 1 −( −4) −(Coefficient of s )
Solution. Sum of its zeroes = + = 1 = =
2 2 4 Coefficient of s 2
1 1 1 Constant term
and product of its zeroes = × = =
2 2 4 Coefficient of s 2
Hence, option (c) is the correct answer.
Example 5. The set of the zeroes of the polynomial x2 – 25, their sum and product is
(a) 4, 3; 7; 12 (b) –3, 3; 0; –9 (c) 5, –5; 0; –25 (d) None of these
2 2
Solution. We know that a – b = (a – b) (a + b), so we can write
x2 – 25 = (x – 5) (x + 5)
2
Thus value of p(x) = x – 25 is zero, when either x – 5 = 0 or x + 5 = 0 fi x = 5 or x = –5
– (Coefficient of x)
So, sum of the zeroes = 5 + (–5) = 0 =
Coefficient of x 2
Constant term
and product of the zeroes = (5) (–5) = –25 =
Coefficient of x 2
Hence, option (c) is the correct answer.

30  Mathematics–10
D:\EG_Mathematics_Standard-10_(Term-1)_(30-07-2021)--\Open_Files\Ch-2\Ch-2
\ 31-Jul-2021   Praveen Kumar  Proof-3 Reader’s Sign _______________________ Date __________

Example 6. If the product of the zeroes of the polynomial ax2 – 6x – 6 is 4, then value of a is
1 1 –5 –3
(a) (b) – (c) (d)
8 4 3 2
2
Solution. Given polynomial is ax – 6x – 6
Comparing it with ax2 + bx + c, we get
c −6
product of the zeroes = = = 4 (given)
a a
−6 −6 −3
\   =4 ⇒ a= =
a 4 2
Hence, option (d) is the correct answer.
Example 7. The value of k, if the sum of the zeroes of the polynomial x2 – (k + 6) x + 2 (2k – 1) is half of
their product is
(a) 7 (b) 11 (c) 12 (d) None of these
Solution. Comparing x2 – (k + 6)x + 2(2k – 1) with ax2 + bx + c, we get
−{−(k + 6)}
Sum of the zeroes of the quadratic polynomial = =k+6
1
c 2 (2k – 1)
Product of the zeroes of the quadratic polynomial = a = = 2 (2k – 1)
1
According to the question, we get
1
k + 6 = × 2(2k − 1) ⇒ k + 6 = 2k – 1 fi k = 7
2
Hence, option (a) is the correct answer.
Example 8. If the zeroes of the polynomial x2 + px + q are double in value to the zeroes of 2x2 – 5x – 3,
the values of p and q are respectively
(a) 5, 6 (b) 4, 7 (c) –5, –6 (d) –4, –7
Solution. Let a and b be the zeroes of 2x2 – 5x – 3.
5 −3
\  a + b = ,  ab =
2 2
As per the question,
It is given that 2a, 2b are the zeroes of x2 + px + q
\ 2a + 2b = –p
5
fi 2(a + b) = –p  fi  2 × = –p  fi  p = –5
2
Also, (2a) (2b) = q
fi  4ab = q
q −3 q
fi ab = ⇒ =   fi  q = – 6
4 2 4
Hence, option (c) is the correct answer.
Example 9. A quadratic polynomial whose zeroes are 1 and –3 is
(a) x2 + 3x – 2 (b) x2 + 5x – 5 (c) x2 + 2x – 3 (d) None of these
Solution. Given that zeroes of the polynomial are 1 and –3.
Thus sum of the zeroes = 1 + (–3) = –2
and product of the zeroes = 1 × (–3) = –3

Polynomials  31
D:\EG_Mathematics_Standard-10_(Term-1)_(30-07-2021)--\Open_Files\Ch-2\Ch-2
\ 31-Jul-2021   Praveen Kumar  Proof-3 Reader’s Sign _______________________ Date __________

Required quadratic polynomial is


p(x) = x2 – (Sum of the zeroes) x + Product of the zeroes = x2 + 2x – 3
Hence, option (c) is the correct answer.
Example 10. The quadratic polynomial, sum of whose zeroes is 8 and their product is 12, is given by
(a) x2 – 8x + 12 (b) x2 + 8x – 12 (c) x2 – 5x + 7 (d) x2 + 5x – 7
Solution. Sum of the zeroes is 8 and product of zeroes is 12.
So, the required polynomial p(x) = x2 – (Sum of zeroes) x + Product of zeroes
= x2 – 8x + 12
Hence, option (a) is the correct answer.
Example 11. If a, b are the zeroes of the polynomial 2x2 – 5x + 7, then a polynomial whose zeroes are
2a + 3b, 3a + 2b is
 3   25   9 
(a) k  x 2 − x + 21 (b) k  x 2 − x + 41 (c) k  x 2 + x − 45 (d) None of these
 5   2   2 
2
Solution. Since a, b are the zeroes of 2x – 5x + 7
RS V
SS sum of the zeroes = –b WWW
−(−5) 5 7 SS a WW
\ a + b = = and αβ =  SS
2 2 2 c WW
SSproduct of the zeroes = W
a W
T X
The given zeroes of required polynomial are 2a + 3b and 3a + 2b
5 25
Sum of the zeroes = 2a + 3b + 3a + 2b = 5a + 5b = 5(a + b) = 5 × =
2 2
Again, product of the zeroes = (2a + 3b) (3a + 2b) = 6 (a2 + b2) + 13ab
= 6 [(a + b)2 – 2ab)] + 13 ab = 6(a + b)2 + ab
2
 5 7 75 7 82
= 6   + = + = = 41
 2 2 2 2 2
Now, required polynomial is given by
2 25
k [x2 – (Sum of the zeroes) x + Product of the zeroes] = k [ x − x + 41 ],
2
where k is any non-zero real number.
Hence, option (b) is the correct answer.
13 3
Example 12. A quadratic polynomial whose product and sum of zeroes are − and , respectively.
5 5
(a) k(x2 + 12x + 5) (b) k[x2 – (8x) + (–9))
  1   7    3   13  
(c) k  x 2 −  x +  −   (d) k  x 2 −  x +  −  
  2   
5   5   5 
13
Solution. Product of zeroes of a quadratic polynomial = –
5
3
and sum of zeroes of quadratic polynomial = .
5
  3   − 13  
Thus, required quadratic polynomial = k  x 2 −  x +  , where k is a non-zero real number.
  5   5  
Hence, option (d) is the correct answer.

32  Mathematics–10
D:\EG_Mathematics_Standard-10_(Term-1)_(30-07-2021)--\Open_Files\Ch-2\Ch-2
\ 31-Jul-2021   Praveen Kumar  Proof-3 Reader’s Sign _______________________ Date __________

Exercise 2.2
A. Multiple Choice Questions (MCQs)
Choose the correct answer from the given options:
1. If one of the zeroes of quadratic polynomial (k – 1) x2 + kx + 1 is –3, then k =
4 4 2 3
(a) (b) – (c) (d)
3 3 3 2
2. Sum of the zeroes of the polynomial x2 + 7x + 10 are
(a) 7 (b) – 7 (c) 10 (d) – 10
3. The quadratic polynomial, the sum of whose zeroes is –5 and their product is 6, is
(a) x2 + 5x + 6 (b) x2 – 5x + 6 (c) x2 – 5x – 6 (d) –x2 + 5x + 6
4. Quadratic polynomial having zeroes a and b is
α
(a) x2 – (ab)x + (a + b) (b) x2 – (a + b)x + ab (c) x 2 − x + αβ (d) None of these
β
If one zero of the quadratic polynomial x2 – 5x – 6 is 6, then other zero is
5.
(a) 0 (b) 1 (c) –1 (d) 2
If a, b are the zeroes of the polynomial 2y2 + 7y + 5, the value of a + b + ab is
6.
(a) 0 (b) 1 (c) –1 (d) 2
7.
A quadratic polynomial, the sum and product of whose zeroes and (–3) and 2 respectively is
(a) x2 + 3x + 2 (b) x2 – 3x + 2 (c) x2 + 3x – 2 (d) None of these
If the sum of the zeroes of the quadratic polynomial 3x 2 – kx + 6 is 3, then the value
8.
of k is
(a) 3 (b) 6 (c) 9 (d) 0
A quadratic polynomial, whose sum of zeroes is 2 and product is –8 is
9.
(a) x2 – 3x – 3 (b) x2 + 2x + 8 (c) x2 + 3x + 3 (d) x2 – 2x – 8
The other zero of the polynomial 2x3 + x2 – 6x – 3, if two of its zeroes are − 3 and
10. 3 is
1 1 1 1
(a) (b) – (c) (d) –
2 2 3 3
3 2
11.
If two zeroes of the polynomial p(x) = x – 4x – 3x + 12 are 3 and − 3 , then its third zero is
(a) 2 (b) 5 (c) 5 (d) 4
2
12.
If 2 and –3 are the zeroes of the quadratic polynomial x + (a + 1) x + b; then the values of a and b
respectively are
(a) 0, –6 (b) 0, 0 (c) 6, –6 (d) 2, –3
If a and b are the zeroes of the polynomial ax2 + bx + c; then the value of a2 + b2 is
13.
b 2 − ca b 2 − 2ca b 2 + ca b 2 + 2ca
(a) (b) (c) (d)
a2 a2 a2 a2
A quadratic polynomial whose zeroes are 5 − 3 2 and 5 + 3 2 is
14.
(a) x2 – 10x + 7 (b) x2 + 10x + 7 (c) x2 – 5x + 9 (d) x2 + 5x – 9
2
15.
The zeores of the quadratic polynomial 6x – 3 – 7x are
3 −1 2 3 −3
(a) , (b) , –3 (c) , (d) None of these
2 3 3 5 7
The zeroes of the quadratic polynomial 4x2 – 4x – 3 are
16.
3 −1 3 −1 2 −2
(a) , (b) , (c) , (d) None of these
2 3 2 2 5 5

Polynomials  33
D:\EG_Mathematics_Standard-10_(Term-1)_(30-07-2021)--\Open_Files\Ch-2\Ch-2
\ 31-Jul-2021   Praveen Kumar  Proof-3 Reader’s Sign _______________________ Date __________

1 1
17. If a and b are the zeroes of the polynomial 4x2 + 3x + 7, the value of + is
α β
1 5 3 3
(a) − (b) − (c) − (d)
2 2 7 7
If a, b are the zeroes of the polynomial x2 – 4x + 3, a quadratic polynomial whose zeroes are
18.
3a and 3b is
(a) x2 + 8x + 17 (b) x2 + 12x + 27 (c) x2 – 12x + 27 (d) x2 – 8x + 17
2
19.
If a and b are the zeroes of the polynomial 5x – 7x – 2, the sum of the reciprocals of zeroes is
7 7 5 5
(a) (b) − (c) (d) –
2 2 9 9
2
20.
If a and b are zeroes of a polynomial x + 6x + 9, then a polynomial whose zeroes are –a and –b is
(a) x2 – 6x + 9 (b) x2 + 6x – 9 (c) x2 + 5x + 4 (d) None of these
2
21.
Quadratic polynomial 2x – 3x + 1 has zeroes as a and b. A quadratic polynomial whose zeroes are
3a and 3b is
k k k k
(a) (3x2 + 5x – 5) (b) (3x2 – 5x + 5) (c) (2x2 + 9x + 9) (d) (2x2 – 9x + 9)
2 2 2 2
The value of k such that the polynomial x2 – (k + 6) x + 2(2k – 1) has sum of its zeros equal to half of
22.
their product is
(a) 3 (b) 5 (c) 7 (d) None of these
2
23.
The zeroes of the quadratic polynomial x + 7x + 10 are
(a) –2, –5 (b) 2, 5 (c) –3, –8 (d) 3, 8
24.
A quadratic polynomial whose zeroes are reciprocals of the zeroes of the polynomial :
f(x) = ax2 + bx + c, a π 0, c π 0 is
1 2 1 1 1
(a) (cx + bx + a) (b) (cx2 – bx + a) (c) (cx2 – bx – a) (d) (–cx2 + bx + a)
c c c c
If one zero of the polynomial (a2 + 9)x2 + 13x + 6a is reciprocal of the other, then ‘a’ is
25.
(a) 1 (b) 3 (c) 12 (d) 19
B. Assertion-Reason Type Questions
In the following questions, a statement of assertion (A) is followed by a statement reason (R). Choose
the correct choice as:
(a) Both assertion (A) and reason (R) are true and reason (R) is the correct explanation of assertion (A).
(b) Both assertion (A) and reason (R) are true but reason (R) is not the correct explanation of assertion (A).
(c) Assertion (A) is true but reason (R) is false.
(d) Assertion (A) is false but reason (R) is true.
1. Assertion (A): If one zero of polynomial p(x) = (k2 + 4)x2 + 13x + 4k is reciprocal of each other, then
k = 2.
Reason (R): If (x – a) is a factor of p(x), then p(a) = 0, i.e. a is a zero of p(x).
2. Assertion (A): If both zeroes of the quadratic polynomial x2 –2kx + 2 are equal in magnitude but
1
opposite in sign, then value of k is
.
2 b
Reason (R): Sum of zeroes of a quadratic polynomial ax2 + bx + c is − .

a

34  Mathematics–10
D:\EG_Mathematics_Standard-10_(Term-1)_(30-07-2021)--\Open_Files\Ch-2\Ch-2
\ 31-Jul-2021   Praveen Kumar  Proof-3 Reader’s Sign _______________________ Date __________

Case Study Based Questions


I.
Skipping Rope
Skipping rope is a good exercise. It burns calories, makes bones strong and improves heart health.
During skipping, when rope goes up and down it makes the shape of parabolas (graphs of quadratic
polynomials). Observe the following skipping pictures.

Picture 1 Picture 2 Picture 3


Refer Picture 1
1.
The graph of polynomial p(x) represented by Picture 1 is shown below. The number of zeroes of the
polynomial is

(a) 0 (b) 1 (c) 2 (d) 4


Refer Picture 2
2.
The graph of polynomial p(x) represented by Picture 2 is shown below. Which of the following has
negative (–) sign?

(a) a (b) b (c) c (d) All of these.

Polynomials  35
D:\EG_Mathematics_Standard-10_(Term-1)_(30-07-2021)--\Open_Files\Ch-2\Ch-2
\ 31-Jul-2021   Praveen Kumar  Proof-3 Reader’s Sign _______________________ Date __________

1
3.
If and –1 are the sum and product of zeroes of a polynomial whose graph is represented by
4
Picture (3), the quadratic polynomial is

 1  1   1  1 
(a) k  x 2 − x − 1 (b) k  x 2 − x − 1 (c) k  x 2 + x + 1 (d) k  x 2 + x + 1
 4  4   4  4 
4. Let the Picture (1) represents the quadratic polynomial f (x) = x2 – 8x + k whose sum of the squares
of zeroes is 40. The value of k is
(a) 8 (b) 10 (c) 12 (d) 20
5. Let the Picture (3) represents the quadratic polynomial f (x) = x2 + 7x + 10. Then its zeroes are
(a) –1, –5 (b) –2, –5 (c) 1, 5 (d) 2, 5
II.
Rainbow is an arch of colours that is visible in the sky, caused by the refraction and dispersion of
the sun’s light after rain or other water droplets in the atmosphere. The colours of the rainbow are
generally said to be red, orange, yellow, green, blue, indigo and violet.

E ach colour of rainbow makes a parabola. We know that for any quadratic polynomial
ax2 + bx + c, a ≠ 0, the graph of the corresponding equation y = ax2 + bx + c has one of the two shapes
either open upwards like ∪ or open downwards like ∩ depending on whether a > 0 or a < 0. These
curves are called parabolas.
1. A rainbow is represented by the quadratic polynomial x2 + (a + 1)x + b whose zeroes are 2 and –3.
Then
(a) a = –7, b = –1 (b) a = 5, b = –1 (c) a = 2, b = –6 (d) a = 0, b = –6
The polynomial x2 – 2x – (7p + 3) represents a rainbow. If –4 is zero of it, then the value of p is
2.
(a) 1 (b) 2 (c) 3 (d) 4
The graph of a rainbow y = f (x) is shown below.
3.

36  Mathematics–10
D:\EG_Mathematics_Standard-10_(Term-1)_(30-07-2021)--\Open_Files\Ch-2\Ch-2
\ 31-Jul-2021   Praveen Kumar  Proof-3 Reader’s Sign _______________________ Date __________

The number of zeroes of f (x) is


(a) 0 (b) 1 (c) 2 (d) 3
4. If graph of a rainbow does not intersect the x-axis but intersects y-axis in one point, then number of
zeroes of the polynomial is equal to
(a) 0 (b) 1 (c) 0 or 1 (d) none of these
5. The representation of a rainbow is a quadratic polynomial. The sum and the product of its zeroes are
3 and –2 respectively. The polynomial is
(a) k(x2 – 2x – 3), for some real k. (b) k(x2 – 5x – 9), for some real k.
(c) k(x2 – 3x – 2), for some real k. (d) k(x2 – 8x + 2), for some real k.
III.
The below picture are few natural examples of parabolic shape which is represented by a quadratic
polynomial. A parabolic arch is an arch in the shape of a parabola. In structures, their curve represents
an efficient method of load, and so can be found in bridges and in architecture in a variety of forms.

In the standard form of quadratic polynomial, ax2 + bx + c; a, b and c are


1.
(a) All are real numbers. (b) All are rational numbers.
(c) ‘a’ is a non-zero real number and b and c are any real numbers.
(d) All are integers.
If the roots of the quadratic polynomial are equal, where the discriminant D = b2 – 4ac, then
2.
(a) D > 0 (b) D < 0 (c) D ≥ 0 (d) D = 0
1
3.
If a and are the zeroes of the quadratic polynomial 2x2 – x + 8k, then k is
α
1 −1
(a) 4 (b) (c) (d) 2
2 4 4
4.
The graph of x + 1 = 0
(a) Intersects x-axis at two distinct points (b) Touches x-axis at a point
(c) Neither touches nor intersects x-axis (d) Either touches or intersects x-axis

Polynomials  37
D:\EG_Mathematics_Standard-10_(Term-1)_(30-07-2021)--\Open_Files\Ch-2\Ch-2
\ 31-Jul-2021   Praveen Kumar  Proof-3 Reader’s Sign _______________________ Date __________

1
5.
If the sum of the roots is –p and product of the roots is – , then the quadratic polynomial is
p
 x   2 x   1  2 1
(a) k  − px 2 + + 1 (b) k  px − − 1 (c) k  x 2 + px −  (d) k  x − px + 
 p   p   p  p

Answers and Hints


A. Multiple Choice Questions (MCQs)
11. (d) As 3 and − 3 are the zeroes of the
4
1. (a) 2. (b) – 7 polynomial p(x).
3
3. (a) x2 + 5x + 6 4. (b) x2 – (a + b)x + ab \  ( x + 3 )( x − 3 ) are the factors of p(x).
5. (c) –1 Now, p(x) = x3 – 4x2 – 3x + 12
6. (c) p(y) = 2y2 + 7y + 5 = x2(x – 4) – 3(x – 4)
−7 = (x2 – 3)(x – 4)
\ a + b =
2 = ( x − 3 )( x + 3 )( x − 4)
5 So third zero = 4.
ab =
2 12. (a) Since 2 and –3 are zeroes of p(x) = x2 + (a + 1)
−7 5 x+b
So, a + b + ab = + = −1
2 2 \ 2 + (–3) = –(a + 1) fi a = 0
7. (a) Let a and b be the zeroes of the required and 2(–3) = b fi b = – 6.
polynomial f(x). 2
b – 2ca
Here, a + b = –3 and ab = 2 13. (b) 2
a
\ f(x) = x2 – (a + b)x + ab 14. (a) Sum of zeroes = 5 − 3 2 + 5 + 3 2 = 10 


= x2 – (–3)x + 2
= x2 + 3x + 2
(
Product of zeroes = 5 − 3 2 5 + 3 2 = 7 )( )
2
Therefore   P(x) = x – 10x + 7
k
8. (c) a + b = 15. (a) p(x) = 6x2 – 7x – 3 ...(i)
3 2
= 6x – 9x + 2x – 3
k
3 = = 3x(2x – 3) + 1(2x – 3)
3
= (2x – 3) (3x + 1)
\ k = 9
9. (d) Quadratic polynomial is given by The zeroes of p(x) is given by p(x) = 0.
3 −1
x2 – (a + b)x + ab = x2 – 2x – 8 So the two zeroes are x = ,
2 3
10. (b) Let p(x) = 2x3 + x2 – 6x – 3
3 −1 –3
= x2(2x + 1) – 3(2x + 1) 16. (b) , 17. (c)
2 2 7
= (x2 – 3)(2x + 1) –7
18. (c) x2 – 12x + 27 19. (b)
= ( x − 3 ) ( x + 3 )(2 x + 1) 2
Since − 3 , 3 are two zeroes of p(x). 20. (a) p(x) = x2 + 6x + 9
\  All zeroes of given polynomial are Since a, b are zeroes of p(x)
1
\ a + b = – 6, ab = 9
− 3 , 3 and − .
2

38  Mathematics–10
D:\EG_Mathematics_Standard-10_(Term-1)_(30-07-2021)--\Open_Files\Ch-2\Ch-2
\ 31-Jul-2021   Praveen Kumar  Proof-3 Reader’s Sign _______________________ Date __________

Now, – a – b = – (a + b) ⇒ x = –2, –5


= 6 Thus, the zeroes of x2 + 7x + 10 are a = –2
and (–a) (–b) = ab and b = –5
= 9 24. (a) Let a and b are the zeroes of polynomial,
\  Required quadratic polynomial is given by f(x) = ax2 + bx + c, a π 0, c π 0
q(x) = x2– (–a – b) x + (–a) (–b) −b
Then, a + b =
= x2 – 6x + 9 a
c
21. (d) a and b are zeroes of the polynomial a ⋅ b =
2x2 – 3x + 1. a
1 1 α+β
b 3 c 1 Now, + =
\  a + b = − = and ab = = α β αβ
a 2 a 2 b
Now zeroes of the required polynomial are −
−b a −b
3a, 3b. = a = × =
c a c c
9
So, 3a + 3b = 3(a + b) = a
2 1 1 1 1 a
9 × = = =
α β αβ c c
and (3a) (3b) = 9(ab) =
2 a
\  Required polynomial \  The required polynomial
= k[x2 – (3a + 3b)x + (3a)(3b)]  −b  a b a
= x 2 −   x + = x 2 + x +
 c  c c c
 9 9 k
= k  x 2 − x +  = ( 2 x 2 − 9 x + 9) 2
cx + bx + a 1
 2 2 2 = = (cx 2 + bx + a )
where k is a constant. c c
25. (b) 3
22. (c) Given polynomial : x2 – (k + 6) x + 2 (2k –1)
−b k +6 B. Assertion-Reason Type Questions
Sum of zeroes = ⇒ α+β = = k +6 1. (b) Both assertion (A) and reason (R) are true but
a 1
⇒ a + b = k + 6 reason (R) is not the correct explanation of
c 2 ( 2k − 1) assertion (A).
Product of zeroes = = 2. (d) Assertion (A) is false but reason (R) is true.
a 1
⇒ a b = 4k – 2 Case Study Based Questions
1 I. 1. (c) 2 2. (d) All of these.
ATQ, a + b = (αβ )
2  1 
1 1
3. (a) k  x 2 −
x − 1 4. (c) 12
 4 
⇒  k + 6 = ( 4k − 2) = × 2 ( 2k − 1)
2 2 5. (b) –2, –5
⇒  k + 6 = 2k – 1  ⇒  k = 7 II. 1. (d) a = 0, b = –6 2. (c) 3
23. (a) We have p(x) = x2 + 7x + 10 3. (c) 2 4. (a) 0
p(x) = x2 + 2x + 5x + 10 5. (c) k(x2 – 3x – 2), for some real k.
= x(x + 2) + 5(x + 2) III. 1. (c) ‘a’ is a non zero real number and b and c
= (x + 2) (x + 5) are any real numbers.
The zeroes of polynomial p(x) is given by 1
2. (d) D = 0
3. (b)
p(x) = 0 4
⇒ (x + 2) (x + 5) = 0 4. (c) Neither touches nor intersects x-axis.

Either x + 2 = 0  or  x + 5 = 0  2 1
5. (c) k  x + px − 

⇒ x = –2, x = –5  p

Polynomials  39
D:\EG_Mathematics_Standard-10_(Term-1)_(30-07-2021)--\Open_Files\Ch-2\Ch-2
\ 31-Jul-2021   Praveen Kumar  Proof-3 Reader’s Sign _______________________ Date __________

Experts’ Opinion
Questions based on following types are very important for exams. So, students are advised to revise
them thoroughly.
1. Finding zeroes of the quadratic polynomial and the relation between the zeroes and the coefficients.
2. Forming a quadratic polynomial with given sum of zeroes and product of zeroes.
3. Finding all the zeroes of a polynomial if its some zeroes are given.

IMPORTANT FORMULAE
• If a, b are zeroes of the quadratic polynomial ax2 + bx + c, then
−b c
(i) a + b = (ii) ab =
a a
• If a, b are zeroes of the quadratic polynomial p (x), then p (x) = x2 – (a + b) x + ab

QUICK REVISION NOTES


•• A polynomial p(x) in one variable x is an algebraic expression in x of the form
p(x) = anxn + an–1xn–1 + ... + a2x2 + a1x + a0 where an, an–1, ..., a0 are real numbers with an ≠ 0.
•• The highest power of x in p(x) is called the degree of the polynomial p(x).
•• A quadratic polynomial in x with real coefficients is of the form ax2 + bx + c, where a, b, c are real
numbers with a π 0.
•• Let p(x) be a polynomial in x and k be any real number, then the value obtained by replacing x by k in
p(x) is called the value of p(x) at x = k and is denoted by p(k).
•• A real number k is said to be a zero of a polynomial p(x) if p(k) = 0.
•• A polynomial may have no zero, one or more than one zeroes.
•• A polynomial p(x) of degree n has atmost n zeroes.
•• The zeroes of a polynomial p(x) are precisely the x-coordinates of the points where the graph of
y = p(x) intersects the x-axis.
−b c
•• If a and b are the zeroes of the quadratic polynomial ax2 + bx + c, then a + b = , ab = .
a a

40  Mathematics–10
D:\EG_Mathematics_Standard-10_(Term-1)_(30-07-2021)--\Open_Files\Ch-2\Ch-2
\ 31-Jul-2021   Praveen Kumar  Proof-3 Reader’s Sign _______________________ Date __________

COMMON ERRORS
Errors Corrections

(i) Finding incorrectly p(x) at x = k. (i) To find p(x) at x = k, replace x by k in p(x)


carefully and get the answer.

(ii) Doing mistakes in finding zeroes of a polynomial (ii) Remember that x-coordinate(s) of point(s), where
from graph. the graph of given polynomial intersects the
x-axis are zeroes of the polynomial.

(iii) Splitting the middle term of a polynomial (iii) To split b of the polynomial ax2 + bx + c, write b
incorrectly to find the zeroes. as the sum of two numbers whose product is ac.

(iv) Formating incorrectly a quadratic polynomial (iv) Form a quadratic polynomial with zeroes a, b as
when zeroes are a, b as (x – a) (x – b). k (x – a) (x – b), where k is real.

(v) Finding the incorrect value of y for various values (v) Transpose y term to either side so as to get
of x. positive coefficient.

(vi) Finding the incorrect value of y for various values (vi) Transpose y term to either side so as to get
of x. positive coefficient.

Polynomials  41
D:\EG_Mathematics_Standard-10_(Term-1)_(30-07-2021)--\Open_Files\Ch-3\Ch-3
\ 31-Jul-2021   Praveen Kumar  Proof-3 Reader’s Sign _______________________ Date __________

3 Pair of Linear Equations in Two Variables

Topics Covered
1. Pair of Linear Equations in Two Variables and Finding its Solution by Graphical Method
2. Solving a Pair of Linear Equations by Algebraic Methods
3. Solving a System of Equations Reducible to Simultaneous Linear Equations
4. Solving Word Problems on Pair of Linear Equations in Two Variables

Introduction
An equation which can be expressed in the form ax + by + c = 0, where a, b and c are real numbers, and
a π 0, b π 0 is called a linear equation in two variables x and y. A solution of a linear equation in two
variables is a pair of values of x and y, which satisfy the equation. The graph of a linear equation in two
variables is a straight line.
Every linear equation in two variables has infinitely many solutions and each one of them is represented
by a point on the graph of the equation.
1. Pair of Linear Equations in Two Variables and Finding its Solution by Graphical Method
Pair of Linear Equations in Two Variables
Two linear equations in the same two variables are called a pair of linear equations in two variables.
General form of a pair of linear equations is
a1x + b1y + c1 = 0
a2x + b2y + c2 = 0
where a1, a2, b1, b2, c1, c2 are real numbers such that a12 + b12 ≠ 0, a22 + b22 ≠ 0.
Some examples of a pair of linear equations in two variables are:
(i) x + 2y = 17, 3x – 5y = 0
(ii) 2a + 3b – 2 = 0, a + b – 2 = 0
The solution of a given pair of linear equations in x and y is a pair of values of x and y, which satisfy
each of the equations. A pair of linear equations in two variables is said to be consistent, if it has at least
one solution. A pair of linear equations in two variables is said to be inconsistent, if it has no solution.
Graphical Method of Solution of a Pair of Linear Equations
In order to find a solution of a pair of linear equations in two variables by graphical method, we draw
the graph of each of the given linear equations. When we represent a pair of linear equations graphically
as two lines, we can observe that the lines
(i) may intersect. (ii) may be parallel. (iii) may coincide.
The number of solutions for a system of two linear equations in two variables is given by one of the
following observations.
What this Means Graphically Number of Solutions Type of Equations
The two lines intersect at a single point. Unique common solution. Consistent
The two lines are parallel. No common solution. Inconsistent
The two lines are coincident. Infinitely many solutions. Dependent Consistent

42
D:\EG_Mathematics_Standard-10_(Term-1)_(30-07-2021)--\Open_Files\Ch-3\Ch-3
\ 31-Jul-2021   Praveen Kumar  Proof-3 Reader’s Sign _______________________ Date __________

Conditions for Consistency of a Pair of Linear Equations


Again, consider a pair of two linear equations
a1x + b1y + c1 = 0
a2x + b2y + c2 = 0
where a1, b1, c1 and a2, b2, c2 denote the coefficients of the equations given above in the general form.
a b c
By comparing the values of 1 , 1 and 1 , we can observe that if the lines represented by the above
a2 b2 c2
stated equations are intersecting or parallel or coincident.
For the system of two linear equations
a1x + b1y + c1 = 0; a2x + b2 y + c2 = 0
S.No. Compare the ratios Graphical representations Algebraic interpretation
a1 b1 • Exactly one solution (Unique)
1. ≠ Intersecting lines • Consistent system of equations
a2 b2
• Equations are independent
a1 b1 c • Infinitely many solutions
2. = = 1 Coincident lines • System is consistent
a2 b2 c2
• Equations are dependent
a1 b1 c1 • No solution
3. = ≠ Parallel lines
a2 b2 c2 • System is inconsistent

Example 1. Given: 10 students of Class X took part in a Mathematics quiz. If the number of girls is 4
more than the number of boys, find the number of boys and girls who took part in the quiz.
The pair of linear equations representing the above situation and their solution, respectively are
(a) x + y = 10, x – y = –4; (3, 7) (b) x – y = 10, x + y = 4; (3, 5)
(c) 2x + y = 10, 2x – y = 4; (2, 5) (d) 2x – y = 10, 2x + y = 4
Solution. Let the number of boys and girls who took
Y
part in Mathematics quiz be x and y respectively. l

Then, x + y = 10 ...(i) 10 (0, 10) m


and y = x + 4 ...(ii)
4
x+
y=

Now, we draw the graph for equations (i) 8


(3, 7)
and (ii).
6
x+

We find two solutions of each of the equations


y=

which are given in table:


10

4
(0, 4)
Table for x + y = 10
2
x 0 3
y = 10 – x 10 7 X' X
O 2 4 6 8 10

Table for y = x + 4
x 0 3
Y'
y=x+4 4 7

Pair of Linear Equations in Two Variables  43


D:\EG_Mathematics_Standard-10_(Term-1)_(30-07-2021)--\Open_Files\Ch-3\Ch-3
\ 31-Jul-2021   Praveen Kumar  Proof-3 Reader’s Sign _______________________ Date __________

From graph, we observe that the two lines representing two equations intersect each other at point
(3, 7). So, the solution of the given pair of linear equations is x = 3 and y = 7.
Thus, 3 boys and 7 girls took part in Mathematics quiz.
Hence, option (a) is the correct ansswer.
Example 2. Given: 5 pencils and 7 pens together cost ` 50, whereas 7 pencils and 5 pens together cost
` 46.
The pair of linear equations representing the above situation and the cost of one pencil and that of one
pen, respectively are
(a) 5x – 7y = 50, 7x – 5y = 46; (3, 5) (b) 5x + 7y = 50, 7x + 5y = 46; (3, 5)
(c) 3x – 8y = 48, 3x + 8y = 48; (5, 3) (d) None of these
Solution. Let cost of one pencil be ` x and cost of one pen be ` y.
Then, 5x + 7y = 50 ...(i)
and 7x + 5y = 46 ...(ii)
Now, we draw the graph of equations (i) and (ii).
We will now complete the tables as follows: Y

Table for 5x + 7y = 50 10

x 10 3
8
50 − 5 x
y= 0 5
7 6
(3, 5)

4
Table for 7x + 5y = 46 5x
+
7x

7y
x 8 3 =
+

50
5y

2
=

46 – 7x
46

y= –2 5 (10, 0)
5 X'
O
X
2 4 6 8 10
(8, –2)
Plotting the values of x and y from these –2

tables on the graph, we observe that two lines Y'

intersect each other at point (3, 5). So, the


solution of the pair of linear equations is x = 3 and y = 5.
Thus, the cost of 1 pencil is ` 3 and that of 1 pen is ` 5.
Hence, option (b) is the correct answer.
Example 3. The area of the quadrilateral formed by the lines x = 3, x = 6, 2x – y – 4 = 0 and x-axis is
(a) 8 sq. units (b) 12 sq. units
(c) 15 sq. units (d) None of these
Solution. First we find three solutions of the equation 2x – y – 4 = 0 (see table) and then plot these
solutions on the graph.

44  Mathematics–10
D:\EG_Mathematics_Standard-10_(Term-1)_(30-07-2021)--\Open_Files\Ch-3\Ch-3
\ 31-Jul-2021   Praveen Kumar  Proof-3 Reader’s Sign _______________________ Date __________

Table for 2x – y – 4 = 0 ⇒ y = 2x – 4 Y

x 0 2 6 8 D (6, 8)
y –4 0 8
7
Drawing the graph of x = 3, x = 6,
we get the lines parallel to y-axis and
6
passing through the points (3, 0) and

0
(6, 0) respectively.

4=
5
Area of the trapezium ABCD

y–

x=3
1

2x
= (sum of parallel sides) × height 4
2

x=6
1
= (2 + 8) × 3 3
2
= 5 × 3 = 15 sq. units 2
A
Hence, option (c) is the correct
answer. 1
Example 4. If the pair of equations (6, 0)
B (3, 0)
x + y = 5 and 2x + 2y = 10 is consistent,
(2, 0) C
X' X
O
the two solutions obtained graphically 1 2 3 4 5 6 7

are –1
(a) (0, 4), (4, 0)
(b) (7, –2), (2, 7) –2
(c) (0, 5), (5, 0)
(d) None of these [Imp.] –3
Solution. The given equations can be
written –4 (0, –4)
x + y – 5 = 0 ...(i)
Y'
and 2x + 2y – 10 = 0 ...(ii)
Here, a1 = 1, b1 = 1, c1 = –5
and a2 = 2, b2 = 2, c2 = –10
a1 1 b1 1 c1 −5 1
Now, = , = , = =
a2 2 b2 2 c2 −10 2
a b c
Clearly, 1 = 1 = 1
a2 b2 c2
So, the linear equations represented by (i) and (ii) are
coincident. The system of equations has infinitely many
solutions. The system is consistent and the equations are
dependent.
Now, we draw the graph of equations. First we find two
solutions of each equation (see tables).
Table for x + y – 5 = 0 Table for 2x + 2y – 10 = 0 fi x + y – 5 = 0
x 0 5 x 0 5
y=5–x 5 0 y=5–x 5 0

Pair of Linear Equations in Two Variables  45


D:\EG_Mathematics_Standard-10_(Term-1)_(30-07-2021)--\Open_Files\Ch-3\Ch-3
\ 31-Jul-2021   Praveen Kumar  Proof-3 Reader’s Sign _______________________ Date __________

The graph of the given system of equations is shown in the figure.


There are infinitely many solutions.
Hence, option (c) is the correct answer.
Example 5. The nature of the system of equations 2x – 2y – 2 = 0 and 4x – 4y – 5 = 0 is
(a) Unique (b) Consistent (c) Inconsistent (d) None of these
Solution. We have
2x – 2y – 2 = 0 ...(i)
and 4x – 4y – 5 = 0 ...(ii)
Here, a1 = 2, b1 = –2, c1 = –2 and a2 = 4, b2 = –4, c2 = –5
a1 2 1 b1 −2 1 c1 −2 2
Now, = = , = = , = =
a2 4 2 b2 −4 2 c2 −5 5
a1 b c
Clearly, = 1 ≠ 1
a2 b2 c2
So, the lines are parallel, i.e., they have no point in common. So, the given system of equations has
no solution, i.e., it is inconsistent.
Hence, option (c) is the correct answer.
Example 6. Half the perimeter of a rectangular garden, whose length is 4 m more than its width, is 36 m.
The dimensions of the garden are (use graphical method)
(a) Length = 20 m, width = 16 m (b) Length = 16 m, width = 24 m
(c) Length = 18 m, width = 12 m (d) Length = 24 m, width = 15 m
Solution. Let the length and width of the garden be x m and y m respectively.

Then, x = y + 4 ...(i)
Also x + y = 36 ...(ii)
Now, we find two solutions of each
equation and then draw their graphs.

Table for x = y + 4
x 0 4
y=x–4 –4 0

Table for x + y = 36
x 16 20
y = 36 – x 20 16

From the graph, we observe that the lines intersect each other at the point (20, 16).
Therefore, x = 20 and y = 16 are the required solutions.
Thus, the length of the garden is 20 m and the width of the garden is 16 m.
Hence, option (a) is the correct answer.

46  Mathematics–10
D:\EG_Mathematics_Standard-10_(Term-1)_(30-07-2021)--\Open_Files\Ch-3\Ch-3
\ 31-Jul-2021   Praveen Kumar  Proof-3 Reader’s Sign _______________________ Date __________

Example 7. The solution of the following system of equations when solved graphically is:
2x – 3y – 6 = 0
2x + y + 10 = 0
(a) (3, 4) (b) (–3, –4) (c) (4, 5) (d) (–4, –5)
Solution. We have, 2x – 3y – 6 = 0 ...(i)
2x + y + 10 = 0 ...(ii)
Let us find the table of values for each equation.

2x − 6
Table for 2x – 3y – 6 = 0 ⇒ y = Table for 2x + y + 10 = 0 ⇒ y = –10 – 2x
3

x 3 6 0 x –5 –6 –4

y 0 2 –2 y 0 2 –2

We plot these values to get the following graph:


Y

3
m
l
(–6, 2)
2x

2
+y

(6, 2)
0
+1

6=
y–
0=

1 –3
2x
0

(–5, 0) (3, 0)
X' X
–6 –5 –4 –3 –2 –1 0 1 2 3 4 5 6

–1

–2
(–4, –2) (0, –2)

–3

(–3, –4)
–4

Y'

We observe that the point (–3, –4) is an intersecting point of graphs of the equations (i) and (ii).
Thus, the required solution is x = –3, y = –4.
Hence, option (b) is the correct answer.
Example 8. The coordinates of vertices of a triangle formed by the equations x – y + 1 = 0,
3x + 2y – 12 = 0 and x-axis are
(a) (1, 3), (–2, 3), (3, 0) (b) (3, 4), (–3, 5), (2, 3)
(c) (2, 5), (–3, 0), (5, 0) (d) (2, 3), (–1, 0), (4,0)

Pair of Linear Equations in Two Variables  47


D:\EG_Mathematics_Standard-10_(Term-1)_(30-07-2021)--\Open_Files\Ch-3\Ch-3
\ 31-Jul-2021   Praveen Kumar  Proof-3 Reader’s Sign _______________________ Date __________

Solution. We find the tables of values to draw the graph of the equations,
m
Table for x – y + 1 = 0 Y l
(0, 6)

0
6
x 0 –1

=
1
+
y
y=x+1 1 0


x
4
P (2, 3)

Table for 3x + 2y – 12 = 0 2
(0, 1)
A (–1, 0)
x 2 0 X'
B (4, 0)
X
–2 0

3x
2 4 6 8

+2
12 − 3 x 3 6

y–
–2
y=

12
2

=0
Y'

From the graph we observe that the two lines intersect each other at the point P(2, 3). So,
x = 2 and y = 3 is the solution of the given system of equations.
The triangle formed by these lines and the x-axis is DPAB whose vertices are P(2, 3), A(–1, 0) and
B(4, 0).
Hence, option (d) is the correct answer.
Example 9. The coordinates of the vertices of a triangle formed by the equations of sides are:
y = x; y = 2x; x + y = 6 are
(a) (0, 0), (3, 3), (2, 4) (b) (0, 1), (5, 5), (2, 5)
(c) (4, 4), (3, 0), (1, 6) (d) None of these
Solution. We find the table of values for drawing the graph of equations.
Table of values for y = x Table of values for y = 2x n Y
x 0 2 x 0 1 (0, 6) m
6
y 0 2 y 0 2
(1, 5)
Now, we find the table of values for 5 l
x + y = 6 B
(2, 4)
fi y = 6 – x
x
4
y=

Table for x + y = 6
2x

3 (3, 3)
y=

x
A
0 1
x+
y

(1, 2)
y 6 5
=

2
6

(2, 2)

From the graph, we observe that a triangle is formed, 1


namely OAB and the vertices of triangle OAB are O(0, 0),
A(3, 3) and B (2, 4). X'
(0, 0)
X
Hence, option (a) is the correct answer. 0 1 2 3 4

Example 10. The vertices of the triangle formed by the –1


graphs of the equations 4x – 3y – 6 = 0, x + 3y – 9 and
y-axis are Y'

(a) (1, 3), (2, 4), (–3, 5) (b) (0, 3), (3, 2), (0, –2)
(c) (0, 4), (3, 3), (2, 6) (d) None of these

48  Mathematics–10
D:\EG_Mathematics_Standard-10_(Term-1)_(30-07-2021)--\Open_Files\Ch-3\Ch-3
\ 31-Jul-2021   Praveen Kumar  Proof-3 Reader’s Sign _______________________ Date __________

Solution. We first find the table of solutions of each of the equations, (see table).
Table for 4x – 3y – 6 = 0 Y
m
x 3 0 3
A (0, 3) x
+ 3y –
9=
4x − 6 0
y= 2 –2 P (3, 2)
3 2

0
l

=
Table for x + 3y – 9 = 0

6
1


3y
x 0 3


4x
X' X
9−x –1 0
y= 3 2 1 2 3 4
3 –1
Now, we plot the points of both tables in the graph which B (0, –2)
is shown alongside. –2

We observe that there is a point (3, 2) common to both Y'


the lines l and m, so the solution of linear equations is x =
3 and y = 2, i.e., the given system of equations is consistent. The vertices of the triangle formed by lines
l, m and the y-axis are A(0, 3), P(3, 2) and B(0, –2).
Hence, option (b) is the correct answer.
Example 11. The nature of graphs of equations x + 4y = 3, 2x + 8y = 6 and the number of their solutions are
(a) Consistent, one (b) Consistent, two
(c) Dependent, many (d) Inconsistent, no solution
Solution. We have x + 4y = 3 ...(i)
and 2x + 8y = 6 ...(ii)
To draw the graph for given equations, we find the tables of values which are shown below:
Table for x + 4y = 3 Table for 2x + 8y = 6 fi x + 4y = 3
x 7 –1 –5 x –1 –5 –3
3– x 3– x
y= –1 1 2 y= 1 2 1.5
4 4
We plot the points of both the tables to get the following graph.
Y
(–5, 2)
2

(–3, 1.5)
1 x+
(–1, 1) 4y =
3

X' X
–5 –4 –3 –2 –1 0 1 2 3 4 5 6 7
2x +
8y =
–1 6
(7, –1)
Y'

We observe that graphs of given two equations coincide. So, the given equations are pair of dependent
equations and they have infinitely many solutions.
Hence, option (c) is the correct answer.

Pair of Linear Equations in Two Variables  49


D:\EG_Mathematics_Standard-10_(Term-1)_(30-07-2021)--\Open_Files\Ch-3\Ch-3
\ 31-Jul-2021   Praveen Kumar  Proof-3 Reader’s Sign _______________________ Date __________

Example 12. The value of k so that the following system of equations has no solution is
3x – y – 5 = 0, 6x – 2y + k = 0 [Imp.]
(a) 10 (b) –10
(c) Both 10 and –10 (d) All real values of k except –10
Solution. We have 3x – y – 5 = 0 ...(i)
and 6x – 2y + k = 0 ...(ii)
Here, a1 = 3, a2 = 6, b1 = – 1, b2 = – 2, c1 = – 5, c2 = k
a1 b c
For no solution, = 1 ≠ 1
a2 b2 c2
3 −1 − 5 1 −5
\ = ≠   ⇒  ≠   ⇒  k ≠ – 10
6 −2 k 2 k
\  For all real values of k except –10, the system of equations has no solution.
Hence, option (d) is the correct answer.
Example 13. If the pair of equations x sin q + y cos q = 1 and x + y = 2 has infinitely many solutions,
then the value of q is
(a) 30° (b) 45° (c) 60° (d) 90°
Solution. We have x sin q + y cos q = 1 ...(i)
and x + y = 2 ...(ii)
Here, a1 = sin q, a2 = 1, b1 = cos q, b2 =1, c1 = – 1, c2 = – 2
For infinitely many solutions,
a1 b c −1
= 1 = 1   ⇒  sin q = cos q =   ⇒  q = 45°
a2 b2 c2 − 2
Hence, option (b) is the correct answer.
Example 14. The number of solutions of the following pair of linear equations is
x + 2y – 8 = 0
2x + 4y = 16
(a) No solutions (b) One solution
(c) Two solutions (d) Infinitely many solutions
Solution. We have x + 2y – 8 = 0
and 2x + 4y = 16
Here, a1 = 1, a2 = 2, b1 = 2, b2 = 4, c1 = –8, c2 = –16
a1 1 b1 2 1 c1 −8 1
Now, = , = = , = =
a2 2 b2 4 2 c2 − 16 2
a1 b c 1
Since = 1 = 1 =
a2 b2 c2 2
\ The pair of linear equations is consistent and has infinitely many solutions.
Hence, option (d) is the correct answer.
Example 15. The lines represented by the equations 5x – 4y + 8 = 0, 7x + 6y – 9 will
(a) intersect at a point (b) be parallel (c) be coincident (d) None of these

50  Mathematics–10
D:\EG_Mathematics_Standard-10_(Term-1)_(30-07-2021)--\Open_Files\Ch-3\Ch-3
\ 31-Jul-2021   Praveen Kumar  Proof-3 Reader’s Sign _______________________ Date __________

Solution. We have
5x – 4y + 8 = 0 ...(i)
and 7x + 6y – 9 = 0 ...(ii)
Here, a1 = 5, b1 = –4, c1 = 8 and a2 = 7, b2 = 6, c2 = –9
a 5 b −4 2 a b
Since, 1 = and 1 = = − . So, 1 ≠ 1
a2 7 b2 6 3 a2 b2
Thus, the lines representing the given equations are intersecting. The system of equations is
consistent and has a unique solution. Equations are independent.
Hence, option (a) is the correct answer.
Example 16. The lines represented by the equations 9x + 3y + 12 = 0 and 18x + 6y + 24 will
(a) intersect at a point (b) be parallel (c) be coincident (d) None of these
Solution. We have
9x + 3y + 12 = 0 ...(i)
and 18x + 6y + 24 = 0 ...(ii)
Here, a1 = 9, b1 = 3, c1 = 12 and a2 = 18, b2 = 6, c2 = 24
a1 9 1 b 3 1 c 12 1 a b c
Since, = = , 1 = = and 1 = = So, 1 = 1 = 1
a2 18 2 b2 6 2 c2 24 2 a2 b2 c2
Thus, the lines representing the given equations are coincident. The system of equations is consistent
and the equations are dependent. There are infinitely many solutions.
Hence, option (c) is the correct answer.
Example 17. The lines represented by the equations 6x – 3y + 10 = 0 and 2x – y + 9 = 0 will
(a) intersect at a point (b) be parallel (c) be coincident (d) None of these
Solution. We have
6x – 3y + 10 = 0 ...(i)
and 2x – y + 9 = 0 ...(ii)
Here, a1 = 6, b1 = –3, c1 = 10 and a2 = 2, b2 = –1, c2 = 9
a 6 b −3 c 10 a1 b c
Since, 1 = = 3, 1 = = 3 and 1 = . So, = 1 ≠ 1
a2 2 b2 −1 c2 9 a2 b2 c2
Thus, the lines representing the given equations are parallel. The system of equations is inconsistent
and has no solution because the lines never intersect.
Hence, option (b) is the correct answer.
a1 b1 c1
Example 18. On comparing , , , the graphical representation of equations 3x + 2y = 5, 2x – 3y = 7
a2 b2 c2
will be

(a) Intersecting (b) Coincident (c) Parallel (d) None of these


Solution. We have
3x + 2y – 5 = 0 ...(i)
and 2x – 3y – 7 = 0 ...(ii)
Here, a1 = 3, b1 = 2, c1 = –5 and a2 = 2, b2 = –3, c2 = –7
a 3 b 2 2 a b
Now, 1 = , 1 = = −   fi  1 ≠ 1
a2 2 b2 −3 3 a2 b2

Pair of Linear Equations in Two Variables  51


D:\EG_Mathematics_Standard-10_(Term-1)_(30-07-2021)--\Open_Files\Ch-3\Ch-3
\ 31-Jul-2021   Praveen Kumar  Proof-3 Reader’s Sign _______________________ Date __________

Thus, the lines representing the given equations are intersecting. The system of equations is
consistent and has a unique solution. Equations are independent.
Hence, option (a) is the correct answer.
a b c
Example 19. On comparing 1 , 1 , 1 , the graphical representation of equations 2x – 3y = 8 and
a2 b2 c2
4x – 6y – 9 = 0 will

(a) Intersecting lines (b) Coincident lines (c) Parallel lines (d) None of these
Solution. We have
2x – 3y – 8 = 0 ...(i)
and 4x – 6y – 9 = 0 ...(ii)
Here, a1 = 2, b1 = –3, c1 = –8 and a2 = 4, b2 = –6, c2 = –9
a 2 1 b −3 1 c −8 8 a b c
Now, 1 = = , 1 = = and 1 = =   fi  1 = 1 ≠ 1
a2 4 2 b2 −6 2 c2 −9 9 a 2 b2 c2

Thus, the lines representing the given equations are parallel. The system of equations is inconsistent
and has no solution.
Hence, option (c) is the correct answer.

Exercise 3.1
A. Multiple Choice Questions (MCQs)
Choose the correct answer from the given options:
3x 5 y
1. The pair of linear equations + = 7 and 9x + 10y = 14 is
2 3
(a) consistent (b) inconsistent
(c) consistent with one solution (d) consistent with many solutions
2. The value of k for which the system of linear equations x + 2y = 3, 5x + ky + 7 = 0 is inconsistent is
14 2
(a) − (b) (c) 5 (d) 10
3 5
3.
If a pair of linear equations is consistent, then the line represented by them are
(a) parallel (b) intersecting or coincident
(c) always coincident (d) always intersecting
4.
The value of k for which the system of equations kx + 4y = k – 4, 16x + ky = k have infinite number
of solutions is
(a) k = 2 (b) k = 4 (c) k = 6 (d) k = 8
5.
The value of k for which the system of linear equations 3x + y = 1, (2k –1)x + (k –1)y = 2k + 1 have
no solution is
(a) k = 2 (b) k = 4 (c) k = 6 (d) k = 8
6.
If the equations kx – 2y = 3 and 3x + y = 5 represent two intersecting lines at unique point, then the
value of k is
(a) Only 4 (b) Only 5
(c) Only 6 (d) Any number other than –6
7.
The value of k for which the given system has unique solution
2x + 3y – 5 = 0, kx – 6y – 8 = 0 is
(a) k = 2 (b) k ≠ 4 (c) k = 4 (d) k ≠ 4

52  Mathematics–10
D:\EG_Mathematics_Standard-10_(Term-1)_(30-07-2021)--\Open_Files\Ch-3\Ch-3
\ 31-Jul-2021   Praveen Kumar  Proof-3 Reader’s Sign _______________________ Date __________

8. For which values of k, the pair of equations kx + 3y = k – 3 and 12x + ky = k have no solution?
[Imp.]
(a) k = 2 (b) k = 6 (c) – 6 (d) k = –2
9. For what value of k, the following system of equations have infinite solutions: [Imp.]
2x – 3y = 7, (k + 2)x – (2k + 1)y = 3 (2k – 1)?
(a) k = 2 (b) k = 3 (c) k = 4 (d) k = 8
10.
The value of p if the lines represented by the equations 3x – y – 5 = 0 and 6x – 2y – p = 0 are parallel
is [Imp.]
(a) only 8 (b) only 10
(c) only 15 (d) All values of ‘p’ except 10
11.
The value of m for which the pair of linear equations 2x + 3y – 7 = 0 and (m – 1) x + (m + 1)
y = (3m – 1) has infinitely many solutions is
(a) 5 (b) 8 (c) – 5 (d) 8
12.
For what value of k, the pair of linear equations 3x + y = 3 and 6x + ky = 8 does not have a solution?
(a) 2 (b) –2 (c) 4 (d) –4
13.
For what values of p, the pair of equations 4x + py +8 =0 and 2x +2y +2 = 0 have unique solution?
(a) p = 4 (b) p ≠ 4 (c) p = 7 (d) p ≠ 7
14.
What type of straight lines will be represented by the system of equations 2x + 3y = 5 and
4x + 6y = 7? 
(a) Intersecting (b) Parallel (c) Conincident (d) None of these
15.
The values of a and b for which the following pair of linear equations have infinitely many solutions:
2x + 3y = 7, (a + b)x + (2a – b)y = 21, respectively are
(a) a = 5, b = 1 (b) a = 2, b = 3 (c) a = 4, b = 7 (d) None of these
16.
For what value of p, the following pair of linear equations have infinitely many solutions?
(p – 3)x + 3y = p, px + py = 12
(a) 4 (b) 6 (c) 9 (d) 11
17.
The value of k for which the following pair of linear equations have infinitely many solutions:
2x + 3y = 7, (k – 1)x + (k + 2)y = 3k is
(a) 2 (b) 4 (c) 7 (d) 9
2
18.
The value(s) of k for which the pair of linear equations kx + y = k and x + ky = 1 have infinitely many
solutions is
(a) 1 (b) 2 (c) 3 (d) 4
19.
The value of m and n so that the pair of linear equations (2m – 1)x + 3y = 5; 3x + (n – 1)y = 2 has
infinite number of solutions respectively are
15 13 17 11
(a) , (b) , (c) 11 , 11 (d) None of these
4 8 4 5 8 9
20.
For what values of p and q, the following pair of linear equations have infinitely many solutions?
4x + 5y = 2; (2p + 7q)x + (p + 8q)y = 2q – p + 1
(a) p = 1, q = 3 (b) p = 3, q = 4 (c) p = –2, q = 5 (d) p = –1, q = 2
21.
The vertices of the triangle formed by the lines, 5x – y = 5, x + 2y = 1 and 6x + y = 17 are
(a) (1, 0), (3, –1), (2, 5) (b) (2, 3), (5, 6), (3, –1)
(c) (1, 2), (2, 5), (3, 6) (d) None of these

Pair of Linear Equations in Two Variables  53


D:\EG_Mathematics_Standard-10_(Term-1)_(30-07-2021)--\Open_Files\Ch-3\Ch-3
\ 31-Jul-2021   Praveen Kumar  Proof-3 Reader’s Sign _______________________ Date __________

B. Assertion-Reason Type Questions


In the following questions, a statement of assertion (A) is followed by a statement reason (R). Choose
the correct choice as:
(a) Both assertion (A) and reason (R) are true and reason (R) is the correct explanation of assertion (A).
(b) Both assertion (A) and reason (R) are true but reason (R) is not the correct explanation of
assertion (A).
(c) Assertion (A) is true but reason (R) is false.
(d) Assertion (A) is false but reason (R) is true.
1. Assertion (A): Pair of linear equations x + y = 14, x – y = 4 is consistent.
a b a b
Reason (R): By comparing 1 and 1 if we get 1 ≠ 1 , then given system of equations is
consistent. a 2 b2 a2 b2

2.
Assertion (A): For k = 6, the system of linear equations x + 2y + 3 = 0 and 3x + ky + 6 = 0 is inconsistent.
Reason (R): The system of linear equations a1 x + b1 y + c1 = 0 and a2 x + b2 y + c2 = 0 is inconsistent
a b c
if 1 = 1 = 1 .
a2 b2 c2
Case Study Based Questions
I.
The state governments revise fares from time to time based on various factors such as inflation, fuel
price, demand from various quarters, etc. The government notifies different fares for different types
of vehicles like Auto Rickshaws, Taxis, Radio Cab, etc.

The auto charges in a city comprise of a fixed charge together with the charge for the distance covered.
Study the following situations:
Situation-I: In city A, for a journey of 10 km, the charge paid is ` 75 and for a journey of 15 km, the
charge paid is ` 110.
Situation-II: In city B, for a journey of 8 km, the charge paid is ` 91 and for a journey of 14 km, the
charge paid is ` 145.
Refer Situation I
1. If the fixed charges of auto rickshaw be ` x and the running charges be ` y km/hr, the pair of linear
equations representing the situation is
(a) x + 10y = 110, x + 15y = 75 (b) x + 10y = 75, x + 15y = 110
(c) 10x + y = 110, 15x + y = 75 (d) 10x + y = 75, 15x + y = 110
2. What will a person have to pay for travelling a distance of 25 km?
(a) ` 160 (b) ` 280 (c) ` 180 (d) ` 260
3. A person travels a distance of 50 km. The amount he has to pay is
(a) ` 155 (b) ` 255 (c) ` 355 (d) ` 455

54  Mathematics–10
D:\EG_Mathematics_Standard-10_(Term-1)_(30-07-2021)--\Open_Files\Ch-3\Ch-3
\ 31-Jul-2021   Praveen Kumar  Proof-3 Reader’s Sign _______________________ Date __________

Refer Situation II
4. What will a person have to pay for travelling a distance of 30 km?
(a) ` 185 (b) ` 289 (c) ` 275 (d) ` 305
5. The graphs of lines representing the conditions are
(a) (b)

(c) (d)

Answers and Hints


A. Multiple Choice Questions (MCQs) Now, 2(m + 1) = 3(m – 1)  ⇒  m = 5
1. (b) inconsistent 2. (d) 10 and 3(3m – 1) = 7(m + 1)  ⇒  m = 5
3. (b) intersecting or coincident Hence, for m = 5, the system has infinitely
4. (d) k = 8 5. (a) k = 2 many solutions.
6. (d) any number other than –6 12. (a) 2
7. (d) k ≠ –4 3 1 3
8. (c) –6 = ≠
6 k 8
For no solution,
3 1
a1 b c k 3 −k + 3 =
= 1 ≠ 1 ⇒ = ≠ 6 k
a2 b2 c2 12 k −k k = 2
k 3
Now, =   ⇒  k2 = 36  ⇒  k = –6 13. (b) p ≠ 4
12 k
a b
9. (c) k = 4 1 ≠ 1 is the condition for the given pair
10. (c) All real values of ‘p’ except ‘10’. a2 b2
11. (a) 5 of equations to have a unique solution.
For infinitely many solutions the condition is 4 p
⇒     ≠
a1 b1 c1 2 3 7 2 2
= = ⇒ = =
a b2 c2 m − 1 m + 1 3m − 1 ⇒     p ≠ 4
2

Pair of Linear Equations in Two Variables  55


D:\EG_Mathematics_Standard-10_(Term-1)_(30-07-2021)--\Open_Files\Ch-3\Ch-3
\ 31-Jul-2021   Praveen Kumar  Proof-3 Reader’s Sign _______________________ Date __________

Therefore, for all real values of p except 4, 17. (c) 7


the given pair of equations will have a unique For infinitely many solutions the condition is
solution. a1 b1 c1
= =
14. (b) Parallel a2 b2 c2
a1 2 3 7
2 1 ⇒ = =
Here, = = k −1 k + 2 3k
a2 4 2
Now, 2k + 4 = 3k – 3  ⇒  k = 7
b1 3 1 c1 5 and 9k = 7k + 14  ⇒  k = 7
= = and =
b2 6 2 c2 7 Hence, the value of k is 7.
1 1 5 18. (a) 1
= ≠  For pair of equations kx + y = k2 and x + ky = 1
2 2 7
a1 k b1 1 c1 k 2
a1 b c = , = , =
= 1 ≠ 1 is the condition for which a2 1 b2 k c2 1
a2 b2 c2 a b c
For infinitely many solutions, 1 = 1 = 1
the given system of equations will represent a2 b2 c2
parallel lines. k 1
\ =
So, the given system of linear equations will 1 k
2
represent a pair of parallel lines. ⇒ k = 1  ⇒  k = 1, –1 ...(i)
15. (a) a = 5, b = 1 k k2
and =
For infinitely many solutions, 1 1
⇒ k = 1 ...(ii)
a1 b c From (i) and (ii), k = 1
= 1 = 1
a2 b2 c2 17 11
19. (b) ,
2 3 7 4 5
= = For infinite number of solutions,
a+b 2a − b 21
2m − 1 3 5
2 1 = =
Now = ⇒ a + b = 6 ...(i) 3 n −1 2
a+b 3
2m − 1 5 3 5
3 1 Now = and =
Also = ⇒ 2a − b = 9 ...(ii) 3 2 n −1 2
2a − b 3 ⇒ 4m – 2 = 15
Solving (i) and (ii), we get, a = 5, b = 1. and 6 = 5n – 5
⇒ 4m = 17
16. (b) 6
and 5n = 11
For infinitely may solutions, 17 11
⇒ m = and n =
p−3 3 p 4 5
= = ...(i) 20. (d) p = –1, q = 2
p p 12
For infinitely many solutions,
p−3 3 4
Now = 5 2
p p = =
2 p + 7q p + 8q 2q − p + 1
⇒ p – 3 = 3  ⇒  p = 6
4 5
3 p Now, =
and = 2 p + 7q p + 8q
p 12
5 2
⇒ p2 = 36  ⇒  p = ± 6 and =
p + 8q 2q − p + 1
Q For p = 6, (i) is true. So, for p = 6, system
⇒ 4p + 32q = 10p + 35q
has infinitely many solutions. and 10q – 5p + 5 = 2p + 16q

56  Mathematics–10
D:\EG_Mathematics_Standard-10_(Term-1)_(30-07-2021)--\Open_Files\Ch-3\Ch-3
\ 31-Jul-2021   Praveen Kumar  Proof-3 Reader’s Sign _______________________ Date __________

⇒ 6p + 3q = 0 Case Study Based Questions


and –5p – 2p + 10q – 16q + 5 = 0 I. 1. (a) x + 10y = 110, x + 15y = 75
⇒ 2p + q = 0 ...(i) 2. (c) ` 180
and 7p + 6q = 5 ...(ii) 3. (c) ` 355
On solving the equations (i) and (ii), we get 4. (b) ` 289
p = –1 5. (c)
and q = 2
Hence, for p = –1 and q = 2 the given system has
infinitely many solutions.
21. (a) (1, 0), (3, –1), (2, 5)
B. Assertion-Reason Type Questions
1. (a) Both assertion (A) and reason (R) are true
and reason (R) is the correct explanation of
assertion (A).
2. (c) Assertion (A) is true but reason (R) is false.

2. Solving a Pair of Linear Equations by Algebraic Methods


The solution of a pair of linear equations can be obtained by using any one of the following algebraic methods:
(i) Substitution Method  (ii) Elimination Method  
A. To find the solution of a pair of linear equations using Substitution Method, we follow these steps:
Step 1: From one equation, find the value of one variable (say y) in terms of other variable, i.e. x.
Step 2: Substitute the value of variable obtained in step 1 in the other equation to get an equation
in one variable.
Step 3: Solve the equation obtained in step 2 to get the value of one variable.
Step 4: Substitute the value of variable so obtained in any given equation to find the value of
other  variable.
B. To find the solution of a pair of linear equations using Elimination Method, we follow these
steps:
Step 1: Multiply the given equations to make the coefficient of one of the unknown variables
(either x or y), numerically equal.
Step 2: Add the equations obtained after multiplication, if the numerically equal coefficients are
opposite in signs or else subtract them.
Step 3: Solve the resulting linear equation in one unknown variable to find its value.
Step 4: Substitute this value in any one of the equations and find the value of the other variable.
The values of the two variables constitute the solution of the given pair of linear equations.
Example 1. In figure, ABCD is a rectangle. The values of x and y, respectively are

(a) x = 12, y = 16 (b) x = 16, y = 10 (c) x = 22, y = 8 (d) x = 15, y = 18

Pair of Linear Equations in Two Variables  57


D:\EG_Mathematics_Standard-10_(Term-1)_(30-07-2021)--\Open_Files\Ch-3\Ch-3
\ 31-Jul-2021   Praveen Kumar  Proof-3 Reader’s Sign _______________________ Date __________

Solution. AB = DC and BC = AD
⇒ x + y = 30 ...(i)
and x – y = 14 ...(ii)
We have x + y = 30  ⇒  x = 30 – y. Substituting x = 30 – y in (ii), we get
x – y = 14
⇒ 30 – y – y = 14
⇒ 30 – 14 = 2y
16
⇒ y = =8
2
\ x = 14 + 8 = 22
Thus, x = 22 and y = 8
Hence, option (c) is correct answer.
Example 2. The solution of the system of equations x + y = 5, x – y = 2 using substitution method is:
7 3 3 1 3 1 2 5
(a) x = , y = (b) x = , y = (c) x = , y = (d) x = , y =
2 2 5 2 5 4 5 2
Solution. We have x + y = 5  ⇒  y = 5 – x
Substituting y = 5 – x in x – y = 2, we get
x – (5 – x) = 2
7
⇒ 2x – 5 = 2  ⇒ 2x = 7  ⇒  x =
2
7 3
\ y = 5 – =
2 2
7 3
Thus, x =  and y =
2 2
Hence, option (a) is correct answer.
Example 3. The solution of given system of equations:
x + y = a + b, ax – by = a2 – b2 is
1 1
(a) x = 2a, y = b (b) x = a, y = 2b (c) x = a, y = b (d) x = , y =
a b
Solution. We have
x + y = a + b ...(i)
2 2
and ax – by = a – b ...(ii)
From (i), we get x = a + b – y...(iii)
Substituting this value of x in (ii), we get
a(a + b – y) – by = a2 – b2
a2 + ab – ay – by = a2 – b2  ⇒  – (a + b)y = a2 – b2 – a2 – ab
⇒ – (a + b) y = – b2 – ab  ⇒  – (a + b)y = – b(b + a)
− b( a + b)
⇒ y = =b
− ( a + b)
Substituting this value of y in (iii), we get x = a + b – b = a
Thus, the solution of the given system is x = a, y = b.
Hence, option (c) is correct answer.

58  Mathematics–10
D:\EG_Mathematics_Standard-10_(Term-1)_(30-07-2021)--\Open_Files\Ch-3\Ch-3
\ 31-Jul-2021   Praveen Kumar  Proof-3 Reader’s Sign _______________________ Date __________

11 31
Example 4. When 3x + 2y = and –7x + 5y = are solved by elimination method, we get
3 3
5 111 9 160 −4 5 −7 170
(a) x = ,y= (b) x = ,y= (c) x = ,y= (d) x = , y=
19 37 85 27 71 28 87 87
Solution. We have
11
3x + 2y =
  ⇒ 9x + 6y = 11 ...(i)
3
31
and –7x + 5y =   ⇒ –21x + 15y = 31 ...(ii)
3
Multiplying equation (i) by 7 and equation (ii) by 3, we get
63x + 42y = 77 ...(iii)
– 63x + 45y = 93 ...(iv)
Adding (iii) and (iv), we get
170
87y = 170  ⇒  y =
87
Substituting this value of y in equation (iii), we get
170 2380
63x + 42 × = 77  ⇒  63x + = 77
87 29
2380 2233 − 2380 147
⇒ 63x = 77 − = =−
29 29 29
147 7
⇒ x = − =−
29 × 63 87
7 170
Thus, the required solution is x = − and y = .
87 87
Hence, option (d) is correct answer.
Example 5. Solving 3x – 5y – 4 = 0 and 9x = 2y + 7 by the elimination method, we get the values of x
and y as
9 −5 11 15 17 16
(a) x = ,y= (b) x = ,y= (c) x = ,y= (d) None of these
13 13 24 23 25 9
Solution. We have
3x – 5y – 4 = 0 ...(i)
and 9x – 2y – 7 = 0 ...(ii)
Multiplying (i) by 3, we get
9x – 15y – 12 = 0 ...(iii)
Subtracting (iii) from (ii), we get
5
13y + 5 = 0  ⇒  y = −
13
Substituting this value of y in (i), we get
 5 25
3 x − 5  −  − 4 = 0  ⇒  3 x + −4 =0
 13  13
27 9
⇒ 3x − = 0  ⇒  x =
13 13
9 5
Thus, the solution is x = , y = − .
13 13
Hence, option (a) is correct answer.

Pair of Linear Equations in Two Variables  59


D:\EG_Mathematics_Standard-10_(Term-1)_(30-07-2021)--\Open_Files\Ch-3\Ch-3
\ 31-Jul-2021   Praveen Kumar  Proof-3 Reader’s Sign _______________________ Date __________

Exercise 3.2
A. Multiple Choice Questions (MCQs)
Choose the correct answer from the given options:
2x y 1 x 2y
1. Solution of the simultaneous linear equations: – = – and + = 3 is
y 2 6 2 3
(a) x = 2, y = – 3 (b) x = – 2, y = 3 (c) x = 2, y = 3 (d) x = – 2, y = – 3
2. The value of x satisfying both the equations 4x – 5 = y and 2x – y = 3, when y = –1 is
(a) 1 (b) –1 (c) 2 (d) –2
3. Which of the following is not a solution of the pair of equations 3x – 2y = 4 and 6x – 4y = 8?
(a) x = 2, y = 1 (b) x = 4, y = 4 (c) x = 6, y = 7 (d) x = 5, y = 3
Answer the questions (Q 4 to Q 13) using best suitable algebric method.
4. If 2x + 5y – 1 = 0, 2x + 3y – 3 = 0, then
(a) x = 1, y = – 3 (b) x = 3, y = –1 (c) x = 2, y = 5 (d) x = 5, y = – 3
5. If x + 2y – 3 = 0, 3x – 2y + 7 = 0, then
(a) x = –1, y = 2 (b) x = 1, y = 2 (c) x = 2, y = 3 (d) x = – 2, y = – 3
6. If 2x = 5y + 4, 3x – 2y + 16 = 0, then
(a) x = 2, y = –2 (b) x = 3, y = –3 (c) x = 4, y = 5 (d) x = – 8, y = – 4
7. If 6(ax + by) = 3a + 2b; 6(bx – ay) = 3b – 2a, then
1 1 1 1 1 1 1 1
(a) x = , y = (b) x = – , y = – (c) x = , y = (d) x = – , y = –
2 2 2 2 2 3 2 3
4 6
8. If + 3 y = 8; − 4 y = − 5, then
x x
(a) x = 2, y = 2 (b) x = 1, y = –1 (c) x = 2, y = –2 (d) x = 3, y = – 3
9. If 2x + 3y = 11 and 2x – 4y = –24, then the value of ‘m’ for which y = mx + 3 is
(a) 0 (b) 1 (c) –1 (d) – 2
10.
If 2x + 3y = 11 and x – 2y = –12, then the value of ‘m’ for which y = mx + 3 is
(a) 1 (b) –1 (c) 2 (d) – 2
11.
In the figure, ABCDE is a pentagon with BE Q CD and BC Q DE. BC is perpen-
dicular to CD. AB = 5 cm, AE = 5 cm, BE = 7 cm, BC = x – y and CD = x + y. If
the perimeter of ABCDE is 27 cm, the value of x and y, given x, y ≠ 0, respectively
are
(a) x = 6, y = 1 (b) x = 3, y = 4
(c) x = 2, y = 5 (d) x = 0, y = 2
12.
The value of x and y for the following system of equations, respectively are
21 47
+ = 110
x y

47 21
+ = 162  ⇒  x, y ≠ 0
x y
1 1 1 1 1
(a) x = , y = –1 (b) x = , y = 1 (c) x = , y = 2 (d) x = ,y=
2 3 4 3 4

60  Mathematics–10
D:\EG_Mathematics_Standard-10_(Term-1)_(30-07-2021)--\Open_Files\Ch-3\Ch-3
\ 31-Jul-2021   Praveen Kumar  Proof-3 Reader’s Sign _______________________ Date __________

y 
13.
If 2x + y = 23 and 4x – y = 19, then the value of (5y – 2x) and  − 2 , respectively are
x 

−5 3 5 17
(a) 31, (b) 28, (c) 24, (d) 10,
7 11 8 21
Answer the questions (Q 14 to Q 16) by elimination method.
x 7
If + y = 0.8;
14. = 10, then
2 y
x+
2
(a) x = 2, y = 0.5 (b) x = 0.4, y = 0.6 (c) x = 0.3, y = 3 (d) x = 0.5, y = 0.8
15.
If 7(y + 3) – 2(x + 2) = 14, 4(y – 2) + 3(x – 3) = 2, then
(a) x = 1, y = 4 (b) x = 3, y = 5 (c) x = 5, y = 1 (d) None of these
4 3
If + 5 y = 7; + 4 y = 5, then
16.
x x
1
(a) x = , y = –1 (b) x = 8, y = 3 (c) x = 4, y = 7 (d) x = 5, y = 9
3
Answer the questions (Q 17 and Q 18) by the method of substitution.
17.
If 3 – (x – 5) = y + 2, 2(x + y) = 4 – 3y, then
13 9 7 5 4 9 26 −8
(a) x = ,y= (b) x = , y = (c) x = , y = (d) x = ,y=
4 10 16 8 9 12 3 3
x
18.
If 2x + y = 35 and 3x + 2y = 65, the value of is
y
1 1 1 1
(a) (b) (c) (d)
2 3 4 5

Answers and Hints


A. Multiple Choice Questions (MCQs)

1. (c) x = 2, y = 3 6. (d) x = –8, y = –4


2. (a) 1 1 1
3. (d) x = 5, y = 3 7. (c) x =
,y=
2 3
4. (b) x = 3, y = – 1
6ax + 6by = 3a + 2b...(i)
5. (a) x = –1 and y = 2.
x + 2y = 3  ...(i) 6bx – 6ay = –2a + 3b...(ii)
3x – 2y = –7  ...(ii) Multiplying eq. (i) by ‘a’ and eq. (ii) by ‘b’
On adding the equations (i) and (ii), we get and then adding, we get
4x = –4 6a2x + 6aby = 3a2 + 2ab...(iii)
fi x = –1 6b2x – 6aby = –2ab + 3b2...(iv)
Putting x = –1 in eq. (i), we get

6(a2 + b2)x = 3(a2 + b2)


–1 + 2y = 3
fi 2y = 4  fi  y = 2 1

fi x=
Hence, solution of the system is x = –1 and y = 2. 2

Pair of Linear Equations in Two Variables  61


D:\EG_Mathematics_Standard-10_(Term-1)_(30-07-2021)--\Open_Files\Ch-3\Ch-3
\ 31-Jul-2021   Praveen Kumar  Proof-3 Reader’s Sign _______________________ Date __________

1
Putting x = in eq. (i), we get 5
2 13. (a) 31, –
7
1 1 Given equation are
6a × + 6by = 3a + 2b  fi 6by = 2b   fi  y =
2 3 2x + y = 23 ...(i)
and 4x – y = 19 ...(ii)
1 On adding both equations, we get
Hence, solution of the system is x = and
1 2 6x = 42 ⇒ x = 7
y= Putting the value of x in (i), we get
3 2 × 7 + y = 23 ⇒ 14 + y = 23
8. (a) x = 2,  y = 2 ⇒ y = 23 – 14 ⇒ y = 9
9. (c) –1 We have 5y – 2x = 5 × 9 – 2 × 7
2x + 3y = 11  ...(i) = 45 – 14 = 31
2x – 4y = –24 ...(ii)
y 9 9 − 14
11 − 3 y  and − 2 = −2 =
From (i), x =  ...(iii) x 7 7
 2 
5
= – 
Substituting this value of x in (ii), we get 7
y 
11 − 3 y  Hence the value of (5y – 2x) and  − 2
2  − 4 y = –24 x
 2  5
are 31 and – respectively.
fi 11 – 7y = –24 fi  y = 5 (1) 7
Substituting y = 5 in (iii), we get 14. (b) x = 0.4, y = 0.6
11 − 3(5) x
x =
2 + y = 0.8
2
fi x = –2
16
\ x = –2  and  y = 5 fi x + 2y =
Now, y = mx + 3 10
fi 5 = m(–2) + 3 fi 10x + 20y = 16 ...(i)
Thus, m = –1 Also 7 = 10
10. (b) –1 y
11. (a) x = 6, y = 1 x+
2
x + y = 7 and 2(x – y) + x + y + 5 + 5 = 27 fi 7 = 10x + 5y
∴ x + y = 7 and 3x – y = 17 fi 10x + 5y = 7 ...(ii)
Solving, we get, x = 6 and y = 1 Solving (i) and (ii) by elimination method,
1 15y = 9
12. (b) x = ,y=1
3 9 3
1 = a and 1 = b fi y = = = 0.6
Let 15 5
x y From (ii), 10x + 3 = 7
⇒ 21a + 47 b = 110 and 47a + 21b = 162 4
fi x = = 0.4
Adding and subtracting the two equations, 10
we get \ x = 0.4 and y = 0.6
a + b = 4 and a – b = 2 15. (c) x = 5, y = 1
Solving the above two equations, we get a = 7(y + 3) – 2(x + 2) = 14
3 and b = 1 fi –2x + 7y + 3 = 0 ...(i)
1 and 4(y – 2) + 3(x – 3) = 2
∴ x = and y = 1
3 fi 3x + 4y – 19 = 0 ...(ii)

62  Mathematics–10
D:\EG_Mathematics_Standard-10_(Term-1)_(30-07-2021)--\Open_Files\Ch-3\Ch-3
\ 31-Jul-2021   Praveen Kumar  Proof-3 Reader’s Sign _______________________ Date __________

Solving (i) and (ii) by elemination method, 1


x = 5, y = 1 18. (d)
5 2x + y = 35 ...(i)
1
16. (a) x = ,  y = –1 3x + 2y = 65 ...(ii)
3 Using elimination method,
26 −8 6x + 3y = 105
17. (d) x = ,  y =
3 3 6x + 4y = 130
3 – (x – 5) = y + 2

–  –     –
fi y + 2 – 8 + x = 0 –y = – 25
fi x + y – 6 = 0 ...(i) fi y =  25
2(x + y) = 4 – 3y From (i), 2x + 25 = 35
fi 2x + 2y – 4 + 3y = 0 35 − 25
fi 2x + 5y – 4 = 0 ...(ii) fi x = =5
2
Solving by elimination method, So x = 5  and  y = 25
26 −8 x
x =  and y = Thus the value of = 1
3 3 y 5

3. Solving a System of Equations Reducible to Simultaneous Linear Equations


Now, we shall learn the method to find the solutions of such pairs of equations which are not linear but
can be reduced to linear form by making some suitable substitutions. Consider following examples.
1 1 1 1 13
Example 1. When + = 2 and + = are solved by reducing them to a pair of linear
2x 3y 3x 2 y 6
equations, we get
1 1 1 1 1 1 1 1
(a) x = ,y= (b) x = , y = (c) x = , y = (d) x = , y =
2 7 4 5 5 4 5 7
1 1 1 1 13
Solution. We have + = 2 ...(i) and + = ...(ii)
2x 3y 3 x 2 y 6
1 1
Let = u and = v , then the given equations (i) and (ii) become
x y
u v
+ = 2  ⇒ 3u + 2v = 12 ...(iii)
2 3
u v 13
and + =   ⇒ 2u + 3v = 13 ...(iv)
3 2 6
Solving (iii) and (iv), we get
1 1
x = , y =
2 3
Hence, option (a) is the correct answer.
Example 2. The value of x and y for the following system of equations:
2 3 4 9
+ = 2 and − = − 1, respectively are
x y x y
(a) x = 4 , y = 9 (b) x = 5 , y = 10 (c) x = 8 , y = –9 (d) None of these

Pair of Linear Equations in Two Variables  63


D:\EG_Mathematics_Standard-10_(Term-1)_(30-07-2021)--\Open_Files\Ch-3\Ch-3
\ 31-Jul-2021   Praveen Kumar  Proof-3 Reader’s Sign _______________________ Date __________

2 3 4 9
Solution. We have + = 2 ...(i) and − = − 1 ...(ii)
x y x y
1 1
Let = u and = v, then equations (i) and (ii) become
x y
2u + 3v = 2 ...(iii)
and 4u – 9v = –1 ...(iv)
Multiplying (iii) by 2, we get
4u + 6v = 4 ...(v)
Subtracting (iv) from (v), we get
5 1
15 v = 5  ⇒  v = =
15 3
 1 1
From (iii), we get 2u + 3   = 2  ⇒ 2u = 1 ⇒ u =
 3 2
1 1 1
Now, u =   ⇒  =   ⇒  x=2 ⇒x=4
2 x 2
1 1 1
and v =   ⇒  =   ⇒  y =3⇒y=9
3 y 3
Thus, the solution is x = 4, y = 9.
Hence, option (a) is the correct answer
Example 3. Given the following system of equations:
10 2 15 5
+ = 4 and − =−2
x+ y x− y x+ y x− y
Then which of the following is true?
(a) x = 2 , y = 5 (b) x = 1 , y = 2 (c) x = 3 , y = 2 (d) None of these
10 2 15 5
Solution. We have + = 4 ...(i) and − = − 2 ...(ii)
x+ y x− y x+ y x− y
1 1
Let = u and = v , then equations (i) and (ii) become
x+ y x− y
10u + 2v = 4 ...(iii)
and 15u – 5v = –2 ...(iv)
Solving (iii) and (iv), we get x = 3, y = 2.
Hence, option (c) is the correct option
5 1 6 3
Example 4. If + = 2 and − =1
x −1 y − 2 x −1 y − 2
(a) x = 1 , y = 2 (b) x = 2 , y = 3 (c) x = 4 , y = 5 (d) x = 5 , y = 7
5 1 6 3
Solution. We have + = 2 ...(i) and − = 1 ...(ii)
x −1 y − 2 x − 1 y −2
1 1
Let = u and = v , the equations (i) and (ii) become
x −1 y −2
5u + v = 2 ...(iii)
and 6u – 3v = 1 ...(iv)
Solving (iii) and (iv), we get x = 4, y = 5
Hence, option (c) is the correct answer.

64  Mathematics–10
D:\EG_Mathematics_Standard-10_(Term-1)_(30-07-2021)--\Open_Files\Ch-3\Ch-3
\ 31-Jul-2021   Praveen Kumar  Proof-3 Reader’s Sign _______________________ Date __________

Exercise 3.3
A. Multiple Choice Questions (MCQs)
Choose the correct answer from the given options:

2 3 5 4
1.
If + = 13 and − = –2, then x + y equals
x y x y

1 −1 5 −5
(a) (b) (c) (d)
6 6 6 6

2 2
2.
If + 2 y = 15 and − 4 y = 3, then the values of x and y, respectively are
x x

2 1 1 1
(a) ,2 (b) 3, (c) 4, (d) , 4
11 3 4 4

1 1 2 3
3.
If = u and = v, then + = 13 becomes
x y x y

2 3 2 3
(a) 3u + 2v = 13 (b) 2u + 3v = 13 (c) + = 13 (d) + = 13
u v v u
5 1
4.
Equation + = 2 is reduced in linear equation as 5p + q = 2. Then the values of p and q
x −1 y − 2
respectively are
1 1 1 1
(a) , (b) , (c) x – 1, y – 2 (d) x + 1, y + 2
x −1 y − 2 y −1 x − 2
2 3 5 4
5.
If + = 13 and − = –2, then the values of x and y respectively are
x y x y
1 1 1 1
(a) , (b) , (c) 1 , 1 (d) 2, 3
2 3 3 4 2 5
1 1 3 1 1 −1
6. + = and − = , where 3x + y ≠ 0, 3x – y ≠ 0, then
3x + y 3x − y 4 2(3 x + y ) 2(3 x − y ) 8
(a) 1, 2 (b) 1, 1 (c) 1, 3 (d) 1, 4
3a 2b a 3b
7.
If − + 5 = 0 and + – 2 = 0, then
x y x y
1 1 1 1
(a) x = , y= (b) x = , y= (c) x – a, y = b (d) x = a, y = –b
a b b a

1 1 1 1
8.
If − = –1 and + = 8, (x ≠ 0, y ≠ 0), then
2x y x 2y
1 1 1 1 1 1 1 1
(a) x = ,y= (b) x = ,y= (c) x = , y = (d) x = , y =
4 2 3 5 6 8 6 4

Pair of Linear Equations in Two Variables  65


D:\EG_Mathematics_Standard-10_(Term-1)_(30-07-2021)--\Open_Files\Ch-3\Ch-3
\ 31-Jul-2021   Praveen Kumar  Proof-3 Reader’s Sign _______________________ Date __________

9.
When the following pair of equations is solved by reducing them to a pair of linear equations, we get
1 4 1 3
− = 2 and + = 9
x y x y
1 1 1 1 1
(a) x = , y = 1 (b) x = 1, y = (c) x = , y = 5 (d) x = ,y=
6 5 3 4 3
44 30 55 40
10.
If + = 10 and + = 13 , then
x+ y x− y x+ y x− y
(a) x = 7 , y = 7 (b) x = 2, y = 3 (c) x = 5 , y = 2 (d) x = 8, y = 3
57 6 38 21
11.
If + = 5 and + = 9 , then
x+ y x− y x+ y x− y
(a) x = 5 , y = 3 (b) x = 11, y = 8 (c) x = 9 , y = –9 (d) x = 3, y = 4
7x − 2 y 8x + 7 y
12.
If = 5 and = 15 , then
xy xy
(a) x = 1 , y = 1 (b) x = 2, y = 2 (c) x = 3 , y = 4 (d) x = 2, y = 1

Answers and Hints


A. Multiple Choice Questions (MCQs) 6. (b) 1, 1
5 2 1 1 3
1. (c) 2. (a) , 2 + = ...(i)
6 11 (3 x + y ) (3 x − y ) 4
1 1
3. (b) 2u + 3v = 13 4. (a) , 1 1 −1
x −1 y − 2 − = ...(ii)
1 1 2(3 x + y ) 2(3 x − y ) 8
5. (a) ,
2 3 1 1
1 1 Let = p and  =q
Let = p and = q. Then the given (3 x + y ) (3 x − y )
x y
\ (i) and (ii) can be expressed as
equation become
2p + 3q = 13 ...(i) p + q = 3 ...(iii)
and 5p – 4q = –2 ...(ii) 4
Multiplying equation (i) by 5 and equation p q 1
− = − ...(iv)
(ii) by 2, we get 2 2 8
10p + 15q = 65 ...(iii) 1
Multiplying equation (iii) by and adding
10p –  8q = –4 2
  –  +   – it to (iv), we get

23q = 69   q = 3 p q 3
+ =
2 2 8
From (i), 2p + 9 = 13   p = 2

p q 1
1 1 1 − = −
But p = = 2   x = 2 2 8
x x 2

1 1 1  p p  3 1 2 1
and q =    = 3   y =   +  =  −    fi  p= =
y y 3 2 2 8 8 8 4

66  Mathematics–10
D:\EG_Mathematics_Standard-10_(Term-1)_(30-07-2021)--\Open_Files\Ch-3\Ch-3
\ 31-Jul-2021   Praveen Kumar  Proof-3 Reader’s Sign _______________________ Date __________

From (iii), 1+4 16 – 1


 =
1 3 4x 2
+ q =
4 4 5 15
 =
3 1 2 1 4x 2
fi q = − = = 1
4 4 4 2  x =
1 1 1 6
But p = fi =
3x + y 3x + y 4 1 1
Now, + = 8 [From (ii)]
fi 3x + y = 4 ...(v) x 2y
;a x = E
1 1 1 1 1 1
and q = fi =  + = 8
c m
3x − y 3x − y 2 1 2y 6
6
fi 3x – y = 2 ...(vi) 1 1
 6+ = 8    =2
2y 2y
Adding (v) and (vi) 1
3x + y = 4  y =
4
3x – y = 2 1 1

So, x =  and y =
6 6 4
6x = 6  fi  x = =1
6 1
9. (a) x = , y = 1
Subtracting (vi) from (v) 6
3x + y = 4 Given system is
3x – y = 2 1 4
− = 2 ...(i)
(–)  (+) (–) x y

2 1 3
2y = 2  fi  y = =1 + = 9 ...(ii)
2 x y
x = 1 1 1
Thus, the required solution is:  . Let = a and = b then the system
y = 1 x y
becomes,
7. (c) x = –a, y = b
1 1 a – 4b = 2 ...(iii)
8. (d) x = , y = a + 3b = 9 ...(iv)
6 4
Given equations are O n multiplying equation (iii) by 3 and
equation (iv) by 4 and then adding, we get
1 1
– = –1 ...(i) 3a – 12b = 6
2x y
4a + 12b = 36
1 1
+ = 8 ...(ii) 7a = 42  fi  a = 6
x 2y
Putting a = 6 in equation (iii), we get
1
ultiplying eqn. (i) by
M and then adding 6 – 4b = 2  fi 4b = 4  fi  b = 1
2 Thus a = 6 and b = 1
the result to (ii), we get
1 1
1 1 1 fi = 6 and =1
– = – x y
4x 2 y 2 1
1 1 fi x = and y = 1
+ = 8 6
x 2y 10. (d) x = 8, y = 3

1 1 1 11. (b) x = 11, y = 8


+ = 8 − 12. (a) x = 1, y = 1
4x x 2

Pair of Linear Equations in Two Variables  67


D:\EG_Mathematics_Standard-10_(Term-1)_(30-07-2021)--\Open_Files\Ch-3\Ch-3
\ 31-Jul-2021   Praveen Kumar  Proof-3 Reader’s Sign _______________________ Date __________

4. Solving Word Problems on Pair of Linear Equations in Two Variables


To solve the word problems, we should consider the following points:
(i) Reduce the given situation in terms of x and y as mathematical statements.
(ii) Solve the linear equations thus formed using any algebraic method.
Example 1. Read the following problem.
Ritu can row downstream 20 km in 2 hours and upstream 4 km in 2 hours. Find her speed of rowing
in still water and the speed of the current.
The pair of linear equations representing the above situation is
(a) x + y = 10, x + y = 2 (b) x + y = 10, x – y = 2
(c) x – y = 10, x + y = 2 (d) x – y = 10, x – y = 2
Solution. Let her speed of rowing in still water be x km/h.
and Speed of the current = y km/h.
\ Upstream speed = (x – y) km/h
and Downstream speed = (x + y) km/h
Distance
Since = Time
Speed
20
\ = 2  ⇒  x + y = 10 ...(i)
x+ y
4
and = 2  ⇒  x – y = 2 ...(ii)
x− y
Hence, option (b) is the correct answer.
Example 2. A part of monthly hostel charges in a college is fixed and the remaining depends on the
number of days one has taken food in the mess. When a student ‘A’ takes food for 22 days, he has to
pay ` 1380 as hostel charges; whereas a student ‘B’, who takes food for 28 days, pays ` 1680 as hostel
charges. The fixed charges and the cost of food per day respectively are
(a) `280, `50 (b) `240, `60 (c) `290, `100 (d) `170, `210
Solution. Let the fixed hostel charges be ` x and the cost of food per day be ` y.
According to the question, we get
x + 22y = 1380 ...(i)
and x + 28y = 1680 ...(ii)
Subtracting (i) from (ii), we get
6y = 300  ⇒  y = 300 ÷ 6 = 50
Putting y = 50 in (i), we get
x + 22(50) = 1380  ⇒  x + 1100 = 1380 ⇒ x = 280
\ Fixed hostel charges = ` 280 and cost of the food per day = ` 50.
Hence, option (a) is the correct answer.

68  Mathematics–10
D:\EG_Mathematics_Standard-10_(Term-1)_(30-07-2021)--\Open_Files\Ch-3\Ch-3
\ 31-Jul-2021   Praveen Kumar  Proof-3 Reader’s Sign _______________________ Date __________

Example 3. Atul sold a television set and a mobile phone for ` 10500, thereby making a profit of 10%
on the television set and 25% on the mobile phone. If he had taken a profit of 25% on the television set
and 10% on the mobile phone, he would have got ` 10650. The cost of each item is
(a) TV : `7000, Mobile : `5000 (b) TV : `6000, Mobile : `4000
(c) TV : `4000, Mobile : `5000 (d) TV : `5000, Mobile : `4000
Solution. Let the cost price of a television set be ` x and the cost price of the mobile phone be ` y.
The selling price of the television set when it is sold at a profit of 10%
10  110
= `  x + x = ` x
 100  100
The S.P. of the mobile phone when it is sold at a profit of 25%.

= `  y + 25 y = ` 125 y
 100  100
110 x 125 y
So, + = 10500 ...(i)
100 100
125 x 110 y
Similarly, + = 10650 ...(ii)
100 100
From equations (i) and (ii), we get
110x + 125y = 1050000 ...(iii)
and 125x + 110y = 1065000 ...(iv)
On adding equations (iii) and (iv), we get
235x + 235y = 2115000
fi x + y = 9000
fi x = 9000 – y
Putting this value of x in (i), we get
110 (9000 – y) 125 y
+ = 10500
100 100
fi 990000 – 110 y + 125 y = 1050000
y = 60000
fi 15 
\ y = 4000
and x = 9000 – 4000
= 5000
So, the cost price of the television set is ` 5000 and cost price of the mobile phone is ` 4000.
Hence, option (d) is the correct answer.
2
Example 4. Base of an isosceles triangle is times its congruent sides. Perimeter of the triangle is
3
32 cm. The length of each side of that triangle is
(a) 8 cm, 8 cm, 6 cm (b) 12 cm, 12 cm, 10 cm
(c) 10 cm, 10 cm, 12 cm (d) None of these

Pair of Linear Equations in Two Variables  69


D:\EG_Mathematics_Standard-10_(Term-1)_(30-07-2021)--\Open_Files\Ch-3\Ch-3
\ 31-Jul-2021   Praveen Kumar  Proof-3 Reader’s Sign _______________________ Date __________

Solution. Let each of the congruent sides of an isosceles triangle be x cm long and base be y cm long.
According to the first condition,
2
y = x ⇒ 2x – 3y = 0 ...(i)
3
According to the second condition,
x + x + y = 32  ⇒ 2x + y = 32 ...(ii)
Subtracting (i) from (ii), we get
4y = 32  ⇒  y = 32 ÷ 4 = 8
Putting y = 8 in equation (i), we get
2x – 3 (8) = 0
⇒ 2x = 24  ⇒  x = 24 ÷ 2 = 12
\  Each of the congruent sides is 12 cm long and base 8 cm long.
Hence, option (b) is the correct answer.
Example 5. Places A and B are 80 km apart from each other on a highway. A car starts from A and
another from B at the same time. If they move in same direction they meet in 8 hrs and if they move in
opposite directions they meet in 1 hr 20 minutes.
The speeds of cars started from places A and B respectively are
(a) 40 km/hr, 55 km/hr (b) 20 km/hr, 30 km/hr
(c) 35 km/hr, 25 km/hr (d) 30 km/hr, 28 km/hr
Solution. Let the speed of car starts from A or car A be x km/hr and the speed of car starts from B or car
B be y km/hr
A B
Case I:    80 km
8x
car A
car B
8y
After 8 hours,
 distance 
distance covered by car A = 8x as speed = time 
and distance covered by car B = 8y
So, 8x – 8y = 80  fi  x – y = 10 ...(i)
A 80 km
Case II:  B
4 4
x y
3 3
car A car B

4 4
After 1 hr 20 minutes, i.e. hrs, distance covered by car A = x
3 3
4
and distance covered by car B = y
3
4 4
So, x + y = 80  fi  x + y = 60 ...(ii)
3 3
On solving equations (i) and (ii), we get x = 35  and  y = 25
Thus, speed of car A = 35 km/hr and speed of car B = 25 km/hr.
Hence, option (c) is the correct answer.

70  Mathematics–10
D:\EG_Mathematics_Standard-10_(Term-1)_(30-07-2021)--\Open_Files\Ch-3\Ch-3
\ 31-Jul-2021   Praveen Kumar  Proof-3 Reader’s Sign _______________________ Date __________

Example 6. A boat goes 30 km upstream and 44 km downstream in 10 hours. In 13 hours, it can go 40 km


upstream and 55 km downstream. The speed of the stream and that of the boat in still water, respectively are
(a) 10 km/hr, 12 km/hr (b) 8 km/hr, 3 km/hr
(c) 5 km/hr, 4 km/hr (d) 4 km/hr, 6 km/hr
Solution. Let the speed of the boat in still water be x km/h and the speed of the stream be y km/h.
\  Speed of the boat going upstream = (x – y) km/h
and speed of the boat going downstream = (x + y) km/h
Distance
We know that Time =
Speed
30
\  Time taken in going 30 km upstream = h
x− y
44
Time taken in going 44 km downstream = h
x+ y
30 44
\ + = 10 ...(i) (From given condition)
x− y x+ y
40
Again, time taken in going 40 km upstream = h
x− y
55
and time taken in going 55 km downstream = h.
x+ y
40 55
\ + = 13 ...(ii) (From given condition)
x− y x+ y
1 1
Now, putting = a and = b , we get
x− y x+ y
30a + 44b = 10 ...(iii)
40a + 55b = 13 ...(iv)
Multiplying equation (iii) by 4 and equation (iv) by 3 and subtracting, we get
120a + 176b = 40
120a + 165b = 39
         –   –    –

1
11b = 1  ⇒  b =
11
1
\  From (iii), 30a + 44 × = 10  ⇒ 30a + 4 = 10  ⇒ 30a = 10 – 4 = 6
11
6 1
⇒ a = =
30 5
1 1 1
 a =   ⇒  x – y =   ⇒  x – y = 5 ...(v)
5 5
1 1 1
and b =   ⇒  =   ⇒  x + y = 11 ...(vi)
11 x + y 11
On solving (v) and (vi), we get x = 8, y = 3
Thus, speed of the boat in still water = 8 km/h and speed of the stream = 3 km/h
Hence, option (b) is the correct answer.

Pair of Linear Equations in Two Variables  71


D:\EG_Mathematics_Standard-10_(Term-1)_(30-07-2021)--\Open_Files\Ch-3\Ch-3
\ 31-Jul-2021   Praveen Kumar  Proof-3 Reader’s Sign _______________________ Date __________

Exercise 3.4
A. Multiple Choice Questions (MCQs)
Choose the correct answer from the given options:
1. Aruna has only ` 1 and ` 2 coins with her. If the total number of coins that she has is 50 and the
amount of money with her is ` 75, then the number of ` 1 and ` 2 coins are, respectively
(a) 35 and 15 (b) 35 and 20 (c) 15 and 35 (d) 25 and 25
2. The father’s age is six times his son’s age. Four years hence, the age of the father will be four times
his son’s age. The present ages, (in years) of the son and the father are, respectively
(a) 4 and 24 (b) 5 and 30 (c) 6 and 36 (d) 3 and 24
3. A purse contains 25 paise and 10 paise coins. The total amount in the purse is ` 8.25. If the number
of 25 paise coins is one-third of the number of 10 paise coins in the purse, then the total number of
coins in the purse is
(a) 60 (b) 40 (c) 80 (d) 72
4.
A man has some hens and cows. If the number of heads be 48 and the number of feet equals 140, the
number of hens will be
(a) 18 (b) 26 (c) 32 (d) 40
5.
If 3 chairs and 1 table costs ` 1500 and 6 chairs and 1 table costs ` 2400, the pair of linear equations
to represent this situation is
x
(a) 6x + y = 1500, 3x + y = 2400 (b) + y = 1500, x + y = 2400
3 6
(c) 3x + y = 1500, 6x + y = 2400 (d) None of these
1 1
6.
A fraction becomes when 2 is subtracted from the numerator and it becomes when 1 is subtracted
3 2
from the denominator. The fraction is
2 5 4 7
(a) (b) (c) (d)
5 18 13 15
7.
The difference between two numbers is 26 and the larger number exceeds thrice of the smaller number
by 4. The numbers are
(a) 39, 13 (b) 12, 38 (c) 37, 11 (d) None of these
8.
Meena went to a bank to withdraw ` 2,000. She asked the cashier to give her ` 50 and
` 100 notes only. Meena got 25 notes in all. How many notes of ` 50 and ` 100 she received?
(a) `50 : 10, `100 : 15 (b) `50 : 12, `100 : 10 (c) `50 : 15, `100 : 10 (d) None of these
9.
A motor boat whose speed in still water is 18 km/h, takes 1 hour more to go 24 km upstream than to
return downstream to the same spot. The speed of the stream is
(a) 60 km/hr (b) 6 km/min (c) 6 km/s (d) 6 km/hr
10.
The two consecutive odd positive integers, sum of whose squares is 290 are
(a) 5, 13 (b) 11, 13 (c) 13, 17 (d) None of these
11.
Jamila sold a table and a chair for ` 1050, thereby making a profit of 10% on the table and 25% on the
chair. If she had taken a profit of 25% on the table and 10% on the chair, she would have got ` 1065.
Then the cost price of each is
(a) Table : `500, chair : `400 (b) Table : `400, chair : `500
(c) Table : `700, chair : `600 (d) None of these

72  Mathematics–10
D:\EG_Mathematics_Standard-10_(Term-1)_(30-07-2021)--\Open_Files\Ch-3\Ch-3
\ 31-Jul-2021   Praveen Kumar  Proof-3 Reader’s Sign _______________________ Date __________

12.
A part of monthly Hostel charge is fixed and the remaining depends on the number of days one has
taken food in the mess.When Swati takes food for 20 days, she has to pay ` 3000 as hostel charges
whereas, Mansi who takes food for 25 days pays ` 3500 as hostel charges. The fixed charges and the
cost of food per day, respectively are
(a) `400, `300 (b) `300, `200 (c) `1000, `100 (d) `500, `100
13.
Children were fallen in for drill. If each row contained 4 children less, 10 more rows would have been
made. But if 5 more children were fallen in each row, the number of rows would have reduced by 5.
The number of children in the school is
(a) 150 (b) 160 (c) 170 (d) 120
14.
A two-digit number is obtained by either multiplying the sum of digits by 8 and then subtracting 5 or
by multiplying the difference of digits by 16 and adding 3. The number is
(a) 23 (b) 34 (c) 83 (d) 119
15.
The sum of a two-digit number and the number obtained by interchanging the digits is 132. If the two
digits differ by 2, the number is
(a) 45 (b) 75 (c) 85 (d) 115
16.
Two years ago, a father was five times as old as his son. Two years later, his age will be 8 more than
three times the age of the son. The present age of father and son, respectively are
(a) 40 years, 12 years (b) 30 years, 6 years (c) 32 years, 8 years (d) 42 years, 10 years
17. A boat travels for 7 hours. If it travels 4 hours downstream and 3 hours upstream, then it covers the
distance of 116 km. But if it travels 3 hours downstream and 4 hours upstream, it covers the distance
of 108 km. The speed of the boat is
(a) 16 km/h (b) 22 km/h (c) 18 km/h (d) None of these

Case Study Based Questions


I.
Amit is planning to buy a house and the layout is given below. The design and the measurement has
been made such that areas of two bedrooms and kitchen together is 95 sq.m.
x 2 y

5m Bedroom 1 Bath
room Kitchen

2m

5m Bedroom 2 Living Room

15 m

Based on the above information, answer the following questions:
1.
The pair of linear equations in two variables from above situation is
(a) x + y = 13, 2x + y = 19 (b) 2x + y = 13, x + y = 19
(c) x + 2y = 13, x – y = 19 (d) None of these

Pair of Linear Equations in Two Variables  73


D:\EG_Mathematics_Standard-10_(Term-1)_(30-07-2021)--\Open_Files\Ch-3\Ch-3
\ 31-Jul-2021   Praveen Kumar  Proof-3 Reader’s Sign _______________________ Date __________

2. The length of the outer boundary of the layout is


(a) 35 m (b) 54 m (c) 42 m (d) 60 m
3. The area of each bedroom and kitchen in the layout respectively is
(a) 25 m, 35 m (b) 15 m, 25 m (c) 30 m, 35 m (d) 25 m, 30 m
4. The area of living room in the layout is
(a) 55 sq. m (b) 65 sq. m (c) 75 sq. m (d) 85 sq. m
5. The cost of laying tiles in kitchen at the rate of ` 50 per sq m is
(a) ` 1260 (b) ` 1750 (c) ` 1590 (d) ` 1810
II.
Ankit and his friends went to a shop to purchase some daily use items. He purchased five copies and
one book from the shop which cost him ` 500. His friends purchased the same copies and same books
from other shop. If his friends purchased 10 copies and 3 books for ` 1300, then using variables ‘x’
and ‘y’ for the cost of one copy and one book respectively, answer the following questions:

1.
The algebraic representation of the above situation is given by the equations as
(a) 5x + y = 500, 10x + 3y = 1300 (b) x + 5y = 500, 3x + 10y = 1300
(c) 5x – y = 500, 10x – 3y = 1300 (d) x – 5y = 500, 3x – 10y = 1300
2.
The above situations represent a pair of linear equations. The pair of linear equations show a/an
(a) unique solution (b) infinitely many solutions
(c) no solution (d) None of the above
3.
The above situations are representing a pair of linear equations which can be shown by drawing two
lines in a plane. The following possibilities can happen. The two lines are
(a) intersecting at one point (b) parallel to each other
(c) coincident lines (d) perpendicular to each other
4.
The pair of linear equations shown by above situation are
(a) consistent (b) inconsistent
(c) dependent (d) Both (a) and (c)
5.
Using above situations, the cost of one copy and one book separately is
(a) ` 40, ` 300 (b) ` 60, ` 200
(c) ` 20, ` 400 (d) ` 60, ` 300

74  Mathematics–10
D:\EG_Mathematics_Standard-10_(Term-1)_(30-07-2021)--\Open_Files\Ch-3\Ch-3
\ 31-Jul-2021   Praveen Kumar  Proof-3 Reader’s Sign _______________________ Date __________

Answers and Hints


A. Multiple Choice Questions (MCQs) So, 50x + 100y = 2000 ... (i)
1. (d) 25 and 25 2. (c) 6 and 36 x + y = 25 ... (ii)
3. (a) 60 4. (b) 26 Multiply (ii) by 50, we get
5. (c) 3x + y = 1500, 6x + y = 2400 50x + 100y = 2000
Let the cost of 1 chair = ` x 50x + 50y = 1250
And the cost of 1 table = ` y – – –
3x + y = 1500 —————­———
6x + y = 2400 50y = 750
7 ⇒ y = 15
6. (d)
15 Putting y = 15 in equation (ii), we get
Let the numerator be x and denominator be y x + 15 = 25
x ⇒ x = 10
Then, fraction = Meena has received 10 pieces of ` 50 notes
y
and 15 pieces of ` 100 notes.
x−2 1
ATQ, = 9. (d) 6 km/hr
y 3
Given: Speed of boat = 18 km/hr,
⇒ 3x – 6 = y  ⇒ 3x – y = 6 ...(i)
Distance = 24 km
x 1
and =   ⇒ 2x = y – 1 Let x be the speed of stream.
y −1 2
Let t1 and t2 be the time for upstream and
⇒ 2x – y = –1 ...(ii) downstream.
Subtracting (ii) from (i), we have As we know that,
3x – y = 6 distance
2x – y = – 1 ⇒ time =
speed
    – +  +

For upstream, Speed = (18 – x) km/hr,


x = + 7

Time = t1
From equation (i) 24
3 × 7 – y = 6 ⇒ y = 21 – 6 = 15 Therefore, t1 =
18 − x
7
Thus required fraction = . For downstream, Speed = (18 + x) km/hr,
15 Time = t2
7. (c) 37, 11
Let the larger number be x and the smaller 24
Therefore, t2 =
number be y. 18 + x
Then, x – y = 26 ...(i) Now according to the question:
and x – 3y = 4 ...(ii) t1 = t2 + 1
     – +   – 24 24
= +1

2y = 22  ⇒  y = 11 18 − x 18 + x
From (i), x = 26 + 11 = 37
⇒ 48x = (18 – x)(18 + x)
So, the larger number is 37 and the smaller
number is 11. ⇒ 48x = 324 + 18x – 18x – x2
8. (a) ` 50 : 10, ` 100 : 15 ⇒ x2 + 48x – 324 = 0
Let Meena has received x no. of ` 50 notes ⇒ x2 + 54x – 6x – 324 = 0
and y no. of ` 100 notes. ⇒ x(x + 54) – 6(x + 54) = 0

Pair of Linear Equations in Two Variables  75


D:\EG_Mathematics_Standard-10_(Term-1)_(30-07-2021)--\Open_Files\Ch-3\Ch-3
\ 31-Jul-2021   Praveen Kumar  Proof-3 Reader’s Sign _______________________ Date __________

⇒ (x + 54)(x – 6) = 0 125 x
⇒ x = –54 or x = 6 the S.P of table =
100
Since speed cannot be negative. 110 y
⇒ x = –54 will be rejected and S.P of chair =
100
\ x = 6 125 x 110 y
Then, + = 1065 ...(ii)
Thus, the speed of stream is 6 km/hr. 100 100
10. (b) 11, 13 Solving (i) and (ii), we get
Let one of the odd positive integer be x then x = 500
the other odd positive integer is x + 2 and y = 400
ATQ, x2 + (x + 2)2 = 290 Thus, CP of table = ` 500 and CP of chair =
⇒ x + x2 + 4x + 4 = 290
2 ` 400
⇒ 2x2 + 4x – 286 = 0 12. (c) `1000, `100
Let fixed hostel charges be ` x
⇒ 2(x2 + 2x – 143) = 0
Charge per day is ` y
⇒ x2 + 2x – 143 = 0 Charge paid by Swati = ` 3000
2
⇒ x + 13x – 11x – 143 = 0
\ Ist condition is x + 20y = ` 3000 ...(i)
⇒ x(x + 13) – 11(x + 13) = 0
Charge paid by Mansi = ` 3500
⇒ (x – 11)(x + 13) = 0
\ 2nd condition is x + 25y = ` 3500 ...(ii)
⇒ (x – 11) = 0, (x + 13) = 0
Therefore, x = 11 or –13 Subtracting (ii) from (i)
According to question, x is a positive odd x + 20y = 3000 ...(i)
integer. x + 25y = 3500 ...(ii)
Hence, we take positive value of x

–  – –
So, x = 11 and (x + 2) = 11 + 2 = 13

– 5y = – 500
Therefore, the odd positive integers are 11 fi y = ` 100
and 13. Put value of y = 100 in equation (i), we get
11. (a) Table : ` 500, Chair : ` 400 fi x + 20(100) = 3000
Let C.P of table be ` x  and C.P of chair be x = 3000 – 2000 = ` 1000
` y. Hence, fixed charges is ` 1000 and charges
Profit on table is 10%. per day is ` 100.
\ S.P of table = x + 10% of x 13. (a) 150 14. (c) 83
15. (b) 75 16. (d) 42 years, 10 years
10 x 110 x 17. (a) 16 km/hr
= x + = 
100 100
Case Study Based Questions
Profit on chair = 25%
\ S.P of chair = y + 25% of y I. 1. (a) x + y = 13, 2x + y = 19
2. (b) 54 m
25 y 125 3. (c) 30 m, 35 m
= y + = y
100 100 4. (c) 75 sq m
As per question
5. (b) ` 1750
110 x 125 y II. 1. (a) 5x + y = 500, 10x + 3y = 1300
+ = 1050
100 100 2. (a) unique solution
...(i)
3. (a) intersecting at one point
gain if profit on table and chair are 25%
A 4. (d) both (a) and (c)
and 10% respectively, then 5. (a) ` 40, ` 300

76  Mathematics–10
D:\EG_Mathematics_Standard-10_(Term-1)_(30-07-2021)--\Open_Files\Ch-3\Ch-3
\ 31-Jul-2021   Praveen Kumar  Proof-3 Reader’s Sign _______________________ Date __________

Experts’ Opinion
Questions based on following types are very important for exams. So, students are advised to revise
them thoroughly.
1. To find the value of unknown for which the given pair of linear equations has unique solution, infinitely
many solutions or no solutions.
2. To find out whether the lines representing the given pairs of linear equations intersect at a point, are
a b c
parallel or coincident by comparing the ratios 1 , 1 and 1 .
a2 b2 c2
3. To check whether the given pairs of linear equations are consistent / inconsistent and obtain the solution
graphically if consistent.
4. To draw the graphs of given equations and determine the coordinates of the vertices of the triangle
formed by the lines and the x-axis.
5. To solve the given pair of linear equations by the substitution method.
6. To solve the given pair of linear equations by the elimination method and substitution method.
7. To solve the given pair of equations by reducing them to a pair of linear equations.
8. To formulate the given problem as a pair of linear equations and hence find its solution.

IMPORTANT FORMULAE
For the system of two linear equations a1x + b1y + c1 = 0; a2x + b2y + c2 = 0
Compare the ratios Graphical representations Algebraic interpretation

a1 b1
≠ Intersecting lines Exactly one solution (Unique)
a2 b2

a1 b1 c1
= = Coincident lines Infinitely many solutions
a2 b2 c2

a1 b1 c1
= ≠ Parallel lines No solution
a2 b2 c2

Pair of Linear Equations in Two Variables  77


D:\EG_Mathematics_Standard-10_(Term-1)_(30-07-2021)--\Open_Files\Ch-3\Ch-3
\ 31-Jul-2021   Praveen Kumar  Proof-3 Reader’s Sign _______________________ Date __________

QUICK REVISION NOTES


•• Two linear equations in the same two variables are called a pair of linear equations in two variables.
The general form of a pair of linear equations is
a1x + b1y + c1 = 0
a2x + b2y + c2 = 0
where a1, a2, b1, b2, c1, c2 are real numbers, such that a12 + b12 ≠ 0, a22 + b22 ≠ 0.
•• A pair of linear equations in two variables can be represented and solved by the
(i) Graphical Method (ii) Algebraic Method
Graphical Method: The graph of a pair of linear equations in two variables is represented by two lines.
(i) If the lines intersect at a point, then the point gives the unique solution of the two equations. The
pair of equations is consistent.
(ii) If the lines are parallel, the pair of lines of equations has no solution. The pair of equations are
inconsistent.
(iii) If the lines coincide, then there are infinitely many solutions. The pair of equations is consistent.
Algebraic Method: The following are the algebraic methods:
(i) Substitution Method  (ii) Elimination Method
•• If a pair of linear equations is given by a1x + b1y + c1 = 0 and a2x + b2y + c2 = 0, then the following
situations can arise:
a b a b c
(i) 1 ≠ 1 fi Equations are consistent. (ii) 1 = 1 ≠ 1 fi Equations are inconsistent.
a2 b2 a2 b2 c2
a1 b1 c1
(iii) = = fi Equations are dependent and are consistent.
a2 b2 c2
•• There are many situations which can be mathematically represented by two equations that are not linear
to start with. But, when we alter them, they are reduced to a pair of linear equations.

COMMON ERRORS
Errors Corrections
(i) Drawing axes as line segments. (i) Do not forget to mark arrow heads for axes (lines).
(ii) Incorrectly transposing the terms. (ii) Be careful about sign of terms while transposing.
a b c a b c
(iii) Writing incorrectly that if 1 = 1 ≠ 1 , there (iii) If 1 = 1 ≠ 1 , then the pair of linear equations
a2 b2 c2 a2 b2 c2
are infinitely many solutions. is inconsistent and so has no solution.
(iv) Making mistake in the problem of upstream and (iv) The speed of the boat downstream
downstream. = (x + y) km / h
For example, let the speed of the boat in still and the speed of the boat upstream
water be x km / h and speed of the stream be = (x – y) km / h
y km / h. Then, the speed of the boat downstream
= (x – y) km / h and the speed of the boat upstream
= (x + y) km / h

78  Mathematics–10
D:\EG_Mathematics_Standard-10_(Term-1)_(30-07-2021)--\Open_Files\Ch-4\Ch-4
\ 31-Jul-2021    Praveen Kumar  Proof-3 Reader’s Sign _______________________ Date __________

4 Coordinate Geometry

Topics Covered
1. Distance Formula 2. Section Formula

1. Distance Formula
• Distance between points A(x1, y1) and B(x2, y2) is given by
AB = (x2 – x1) 2 + (y2 – y1) 2
This formula is called the distance formula.
• Distance of any point Q (a, b) from origin ‘O’ is given by
OQ = a 2 + b 2
• Application of Distance Formula:
(a) To show collinearity of three points say, A, B, C:.
Find AB, BC and CA and then show that the sum of the shorter
lengths is equal to the largest length.
i.e., AB + BC = AC
or BC + CA = AB
or BA + AC = BC
If sum of shorter lengths is not equal to the largest length, then points are not collinear.
(b) To show two points A and B are equidistant from a point P:
Show PA = PB.
(c) To prove given three points A, B, C form:
(i) a right angled triangle: Show that sum of squares of any two sides is equal to square of third
longest side i.e., AB2 = AC2 + CB2 or AC2 = AB2 + BC2 or BC2 = BA2 + AC2.

(ii) a triangle: Show that sum of lengths of any two sides is greater than third side, i.e., AB + BC
> AC or BC + CA > BA or BA + AC > BC.
(iii) an equilateral triangle: Show that all the three sides are equal, i.e., AB = BC = AC.
(iv) an isosceles triangle: Show that any two sides are equal, i.e., AB = AC or BC = BA or
CA = CB.
(d) To prove given four points A, B, C, D form:
(i) a parallelogram: Show both pair of opposite sides are equal, i.e., AD = BC and AB = DC.
(ii) a parallelogram but not rectangle: Show that both pairs of opposite sides are equal but
diagonals are not equal, i.e., AD = BC and AB = DC but AC π DB.
(iii) a rhombus: Show that four sides are equal, i.e., AB = BC = CD = AD.
(iv) a rhombus but not a square: Show that all sides are equal but diagonals are not equal, i.e.,
AB = BC = CD = DA but AC π DB.
(v) a square: Show that four sides are equal and the diagonals are also equal, i.e., AB = BC
= CD = DA and AC = DB.
(vi) a rectangle: Show that both pairs of opposite sides are equal and the diagonals are also equal,
i.e., AB = CD, BC = DA and AC = DB.

79
D:\EG_Mathematics_Standard-10_(Term-1)_(30-07-2021)--\Open_Files\Ch-4\Ch-4
\ 31-Jul-2021    Praveen Kumar  Proof-3 Reader’s Sign _______________________ Date __________

Example 1. The distance between the points (2, 3) and (4, 1) is


(a) 2 (b) 2 2 (c) 3 (d) 3 3
Solution. Here, x1 = 2, y1 = 3 and x2 = 4, y2 = 1
Distance between the points (2, 3) and (4, 1) = (x2 – x1) 2 + (y2 – y1) 2

= (4 – 2) 2 + (1 – 3) 2 = (2) + (–2) =
2 2
4+4 = 2 2
Hence, option (b) is the correct answer.
Example 2. The distance between the points (–5, 7) and (–1, 3) is
(a) 4 3 (b) 4 5 (c) 4 2 (d) 4 7
Solution. Here, x1 = –5, y1 = 7 and x2 = –1, y2 = 3
D
istance between points (–5, 7) and (–1, 3) = (x2 – x1) 2 + (y2 – y1) 2

= (–1 + 5) 2 + (3 – 7) 2 = 16 + 16 = 4 2
H
ence, option (c) is the correct answer.
Example 3. The value of a, so that the point (4, a) lies on the line 3x – 2y = 5 is
7 5
(a) 2 (b) 3 (c) (d)
2 2
Solution. Since (4, a) lies on the line 3x – 2y = 5
Then 3(4) – 2(a) = 5  fi  12 – 2a = 5
7
fi = a
2
H
ence, option (c) is the correct answer.
Example 4. (5, –2), (6, 4) and (7, –2) are the vertices of a/an
(a) Scalene triangle (b) Equilateral triangle (c) Isosceles triangle (d) None of these
Solution. Let A (5, –2), B (6, 4) and C (7, –2) be the vertices of a triangle.
Now AB = (6 – 5) 2 + (4 + 2) 2 = 37 A (5, – 2)

BC = (7 – 6) + (–2 – 4) = 37
2 2

AC = (7 – 5) 2 + (–2 + 2) 2 = 4 = 2
Here AB = BC
\  DABC is an isosceles triangle. C (7, –2) B (6, 4)
H
ence, option (c) is the correct answer.
Example 5. The point on the y-axis which is equidistant from (2, –5) and (–2, 9) is
(a) (0, 3) (b) (0, 2) (c) (0, 5) (d) None of these
Solution. Let P (0, y) be any point on the y-axis.
Since P (0, y) is equidistant from A (2, –5) and B (–2, 9)
So PA = PB  fi PA2 = PB2
fi (2)2 + (y + 5)2 = (2)2 + (y – 9)2  fi  y2 + 10 y + 25 = y2 – 18y + 81
fi 28y = 81 – 25  fi 28y = 56
56
y = =2
28
So, the point is (0, 2)
H
ence, option (b) is the correct answer.

80  Mathematics–10
D:\EG_Mathematics_Standard-10_(Term-1)_(30-07-2021)--\Open_Files\Ch-4\Ch-4
\ 31-Jul-2021    Praveen Kumar  Proof-3 Reader’s Sign _______________________ Date __________

Exercise 4.1
A. Multiple Choice Questions (MCQs)
Choose the correct answer from the given options:
1. The distance of the point P (–3, –4) from the x-axis (in units) is
(a) 3 (b) –3 (c) 4 (d) 5
2. The distance of the point P (3, –4) from the origin is
(a) 7 units (b) 5 units (c) 4 units (d) 3 units
3. The distance between the points (3, –2) and (–3, 2) is
(a) 52 units (b) 4 10 units (c) 2 10 units (d) 40 units
The distance between the points (a cos q + b sin q, 0) and (0, a sin q – b cos q), is
4.
 [CBSE Standard 2020]
(a) a2 + b2 (b) a2 – b2 (c) a 2 + b 2 (d) a 2 − b 2
The point P on x-axis equidistant from the points A(–1, 0) and B(5, 0) is
5. [CBSE Standard 2020]
(a) (2, 0) (b) (0, 2) (c) (3, 0) (d) (2, 2)
The coordinates of the point which is reflection of point (–3, 5) in x-axis are [CBSE Standard 2020]
6.
(a) (3, 5) (b) (3, –5) (c) (–3, –5) (d) (–3, 5)
The distance of a point from the origin is
7.
(a) x2 + y2 (b) x2 – y2 (c) x 2 + y 2 (d) None of these

The distance between the points c – , 2 m and c , 2 m is


8 2
8.
5 5
(a) 0 units (b) 1 unit (c) 2 units (d) 5 units
9. The perpendicular distance of A(5, 12) from the y-axis is
(a) 4 (b) 5 (c) 7 (d) 8
AOBC is a rectangle whose three vertices are A(0, –3), O(0, 0) and B(4, 0). The length of its diagonal is
10.
(a) 3 units (b) 5 units (c) 7 units (d) 9 units
The distance between ( 2 + 1, 2) and (1, 2 − 2 ) is
11.
(a) 2 (b) 3 (c) 11 (d) 17
The value of k for which the point (0, 2) is equidistant from two points (3, k) and (k, 5) is
12.
(a) 1 (b) 2 (c) 5 (d) 9
1 1
If points (a, 0), (0, b) and (1, 1) are collinear, then the value of a + is
13.
b
(a) 1 (b) 2 (c) 5 (d) 9
If the distance between the points (4, k) and (1, 0) is 5, then what can be the possible values of k?
14.
(a) k = 4 (b) k = –4 (c) k = ± 4 (d) None of these
The value(s) of x, if the distance between the points A(0, 0) and B(x, – 4) is 5 units, is
15.
(a) ± 2 (b) ± 3 (c) ± 4 (d) ± 5
If the point A(0, 2) is equidistant from the points B(3, p) and C(p, 5), the value of p is
16.
(a) 5 units (b) 8 units (c) 10 units (d) None of these
Points A(2, –1), B(3, 4), C(–2, 3), D(–3, –2) are the vertices of a:
17.
(a) Rectangle (b) Square (c) Rhombus (d) None of these
The value of k if the triangle formed by A(8, –10), B(7, –3) and C(0, k) is right-angled at B, is
18.
(a) k = 2 (b) k = –2 (c) k = 4 (d) k = –4

Coordinate Geometry  81
D:\EG_Mathematics_Standard-10_(Term-1)_(30-07-2021)--\Open_Files\Ch-4\Ch-4
\ 31-Jul-2021    Praveen Kumar  Proof-3 Reader’s Sign _______________________ Date __________

The value(s) of y, if the distance between the points (2, y) and (– 4, 3) is 10, is
19.
(a) 11 (b) –5 (c) Both (a) and (b) (d) None of these
Points A(3, 1), B(5, 1), C(a, b) and D(4, 3) are vertices of a parallelogram ABCD. The values of a
20.
and b are respectively
(a) a = 6, b = 3 (b) a = 2, b = 1 (c) a = 4, b = 2 (d) None of these
The x-coordinate of a point P is twice its y-coordinate. If P is equidistant from Q(2, –5) and R(–3, 6),
21.
the coordinates of P are
(a) (12, 6) (b) (8, 9) (c) (4, 11) (d) (16, 8)
The points (a, a), (–a, –a) and ( − 3a, 3a ) are the vertices of a/an
22.
(a) Equilateral triangle (b) Isosceles triangle (c) Scalene triangle (d) None of these
If the distance of P(x, y) from A(5, 1) and B(–1, 5) are equal, then 3x equals
23.
y 2
(a) y (b) (c) 2y (d)
2 y
The point on x-axis which is equidistant from the points (2, –2) and (–4, 2) is
24.
(a) (1, 0) (b) (2, 0) (c) (0, 2) (d) (–1, 0)
A relation between x and y such that the point (x, y) is equidistant from the points (7, 1) and (3, 5), is
25.
(a) x + y = 2 (b) x – y = 2 (c) 2x + y = 8 (d) None of these
The centre of a circle is (2a – 1, 7) and it passes through the point (–3, –1). If the diameter of the
26.
circle is 20 units, then the value of a is
(a) a = –2, 3 (b) a = 4, 2 (c) a = 5, –3 (d) a = –4, 2
If A(–2, 5), B(1, –3) and C(a, b) form an isosceles triangle with CB = CA, then 6a – 16b + 19 equals
27.
(a) 0 (b) 2 (c) 5 (d) 9
If the distance of P(x, y) from the points A(3, 6) and B(–3, 4) are equal, then 3x + y =
28.
(a) 4 (b) 5 (c) 8 (d) 12
If two vertices of an equilateral triangle are (3, 0) and (6, 0), the third vertex is
29.
 5 2 2  9 3 3
(a)  ,  (b) (3, 9) (c)  ,  (d) None of these
7 7  2 2 
PA 2
Point A lies on the line segment PQ joining P(6, –6) and Q(–4, –1) in such a way that
30. = . If
point P also lies on the line 3x + k(y + 1) = 0, the value of k is PQ 5
12 9 18
(a) k = (b) k = (c) k = (d) k = 0
5 5 5
B. Assertion-Reason Type Questions
In the following questions, a statement of assertion (A) is followed by a statement of reason (R).
Choose the correct choice as:
(a) Both assertion (A) and reason (R) are true and reason (R) is the correct explanation of assertion (A).
(b) Both assertion (A) and reason (R) are true but reason (R) is not the correct explanation of assertion (A).
(c) Assertion (A) is true but reason (R) is false.
(d) Assertion (A) is false but reason (R) is true.
1. Assertion (A): The point (0, 4) lies on y-axis.
Reason (R): The x-coordinate on the point on y-axis is zero.
2. Assertion (A): The value of y is 6, for which the distance between the points P(2, –3) and Q(10, y)
is 10.
Reason (R): Distance between two given points A (x 1 , y 1 ) and B (x 2 , y 2 ) is given by

AB = ( x2 − x1 )2 + ( y2 − y1 )2

82  Mathematics–10
D:\EG_Mathematics_Standard-10_(Term-1)_(30-07-2021)--\Open_Files\Ch-4\Ch-4
\ 31-Jul-2021    Praveen Kumar  Proof-3 Reader’s Sign _______________________ Date __________

Case Study Based Questions


Class X students of a secondary school in Krishnagar have been allotted a rectangular plot of a land
I.
for gardening activity. Saplings of Gulmohar are planted on the boundary at a distance of 1m from
each other. There is a triangular grassy lawn in the plot as shown in the figure. The students are to
sow seeds of flowering plants on the remaining area of the plot.


Considering A as origin, answer question (i) to (v).
Considering A as the origin, what are the coordinates of A?
1.
(a) (0, 1) (b) (1, 0) (c) (0, 0) (d) (–1, –1)
What are the coordinates of P?
2.
(a) (4, 6) (b) (6, 4) (c) (4, 5) (d) (5, 4)
What are the coordinates of R?
3.
(a) (6, 5) (b) (5, 6) (c) (6, 0) (d) (7, 4)
What are the coordinates of D?
4.
(a) (16, 0) (b) (0, 0) (c) (0, 16) (d) (16, 1)
What are the coordinates of P if D is taken as the origin?
5.
(a) (12, 2) (b) (–12, 6) (c) (12, 3) (d) (6, 10)

Answers and Hints


A. Multiple Choice Questions (MCQs)
16. (c) 10 units
1. (c) 4 2. (b) 5 units
Here, AB = AC [Given]
3. (a) 52 units 4. (c) a 2 + b 2 fi AB = AC 2 2
5. (a) (2, 0) 6. (c) (–3, –5) fi (3 – 0)2 + (p – 2)2 = (p – 0)2 + (5 – 2)2
7. (c) x + y
2 2
8. (c) 2 units
fi p = 1
9. (b) 5 10. (b) 5 units
11. (a) 2 12. (a) 1 So, point B is (3, 1); point C is (1, 5)
13. (a) 1 Now, length of AB = (3 − 0) 2 + (1 − 2) 2
14. (c) k = ±4
= 9 + 1 = 10 units
(4 − 1) 2 + (k − 0) 2 = 5  fi  k = ±4 17. (c) Rhombus 18. (d) k = –4
15. (b) ±3 19. (c) 11, –5
20. (a) a = 6, b = 3
ABCD is a parallelogram.
AB = ] x – 0g2 + ] – 4 – 0g2
⇒ 5 = x 2 + 16
⇒ (5)2 = ^ x 2 + 16 h
2

⇒ 25 = x2 + 16
⇒ x + 16 – 25 = 0    ⇒  x2 – 9 = 0
2

⇒ x2 = 9
⇒ x = ±3 Since, the diagonals of a parallelogram
Hence, x = 3 or –3. bisect each other.

Coordinate Geometry  83
D:\EG_Mathematics_Standard-10_(Term-1)_(30-07-2021)--\Open_Files\Ch-4\Ch-4
\ 31-Jul-2021    Praveen Kumar  Proof-3 Reader’s Sign _______________________ Date __________

25. (b) x – y = 2
∴ c m = c m
3+a 1+b 4+5 3+1
, ,
2 2 2 2 Let P(x, y) be equidistant from the points
3+a 9 A(7, 1) and B(3, 5)
⇒ =   ⇒  3 + a = 9
2 2 Give AP = BP. So, AP2 = BP2
⇒ a = 6 (x – 7)2 + (y – 1)2 = (x – 3)2 + (y – 5)2
1+b 4 x2 – 14x + 49 + y2 – 2y + 1
and = & 1 + b = 4 & b = 3
2 2 = x2 – 6x + 9 + y2 – 10y + 25
Hence, a = 6 and b = 3. x – y = 2
21. (d) (16, 8) 26. (d) a = –4, 2 27. (a) 0
Let the required point be (2y, y). Let Q(2, –5) 28. (b) 5
and R(–3, 6) are given points.
 9 3 3
Now, PQ = PR 29. (c)  , 
2 2 
fi  (2 y − 2) 2 + ( y + 5) 2
Let A(x, y), B(3, 0) and C(6, 0) are vertices
= (2 y + 3) 2 + ( y − 6) 2 of an equilateral triangle.
fi 4y + 4 – 8y + y2 + 10y + 25
2
Now, AB = AC = BC
= 4y2 + 9 + 12y + y2 – 12y + 36 fi AB2 = AC2 = BC2
fi 2y = 16  fi   y = 8 fi (x – 3)2 + y2 = (x – 6)2 + y2 = 32
Hence coordinates of P are (16, 8). fi (x – 3)2 + y2 = (x – 6)2 + y2...(i)
22. (a) Equilateral triangle and (x – 3)2 + y2 = 32...(ii)
Let P(a, a), Q(– a, – a), R( − 3a, 3a ) are 27 9
Solving (i), we get x = =
the vertices of an equilateral triangle. 6 2
9
PQ = (a + a ) 2 + (a + a ) 2 = 2 2a Put x = in eq. (ii), we get
2
QR = 3 3
( − a + 3a ) 2 + ( − a − 3a ) 2 y =
2
=
2 2a  9 3 3
Hence, the third vertex is  , .
RP = (a + 3a ) 2 + (a − 3a ) 2 2 2 
= 2 2a 18
30. (c) k =
Here, PQ = QR = RP. 5
\  P, Q, R are vertices of an equilateral Coordinates of P are (6, –6). Given that
triangle.   P(6, –6) lies on the line. So,
23. (c) 2y 3x + k(y + 1) = 0
PA2 = PB2 fi 3 × 6 + k(–6 + 1) = 0
⇒ (x – 5)2 + (y – 1)2 = (x + 1)2 + (y – 5)2 fi 18 – 5k = 0
⇒ 12x = 8y  ⇒  3x = 2y 18
fi k =
24. (d) (–1, 0) 5
B. Assertion-Reason Type Questions
Let P(x, 0) be a point on x-axis.
1. (a) Both assertion (A) and reason (R) are true
PA = PB
and reason (R) is the correct explanation of
PA2 = PB2 assertion (A).
(x – 2)2 + (0 + 2)2 = (x + 4)2 + (0 – 2)2 2. (d) Assertion (A) is false but reason (R) is true.
x2 + 4 – 4x + 4 = x2 + 16 + 8x + 4 Case Study Based Questions
–4x + 4 = 8x + 16 I. 1. (c) (0, 0) 2. (a) (4, 6)
x = –1 3. (a) (6, 5) 4. (a) (16, 0)
P(–1, 0) 5. (b) (–12, 6)

84  Mathematics–10
D:\EG_Mathematics_Standard-10_(Term-1)_(30-07-2021)--\Open_Files\Ch-4\Ch-4
\ 31-Jul-2021    Praveen Kumar  Proof-3 Reader’s Sign _______________________ Date __________

2. Section Formula
• Internal Division: If a point P(x, y) divides the line segment XY in the ratio m : n internally, then the
mx + nx1 my2 + ny1 
coordinate of point P is given by  2 , where coordinates of points X and Y are
m + n 
,
 m+n
X (x1, y1) and Y (x2, y2). This is known as section formula.
• Mid-point formula: If P (x, y) is the mid-point of AB, A (x1, y1), B (x2, y2), then coordinates of
 x + x2 y1 + y2 
P (x, y) = P  1 .
2 
,
 2
• External Division: If a point P (x, y) divides the line segment AB, A (x1, y1), B (x2, y2) externally in
 mx − nx1 my2 − ny1 
the ratio m : n, then the coordinates of P (x, y) are P  2 .
m − n 
,
 m−n
 x + x2 + x3 y1 + y2 + y3 
• Coordinate of centroid of a triangle ABC, A (x1, y1), B (x2, y2), C (x3, y3) is  1 ,  .
 3 3
• To find the ratio in which the join of two points is divided according to some given condition, then
kx + x ky + y1 
we take ratio as k : 1 as  2 1 , 2 . If k is positive, then it is internal division.
 k +1 k + 1 
• Application of Section Formula: To show the collinearity of three points say A, B, C by using
section formula, we first assume that any one point say B divides AC in the ratio k : 1. Then, we
find coordinates of B using Section Formula and then equate them with x and y-coordinates of B.
If value of k from both equations are same, then given points i.e., A, B and C are collinear otherwise not.
Example 1. The coordinates of the points P and Q are respectively (4, –3) and (–1, 7). The x-coordinate
PR 3
(abscissa) of a point R on the line segment PQ such that = , is
PQ 5
(a) 0 (b) 1 (c) 2 (d) 3
PQ 5
Solution. Since = 3 2
PR 3
PQ – PR RQ P R Q
5–3 2
fi =   fi  = (4, –3) (–1, 7)
PR 3 PR 3
3 (–1) + 2 (4) –3 + 8
fi  R divides PQ in the ratio 3 : 2.    \  x-coordinate (abscissa) of R = = =1
3+ 2 5
H
ence, option (b) is the correct answer.
Example 2. The ratio in which the line 2x + y – 4 = 0 divides the line segment joining the points
A (2, –2) and B (3, 7) is [Imp.]
(a) 3 : 5 (b) 2 : 9 (c) 5 : 7 (d) 4 : 5
Solution. Let P(x, y) be the point on the line 2x + y – 4 = 0 dividing the line segment joining the points
A(2, –2) and B(3, 7) in the ratio k : 1.
JK 3k + 2 7k – 2 NO
\  The coordinate of P are KK , OO
L k+1 k+1 P JK 3k + 2 NO JK 7k – 2 NO
Since, point (x, y) lies on the line 2x + y = 4.     ⇒   2 KK O+K O =4
L k + 1 OP KL k + 1 OP
6k + 4 + 7k – 2 2
⇒  = 4   ⇒   13k + 2 = 4k + 4   ⇒ 9k = 2  ⇒  k =
k+1 9
2
Thus, required ratio is : 1 or 2 : 9.
9
H
ence, option (b) is the correct answer.

Coordinate Geometry  85
D:\EG_Mathematics_Standard-10_(Term-1)_(30-07-2021)--\Open_Files\Ch-4\Ch-4
\ 31-Jul-2021    Praveen Kumar  Proof-3 Reader’s Sign _______________________ Date __________

Example 3. Let P and Q be the points of trisection of the line segment joining the points A (2, – 2) and B (–7, 4)
such that P is nearer to A. The coordinates of P and Q respectively are
(a) (3, 2), (1, 9) (b) (–4, 3), (5, 0) (c) (–3, 7), (2, 7) (d) (–1, 0), (–4, 2)
Solution. As per the question, the diagram is as follows:

A P Q B
(2, –2) (–7, 4)

\  P divides AB in the ratio 1 : 2.


KJ –7 (1) + 2 (2) 4 (1) + (–2) 2 NOO
\ Coordinates of P = KK , O
L 3 3 P
JK –7 + 4 4 – 4 NO
= KK , OO = (–1, 0)
L 3 3 P
Also Q is the mid-point of PB.
JK –1 + (–7) 0 + 4 NO
\ Coordinates of Q = KK , OO
L 2 2 P
JK –8 NO
= K , 2O = (–4, 2) K O
L 2 P
H
ence, option (d) is the correct answer.
Example 4. If (1, 2), (4, y), (x, 6) and (3, 5) are the vertices of a parallelogram taken in order, x and y
respectively are
(a) x = 6, y = 3 (b) x = 3, y = 6 (c) x = –6, y = –3 (d) x = –3, y = –6
Solution. Let A (1, 2), B (4, y), C (x, 6) and D (3, 5) be the vertices of a parallelogram ABCD.
Since, the diagonals of a parallelogram bisect each other
JK x + 1 6 + 2 NO JK 3 + 4 5 + y NO D (3, 5) C (x, 6)
\ KK , OO = KK , OO
L 2 2 P
L 2 2 P
x+1 7 P
fi =   fi  x = 6
2 2
5+y A (1, 2) B (4, y)
and = 4  fi  y = 3
2
Thus, x = 6 and y = 3.
H
ence, option (a) is the correct answer.

Exercise 4.2
A. Multiple Choice Questions (MCQs)
Choose the correct answer from the given options:
1. The point which lies on the perpendicular bisector of the line segment joining the points
A(– 2, – 5) and B(2, 5) is
(a) (0, 0) (b) (0, 2) (c) (2, 0) (d) (– 2, 0)
2. The mid-point of the line segment joining the points (–5, 7) and (–1, 3) is
(a) (–3, 7) (b) (–3, 5) (c) (–1, 5) (d) (5, –3)

86  Mathematics–10
D:\EG_Mathematics_Standard-10_(Term-1)_(30-07-2021)--\Open_Files\Ch-4\Ch-4
\ 31-Jul-2021    Praveen Kumar  Proof-3 Reader’s Sign _______________________ Date __________

If (3, –6) is the mid-point of the line segment joining (0, 0) and (x, y), then the point (x, y) is
3.
3 
(a) (–3, 6) (b) (6, –6) (c) (6, –12) (d)  , − 3
2 
If the point P(k, 0) divides the line segment joining the points A(2, –2) and B(–7, 4) in the ratio 1 : 2,
4.
then the value of k is [CBSE Standard 2020]
(a) 1 (b) 2 (c) –2 (d) –1
The centre of a circle whose end points of a diameter are (–6, 3) and (6, 4) is  [CBSE Standard 2020]
5.
 7  7
(a) (8, –1) (b) (4, 7)
(c)  0,  (d)  4, 
 2  2
6. The point which divides the line segment joining the points (8, –9) and (2, 3) in ratio 1 : 2 internally
lies in the [CBSE Standard SP 2019-20]
(a) I quadrant (b) II quadrant (c) III quadrant (d) IV quadrant
7. If the point C(–1, 2) divides internally the line segment joining A(2, 5) and B in ratio 3 : 4, the
coordinates of B are
(a) B(5, 2) (b) B(–5, –2) (c) B(–7, 2) (d) B(7, –2)
8. The point which divides the line segment joining the points A(0, 5) and B(5, 0) internally in the ratio
2 : 3 is
(a) (2, 3) (b) (3, 4) (c) (–2, 3) (d) (3, –5)
9. If the point C(k, 4) divides the line segment joining two points A(2, 6) and B(5, 1) in ratio 2 : 3, the
value of k is
15 7 16 9
(a) (b) (c) (d)
9 8 5 10
The coordinates of a point A, where AB is a diameter of the circle with centre (–2, 2) and B is the
10.
point with coordinates (3, 4) are
(a) A(7, 0) (b) A(5, 0) (c) A(–5, 0) (d) A(–7, 0)
The ratio in which the line segment joining the points (6, 4) and (1, –7) is divided by x-axis is
11.
(a) 4 : 7 (b) 4 : 5 (c) 4 : 9 (d) 4 : 11
The coordinates of a point which divides the line AB, A (1, 3), B (2, –1) in the ratio 3 : 2 internally are
12.
 5 3  8 3  9 7
(a)  ,  (b)  ,  (c)  ,  (d) None of these
 7 7  5 5  4 4
a
The value of a, for which point P (
13. , 2) is the mid-point of the line segment joining the points
3
Q(–5, 4) and R(–1, 0) is
(a) –5 (b) –7 (c) –9 (d) –11
The ratio in which the y-axis divides the line segment joining the points (5, –6) and (–1, –4) is
14.
(a) 1 : 5 (b) 5 : 1 (c) 1 : 7 (d) 7 : 1
If A(1, 2), B(4, 3) and C(6, 6) are three vertices of parallelogram ABCD, coordinates of D are
15.
(a) (3, 5) (b) (2, 7) (c) (4, 9) (d) (3, 8)
The coordinates of the point P which divides the join of A(–2, 5) and B(3, –5) in the ratio 2 : 3 are
16.
(a) (1, 0) (b) (2, 0) (c) (3, 0) (d) (0, 1)
The coordinates of the points, which divide the line segment joining P (2, –3) and Q (–4, –6) into three
17.
equal parts are
(a) (0, 4), (–1, 9) (b) (0, –3), (–1, –7) (c) (0, –4), (–2, –5) (d) None of these

Coordinate Geometry  87
D:\EG_Mathematics_Standard-10_(Term-1)_(30-07-2021)--\Open_Files\Ch-4\Ch-4
\ 31-Jul-2021    Praveen Kumar  Proof-3 Reader’s Sign _______________________ Date __________

The centre of a circle is (2a, a – 7). The value of a if the circle passes through the point (11, –9) and
18.
has diameter 10 2 units is
(a) a = 3 or 6 (b) a = 5 or 3 (c) a = 7 or 4 (d) None of these
The value(s) of x for which the distance between the points P(x, 4) and Q(9, 10) is 10 units, is
19.
(a) 15 or 2 (b) 10 or 9 (c) 17 or 1 (d) None of these
P(–2, 5) and Q(3, 2) are two points. The coordinates of the point R on PQ such that PR = 2QR are
20.
4  2  1 
(a) R  , 3 (b) R  , 5 (c) R  , 7 (d) None of these
3  3  3 
The coordinates of the point which divides the line segment joining the points (4, –3) and (8, 5) in
21.
the ratio 3 : 1 internally are
(a) P(4, 3) (b) P(7, 3) (c) P(3, 5) (d) P(7, 3)
Points P and Q trisect the line segment joining the points A(–2, 0) and B(0, 8) such that P is near to
22.
A. The coordinates of points P and Q, respectively are
 −4 8   −2 16   −3 5   2 1   −7 4   3 6   2 −16 
(a)  ,  ,  ,  (b)  ,  ,  ,  (c)  ,  ,  ,  (d)  , , ( 0, 0)
 3 3  3 3   4 8  3 5  3 8  7 7  3 3 
 2 
If the point P(m, 3) lies on the line segment joining the points A  − , 6 and B(2, 8), the value of
23.
 5 
m is
(a) 3 (b) 2 (c) –3 (d) –4
AP k
Point P divides the line segment joining the points A(–1, 3) and B(9, 8) such that
24. = . If P lies
on the line x – y + 2 = 0, the value of k is PB 1
1 2 2
(a) (b) (c) (d) 0
3 5 3
If the point C(–1, 2) divides internally the line-segment joining the points A(2, 5) and B(x, y) in the
25.
ratio 3 : 4, the value of x2 + y2 is
(a) 21 (b) 29 (c) 31 (d) 35
If the coordinates of points A and B are (–2, –2) and (2, –4) respectively, the coordinates of P such
26.
3
that AP = AB , where P lies on the line segment AB is
7
 −2 −20   2 20   11 7   −11 −7 
(a)  ,  (b)  ,  (c)  ,  (d)  , 
 7 7  7 7 9 9 9 9
B. Assertion-Reason Type Questions
In the following questions, a statement of assertion (A) is followed by a statement reason (R). Choose
the correct choice as:
(a) Both assertion (A) and reason (R) are true and reason (R) is the correct explanation of assertion (A).
(b) Both assertion (A) and reason (R) are true but reason (R) is not the correct explanation of assertion (A).
(c) Assertion (A) is true but reason (R) is false.
(d) Assertion (A) is false but reason (R) is true.
1. Assertion (A): The point (–1, 6) divides the line segment joining the points (–3, 10) and (6, –8) in
the ratio 2 : 7 internally.
Reason (R): Given three points, i.e. A, B, C form an equilateral triangle, then AB = BC = AC.
2. Assertion (A): Mid-point of a line segment divides line in the ratio 1 : 1.
Reason (R): The ratio in which the point (–3, k) divides the line segment joining the points (–5, 4)
and (–2, 3) is 1 : 2.

88  Mathematics–10
D:\EG_Mathematics_Standard-10_(Term-1)_(30-07-2021)--\Open_Files\Ch-4\Ch-4
\ 31-Jul-2021    Praveen Kumar  Proof-3 Reader’s Sign _______________________ Date __________

Case Study Based Questions


The diagrams show the plans for a sun room. It will be built onto the wall of a house. The four walls
I.
of the sun room are square clear glass panels. The roof is made using
• four clear glass panels, trapezium in shape, all the same size
• one tinted glass panel, half a regular octagon in shape  [CBSE Standard SP 2020]

Refer to Top View


1. Find the mid-point of the segment joining the points J (6, 17) and I (9, 16).
 33 15   3 1  15 33   1 3
(a)  ,  (b)  ,  (c)  ,  (d)  , 
 2 2  2 2  2 2  2 2
Refer to Front View
2. The distance of the point P from the y-axis is
(a) 4 (b) 15 (c) 19 (d) 25
Refer to Front View
3. The distance between the points A and S is
(a) 4 (b) 8 (c) 16 (d) 20
Refer to Front View
4. Find the coordinates of the point which divides the line segment joining the points A and B in the
ratio 1 : 3 internally.
(a) (8.5, 2.0) (b) (2.0, 9.5) (c) (3.0, 7.5) (d) (2.0, 8.5)
Refer to Front View
5. If a point (x, y) is equidistant from the Q(9, 8) and S(17, 8), then
(a) x + y = 13 (b) x – 13 = 0 (c) y – 13 = 0 (d) x – y = 13

Coordinate Geometry  89
D:\EG_Mathematics_Standard-10_(Term-1)_(30-07-2021)--\Open_Files\Ch-4\Ch-4
\ 31-Jul-2021    Praveen Kumar  Proof-3 Reader’s Sign _______________________ Date __________

In the sport of cricket the Captain sets the field according to a plan. He instructs the players to take a
II.
position at a particular place. There are two reasons to set a cricket field—to take wickets and to stop
runs being scored.
The following graph shows the position of players during a cricket match.


If the distance between the points showing the players at Gully A(1, 0) and wicketkeeper at B(4, p)
1.
is 5 m, then p =
(a) 4 m (b) 8 m (c) 6 m (d) 9 m
Suppose the length of a line segment joining the players of Mid-off and Mid-on be 10 units. If the
2.
coordinates of its one end are (2, –3) and the abscissa of the other end is 10 units, then its ordinate is
(a) 9, 6 (b) 3, –9 (c) –3, 9 (d) 9, –6
The coordinate of the point on x-axis which are equidistant from the points representing the players
3.
at Cover P(–3, 4) and Mid-wicket Q(2, 5) are
4 
(a) (20, 0) (b) (–23, 0) (c)  , 0 (d) None of these.
5 
The ratio in which (4, 5) divides the line segment joining the points Extra Cover S(2, 3) and Fine Leg
4.
(7, 8) is
(a) 4 : 3 (b) 5 : 2 (c) 3 : 2 (d) 2 : 3
If the points (4, 3) and (x, 5) are on the circular field with centre (2, 4), then the value of x is
5.
(a) 0 (b) 1 (c) 2 (d) 3

Answers and Hints


A. Multiple Choice Questions (MCQs)
1. (a) (0, 0) 2. (b) (–3, 5)
3. (c) (6, –12) 4. (d) –1
 7
5. (c)  0,  6. (d) IV quadrant
 2
(–2, 2) = c m
x+3 y+4
7. (b) B (–5, –2) 8. (a) (2, 3) fi ,
2 2
16 x+3
9. (c) Now, = –2  fi  x = – 4 – 3
5 2
y+4
10. (d) A = (–7, 0) fi x = – 7 and =2
2
Let AB be a diameter of circle with centre O. fi y = 4 – 4  fi  y = 0
Since centre is the mid-point of diameter AB. Thus, coordinates of point A = (–7, 0)
So, coordinates of O = c m
x+3 y+4 JK 8 3 NO
2
,
2 11. (a) 4 : 7 12. (b) KK , OO
L5 5P

90  Mathematics–10
D:\EG_Mathematics_Standard-10_(Term-1)_(30-07-2021)--\Open_Files\Ch-4\Ch-4
\ 31-Jul-2021    Praveen Kumar  Proof-3 Reader’s Sign _______________________ Date __________

13. (c) –9  − 4 8  − 2 16 
22. (a)  ,  or 
c m = c , 2m  3 3 
–5 + (–1) 4 + 0 a ,
, 3 3
2 2 3
a –6 Line segment AB is trisected by points P and
3 = 2 & a = –9 Q such that P is near to A.
14. (b) 5 : 1

15. (a) (3, 5)
Let coordinates of D be (x, y) and P is mid- So, AP : PB = 1 : 2
point of AC and BD. Then, coordinates of P are
1×0 + 2 × ] –2g 1×8 + 2×0  − 4 8
[Diagonals of a parallelogram bisect each ' , 1 i.e. 
 3 3 
,
other] 1+2 1+2
 x + 4 y + 3  1+ 6 2 + 6 Point Q is the mid-point of PB.
\  ,  =  ,   −4 8 
 2 2  2 2  +0 +8
x+4  3 
7 y+3 8 So, coordinates of Q are  ,3 
fi = , =  2 2 
2 2 2 2
fi x + 4 = 7;  y + 3 = 8  − 4 32   −2 16 
i.e. 
 6 6   3 3 
, or ,
fi x = 3;  y = 5
\  Coordinates of D are (3, 5). 23. (d) – 4
Let point P divides AB in ratio k : 1.
16. (d) (0, 1)
6–6 –10 + 15
x = = 0, y = =1
5 5
17. (c) (0, –4), (–2, –5) 18. (b) a = 5 or 3
19. (c) 17 or 1 8 × k + 6 ×1 −3
\ 3 =   fi  k =
Given that, PQ = 10 units k +1 5
fi PQ2 = 100  2  3 2
2k +  −  2 −  −
(x – 9)2 + (4 – 10)2 = 100  5  5 5
Solve to get x = 17  or  x = 1. and m = =
k +1 −3
 4  +1
20. (a) R  , 3 fi m = –4
5
3 
PR : QR = 2 : 1 24. (c)
2
1( −2) + 2 (3) 1(5) + 2 ( 2)  3
R ,  AP k
 2 +1 2 +1  = i.e. AP : PB = k : 1
AB 1
9k − 1 8k + 3 
4  ∴  Coordinates of P are : 
 k + 1 k + 1 
R  , 3 ,
3 
21. (d) (7, 3) x–y+2=0
Let P(x, y) be the required point. Using
section-formula k 1
A(–1, 3) P B(9, 8)
 m1 x2 + m2 x1 m1 y2 + m2 y1 
 ,  = ( x, y )
 m1 + m2 m1 + m2 
3(8) + 1(4) If P lies on x – y + 2 = 0, then P must satisfy it.
x = ,
3+1 9 k − 1  8k + 3 
fi −  + 2 = 0
3(5) + 1( −3) k +1  k +1 
y =
3+1 fi 9k – 1 – 8k – 3 + 2k + 2 = 0
x = 7 fi 3k – 2 = 0
y = 3 2
Thus, (7, 3) is the required point. fi k =
3

Coordinate Geometry  91
D:\EG_Mathematics_Standard-10_(Term-1)_(30-07-2021)--\Open_Files\Ch-4\Ch-4
\ 31-Jul-2021    Praveen Kumar  Proof-3 Reader’s Sign _______________________ Date __________

25. (b) 29 \  Coordinates of point P are


 3 × 2 + 4 × ( −2) 3 × ( −4) + 4 × ( −2) 
=  . 
3+ 4 3+ 4
3x + 8  −2 −20 
 –1 =   fi  x = –5 =  , 
7 7 7 
3× y + 4×5 B. Assertion-Reason Type Questions
Similarly, 2 = 1. (b) Both assertion (A) and reason (R) are true
3+ 4
but reason (R) is not the correct explanation
fi y = –2 of assertion (A).
Hence x2 + y2 = (–5)2 + (–2)2 2. (c) Assertion (A) is true but reason (R) is false.
= 25 + 4 = 29 Case Study Based Questions
 −2 −20   15 33 
26. (a)  , I. 1. (c)  ,  2. (a) 4
 7   2 2
7 
3. (c) 16 4. (d) (2.0, 8.5)
3
Given AP = AB 5. (b) x – 13 = 0
7 II. 1. (a) 4 m 2. (b) 3, –9
fi  AP : PB = 3 : 4 3. (d) None of these. 4. (d) 2 : 3
fi  P divides AB in the ratio 3 : 4 5. (a) 0

Experts’ Opinion
Questions based on following types are very important for Exams. So, students are advised to
revise them thoroughly.
1. To use distance formula for solving different problems.
2. To use section formula for solving different problems.
3. To use mid-point formula for solving related problems.

IMPORTANT FORMULAE
•• The distance between P(x1, y1) and Q(x2, y2) = ( x2 − x1 ) 2 + ( y2 − y1 ) 2 .
•• The distance of a point P(x, y) from the origin = x 2 + y 2 .
•• The coordinates of the point P(x, y) which divides the line segment joining the points A(x1, y1) and B(x2, y2)
 m x + m2 x1 m1 y2 + m2 y1 
internally in the ratio m1 : m2 are  1 2 , .
 m1 + m2 m1 + m2 
 x + x2 y1 + y2 
•• The mid-point of the line segment joining the points P(x1, y1) and Q(x2, y2) is  1 , .
 2 2 
COMMON ERRORS
Errors Corrections
(i) Incorrectly interpreting the internal division (i) In case of internal division, point dividing the line
and external division of a line segment and segment lie on it. But in external division, point
so using wrong formula. dividing the line segment does not lie on it. So, use
correct formula depending upon situations.
(ii) Making mistakes in use of distance formula (ii) To show collinearity of three points, say A, B and C
and condition of collinearity. • using distance formula: find AB, BC and CA and
then show the sum of two shorter lengths is equal
to the longest length.
(iii) Exchanging the ordered pairs and ignoring (iii) Be careful. Do not exchange the order pair and must
the negative sign while solving problems. take negative sign if any.

92  Mathematics–10
D:\EG_Mathematics_Standard-10_(Term-1)_(30-07-2021)--\Open_Files\Ch-5\Ch-5
\ 31-Jul-2021    Praveen Kumar  Proof-3 Reader’s Sign _______________________ Date __________

5 Triangles

Topics Covered
1. Similarity of Triangles
2. Criteria for Similarity of Triangles and Areas of Similar Triangles
3. Pythagoras Theorem

1. Similarity of Triangles
• Two triangles are similar, if
(i) their corresponding angles are equal and
(ii) their corresponding sides are in the same ratio (or proportional).
AB AC
In the triangles ABC and A′B′C′ shown below, ∠A = ∠A′, ∠B = ∠B′, ∠C = ∠C′ and =
A ′B′ A ′C′
BC
= . Therefore, DABC and DA′B′C′ satisfy both the conditions necessary for them to be similar.
B′ C′

  
Hence, DABC ~ DA′B′C′.
Note: All congruent triangles are similar but the similar triangles need not be congruent.
• Basic Proportionality Theorem or Thales Theorem: In a triangle, a line drawn parallel to one side,
to intersect the other two sides in distinct points, divides the two sides in the same ratio.
• Converse of Basic Proportionality Theorem: If a line divides any two sides of a triangle in the same
ratio, then the line must be parallel to the third side.
Example 1. In the given figure, AP = 3 cm, AR = 4.5 cm, AQ = 6 cm, AB = 5 cm, AC = 10 cm. The
length of AD is
A

P R Q

B D C
(a) 4.5 cm (b) 6.5 cm (c) 7 cm (d) 7.5 cm
Solution. Given: AP = 3 cm, AR = 4.5 cm, AQ = 6 cm, AB = 5 cm, AC = 10 cm.
To find: The length of AD

93
D:\EG_Mathematics_Standard-10_(Term-1)_(30-07-2021)--\Open_Files\Ch-5\Ch-5
\ 31-Jul-2021    Praveen Kumar  Proof-3 Reader’s Sign _______________________ Date __________

AP 3
Proof: In DABC, = ...(i) [BPT]
AB 5
AQ 6 3
= = ...(ii) [BPT]
AC 10 5
From (i) and (ii), we get
AP AQ
=   fi PQ   BC [Converse of BPT]
AB AC
AP AR 3 4.5
In DABD, PR   BD  fi  =   fi  =
AB AD 5 AD
4.5 × 5
fi AD = = 7.5 cm
3
Hence, option (d) is the correct answer.
Example 2. In the given figure, MN || QR. If PM = x cm, MQ = 10 cm, PN = (x – 2) cm, NR = 6 cm,
then the value of x is
P

x x–2

M N
cm

6 cm
10

Q R

(a) 5 cm (b) 7 cm (c) 8 cm (d) 12 cm


Solution. Since in DPQR, MN || QR
PM PN
⇒ = [Basic proportionality theorem]
MQ NR
x x−2
⇒ =   ⇒ 6x = 10x – 20  ⇒ 4x = 20  ⇒  x = 5 cm
10 6
Hence, option (a) is the correct answer.
Example 3. If in the given figure, AB  DE and BD  EF, then DC2 =
C

F
D
E

A B
(a) CF + AC (b) CF × AC (c) CF + 2AC (d) None of these
Solution. Given: DABC in which DE  AB and BD  EF.
CD CE
In DABC, DE   AB   fi  = ...(i) [Basic proportionality theorem]
AC BC
CF CE
Again in DCDB, EF   BD   fi  = ...(ii) [BPT]
CD CB

94  Mathematics–10
D:\EG_Mathematics_Standard-10_(Term-1)_(30-07-2021)--\Open_Files\Ch-5\Ch-5
\ 31-Jul-2021    Praveen Kumar  Proof-3 Reader’s Sign _______________________ Date __________

From (i) and (ii), we get


CD CF
=   fi CD2 = CF × AC
AC CD
Hence, option (b) is the correct answer.

Exercise 5.1
A. Multiple Choice Questions (MCQs)
Choose the correct answer from the given options:
1. If in two triangles DEF and PQR, –D = –Q and –R = –E, then which of the following is not true?
EF DF DE EF DE DF EF DE
(a) = (b) = (c) = (d) =
PR PQ PQ RP QR PQ RP QR
2. In DABC and DDEF, –B =–E, –F = –C and AB = 3DE. Then, the two triangles are
(a) congruent but not similar (b) similar but not congruent
(c) neither congruent nor similar (d) congruent as well as similar
AD 3
3. In given figure figure, DE || BC. If = and AE = 2.7
DB 2
cm, then EC is equal to [CBSE Standard 2020]
(a) 2.0 cm
(b) 1.8 cm
(c) 4.0 cm
(d) 2.7 cm
4. In the figure shown along side, DE || BC, ∠ADE = 70° A
and ∠BAC = 50°, then ∠BCA =
(a) 30°
(b) 60° D E
(c) 40°
(d) 45°
B C
5. In the figure, if ∠ACB = ∠CDA, AC = 6 cm and AD = 3 cm,
then the length of AB is
(a) 8 cm
(b) 10 cm
(c) 12 cm
(d) 16 cm

6. In the given figure, if DE || BC, the EC equals


(a) 1 cm
(b) 2 cm
(c) 4 cm
(d) 6 cm

7. In the DABC, D and E are points on side AB and AC respectively such that DE || BC. If AE = 2 cm,
AD = 3 cm and BD = 4.5 cm, then CE equals
(a) 1 cm (b) 2 cm
(c) 3 cm (d) 4 cm

Triangles  95
D:\EG_Mathematics_Standard-10_(Term-1)_(30-07-2021)--\Open_Files\Ch-5\Ch-5
\ 31-Jul-2021    Praveen Kumar  Proof-3 Reader’s Sign _______________________ Date __________

8. In the given figure, D and E are points on AB and AC respectively A


1
such that DE || BC. If AD = BD and AE = 4.5 cm, then AC is D E
3
equal to
(a) 12 cm (b) 14 cm
(c) 16 cm (d) 18 cm B C

B
AM
9. In the given figure, if LM || CB and LN || CD, then
MB M
equals
AN AD L C
(a) (b) A
ND ND
AL
(c) (d) None of these N
AN D

AD AE
10.
In the given figure, ∠D = ∠E and = , then triangle BAC
is a/an DB EC
(a) Scalene triangle
(b) Isosceles triangle
(c) Equilateral triangle
(d) None of these

In the given figure, if PQ || AB and AQ || CB, then that AR2 =


11.
(a) PR + CR
(b) PR·CR
(c) PR – CR
(d) None of these

B. Assertion-Reason Type Questions


In the following questions, a statement of assertion (A) is followed by a statement of reason (R).
Choose the correct choice as:
(a) Both assertion (A) and reason (R) are true and reason (R) is the correct explanation of assertion (A).
(b) Both assertion (A) and reason (R) are true but reason (R) is not the correct explanation of assertion (A).
(c) Assertion (A) is true but reason (R) is false.
(d) Assertion (A) is false but reason (R) is true.
1. Assertion (A): If ∆ABC and ∆PQR are congruent triangles, S
then they are also similar triangles. P
Q R

Reason (R): All congruent triangles are similar but the


similar triangles need not be congruent.
2. Assertion (A): In the given figure, PA || QB || RC || SD.
l
Reason (R): If three or more line segments are perpendiculars A B C D

to one line, then they are parallel to each other.

96  Mathematics–10
D:\EG_Mathematics_Standard-10_(Term-1)_(30-07-2021)--\Open_Files\Ch-5\Ch-5
\ 31-Jul-2021    Praveen Kumar  Proof-3 Reader’s Sign _______________________ Date __________

Case Study Based Questions


I. Scale Factor and Similarity
Scale Factor
A scale drawing of an object is the same shape as the object but a different size.
The scale of a drawing is a comparison of the length used on a drawing to
the length it represents. The scale is written as a ratio.
Similar Figures
The ratio of two corresponding sides in similar figures is called the scale factor.
Length in image
Scale factor =
Corresponding length in object
If one shape can become another using resizing, then the shapes are similar.

Rotation or Turn Reflection or Flip Translation or Slide


Hence, two shapes are similar when one can become the other after a resize, flip, slide or turn.
[CBSE Standard SP 2020-2021]
1. A model of a boat is made on the scale of 1 : 4. The model is 120 cm long. The full size of the boat
has a width of 60 cm. What is the width of the scale model?


(a) 20 cm (b) 25 cm (c) 15 cm (d) 240 cm
2. What will effect the similarity of any two polygons?
(a) They are flipped horizontally. (b) They are dilated by a scale factor.
(c) They are translated down. (d) They are not the mirror image of one another.
3. If two similar triangles have a scale factor of a : b, which statement regarding the two triangles is true?
(a) The ratio of their perimeters is 3a : b (b) Their altitudes have a ratio a : b
a
(c) Their medians have a ratio : b (d) Their angle bisectors have a ratio a2 : b2
2
4. The shadow of a stick 5 m long is 2 m. At
the same time, the shadow of a tree 12.5
m high is
(a) 3 m
(b) 3.5 m
(c) 4.5 m
(d) 5 m

Triangles  97
D:\EG_Mathematics_Standard-10_(Term-1)_(30-07-2021)--\Open_Files\Ch-5\Ch-5
\ 31-Jul-2021    Praveen Kumar  Proof-3 Reader’s Sign _______________________ Date __________

5. Below you see a student’s mathematical model of a farmhouse roof with measurements. The attic
floor, ABCD in the model, is a square. The beams that support the roof are the edges of a rectangular
prism, EFGHKLMN. E is the middle of AT, F is the middle of BT, G is the middle of CT, and H is
the middle of DT. All the edges of the pyramid in the model have length of 12 m.

What is the length of EF, where EF is one of the horizontal edges of the block?
(a) 24 m (b) 3 m (c) 6 m (d) 10 m
II.
Kerala
Kerala is a state in Southern India. The state is known as a tropical paradise of waving
palms and wide sandy beaches.
This map of the Indian province of Kerala shows its area that can be approximated
using a simple straight-sided shape. The shape has two parallel sides 561 km and 216
km long. The other sides are 180 km and 211 km long. Its parallel sides are 100 km
apart.
Shreya observed the shape formed by four straight lines and explored it on her notebook
in different ways shown below.

Shape I Shape II
Refer to Shape I
1. Let ABCD is a trapezium with AB || DC, E and F are points on non-parallel sides AD and BC
AE
respectively such that EF is parallel to AB. Then
=
ED
BF AB BF
(a) (b) (c) (d) None of these.
CD CD FC
2. Here, AB || CD. If DO = 3x – 19, OB = x – 5, CO = x – 3 and AO = 3, the value of x is
(a) 5 or 8 (b) 8 or 9 (c) 10 or 12 (d) None of these.
3. Again AB || CD. If DO = 3x – 1, OB = 5x – 3, AO = 6x – 5 and OC = 2x + 1, then the value of x is
(a) 0 (b) 1 (c) 2 (d) 3
Refer to Shape II
4. In DABC, PQ || BC. If AP = 2.4 cm, AQ = 2 cm, QC = 3 cm and BC = 6 cm, AB and PQ are respectively
(a) AB = 6 cm, PQ = 2.4 cm (b) AB = 4.8 cm, PQ = 8.2 cm
(c) AB = 4 cm, PQ = 5.3 cm (d) AB = 8.4 cm, PQ = 2.8 cm
5. In DDEF, if RS || EF, DR = 4x – 3, DS = 8x – 7, ER = 3x – 1 and FS = 5x – 3, then the value of x is
(a) 1 (b) 2 (c) 8 (d) 10

98  Mathematics–10
D:\EG_Mathematics_Standard-10_(Term-1)_(30-07-2021)--\Open_Files\Ch-5\Ch-5
\ 31-Jul-2021    Praveen Kumar  Proof-3 Reader’s Sign _______________________ Date __________

Answers and Hints


A. Multiple Choice Questions (MCQs) AN AL
DE EF Also, = ...(ii)
1. (b) = ND LC
PQ RP
2. (b) similar but not congruent AM AN
By equating (i) and (ii) , =
3. (b) 1.8 cm 4. (b) 60° MB ND
5. (c) 12 cm DACB ~ DADC (AA criterion) 10. (b) Isosceles triangle
AC AB 11. (b) PR ⋅ CR
⇒ = NIn DPQR, AB || PQ
AD AC
∴ AB = 12 cm AR RB
⇒ =  (Using BPT) ...(i)
6. (b) 2 cm PR RQ


AD
=
AE
[By BPT] In DAQR, BC || AQ
BD EC CR RB
1.5 1 ⇒ =  (Using BPT) ...(ii)
⇒ = AR RQ
3 EC From (i) and (ii), we have
∴ EC = 2 cm AR CR
7. (c) 3 cm =
PR AR
AD AE 3 2
=   ⇒ = ⇒ AR2 = PR ⋅ CR
BD CE 4.5 CE
CE = 3 cm . Assertion-Reason Type Questions
B
8. (d) 18 cm A 1. (a) Both assertion (A) and reason (R) are true
1 and reason (R) is the correct explanation
 AD = BD
3 D E of assertion (A)
AD 1 2. (a) Both assertion (A) and reason (R) are true
\ =    ...(i)
BD 3 and reason (R) is the correct explanation
DE || BC, B C of assertion (A)
AD AE
\ =  [By BPT] Case Study Based Questions
BD EC
I. 1. (c) 15 cm
1 4.5 2. (d) They are not the mirror image of one
So, =  [From (i)]
3 EC another.
⇒ EC = 13.5 cm ⇒ AC = AE + EC 3. (b) Their altitudes have a ratio a : b.
= 4.5 + 13.5 = 18 cm
4. (d) 5 m 5.
(c) 6 m
AN
9. (a) BF
ND II. 1. (c) 2. (b) 8 or 9
By BPT, FC
3. (c) 2

AM AL
= ...(i) 4. (a) AB = 6 cm, PQ = 2.4 cm

MB LC 5. (a) 1

2. Criteria for Similarity of Triangles and Areas of Similar Triangles


There are some characteristic properties of similar triangles which will help us establishing similarity
of two triangles.
• AAA Similarity: If in two triangles, corresponding angles are equal or the two corresponding
angles are equal, then the triangles are similar.
• SSS Similarity: If the corresponding sides of two triangles are proportional, then they are similar.

Triangles  99
D:\EG_Mathematics_Standard-10_(Term-1)_(30-07-2021)--\Open_Files\Ch-5\Ch-5
\ 31-Jul-2021    Praveen Kumar  Proof-3 Reader’s Sign _______________________ Date __________

• SAS Similarity: If in two triangles one pair of corresponding sides are proportional and the included
angles are equal, then the two triangles are similar.
• The ratio of the areas of two similar triangles is equal to the square of the ratio of their
corresponding sides.
Example 1. A girl of height 90 cm is walking away from the base of a lamp-post at a speed of 1.2 m/s.
If the lamp is 3.6 m above the ground, the length of her shadow after 4 seconds is
(a) 1.1 cm (b) 1.6 cm (c) 2.3 cm (d) 3.5 cm
Solution. Let AB be the lamp-post and CD be the girl after walking for 4 seconds away from the lamp-
post. DE is the shadow of the girl. Let DE = x cm.
BD = Distance covered by girl in 4 seconds = 1.2 × 4 = 4.8 m A
Consider DABE and DCDE,
C
∠B = ∠D (Each 90°) 3.6 m
∠E = ∠E(Common) 0.9 m

\ DABE ~ DCDE (AA Similarity) B


x
D E
BE AB BD + DE 3.6
⇒ =   ⇒  =
DE CD DE 0.9
4.8 + x 3.6
⇒ =   ⇒  4.8 + x = 4x  ⇒ 3x = 4.8  ⇒  x = 1.6
x 0.9
⇒  Shadow of girl after walking for 4 seconds is 1.6 m long.
Hence, option (b) is the correct answer.
Example 2. Two poles of height a and b (b > a) are c metres apart. The height h (in metres) of the point
of intersection of the lines joining the top of each pole to the foot of the opposite pole is
ab
(a) ab (b) ab + 2ab (c) (d) None of these
a+b
Solution. Given: Two poles, AB = a, CD = b (b > a), BD = c and AD and BC intersect at E (say).
Consider DABD and DEFD C
Here, ∠ABD = ∠EFD = 90°
∠ADB = ∠EDF(Common)
\ DABD ~ DEFD (AA similarity)
A
EF FD
\ =
b
E
AB BD
h c−x a
⇒ = ...(i) h
a c
Similarly, DCDB ~ DEFB B x F c–x D
(AA similarity) c

EF BF h x hc
\ =   ⇒  =   ⇒  x = ...(ii)
CD BD b c b
From (i) and (ii),
hc
c−
h b = b − h   ⇒  bh = ab – ah  ⇒ (a + b)h = ab  ⇒  h = ab
=
a c b a+b
Hence, option (c) is the correct answer.

100  Mathematics–10
D:\EG_Mathematics_Standard-10_(Term-1)_(30-07-2021)--\Open_Files\Ch-5\Ch-5
\ 31-Jul-2021    Praveen Kumar  Proof-3 Reader’s Sign _______________________ Date __________

Example 3. AD is an altitude of an equilateral triangle ABC. On AD as base, another equilateral triangle


ADE is constructed. Then area (DADE) : area (DABC) =
(a) 1 : 2 (b) 2 : 5 (c) 3 : 4 (d) 3 : 7
Solution. Given: AD ^ BC in equilateral triangle ABC and DADE is another equilateral triangle.
Let BC = x units
3 A
\ AD = × x units
2
  DABC and DADE are equilateral triangle. E
\ DABC ~ DADE
JK 3 ON2
KK O B
KK 2 x OOO D C
2
area DADE AD 3
fi = =K x O =
area DABC BC 2 L P 4
\  area (DADE) : area (DABC) = 3 : 4
Hence, option (c) is the correct answer.
Example 4. In the given figure, DABC ∼ DDEF, BC = 3 cm, EF = 4 cm and area of DABC = 54 cm2.
Then the area of DDEF is
(a) 54 cm2 (b) 88 cm2 (c) 96 cm2 (d) 108 cm2
Solution. Given: DABC ∼ DDEF
2
area of DABC BC
\ = D
area of DDEF EF 2 A

54 32 9
fi = 2 =
area (∆DEF) 4 16
54 × 16
fi area (DDEF) = = 96 cm 2 B
3 cm
C E
4 cm
F
9
Hence, option (c) is the correct answer.

Exercise 5.2
A. Multiple Choice Questions (MCQs)
Choose the correct answer from the given options:
AB BC CA
1. If in two triangles ABC and PQR, = = , then
QR PR PQ
(a) DPQR ~ DCAB (b) DPQR ~ DABC (c) DCBA ~ DPQR (d) DBCA ~ DPQR
2. If DPQR ~ DXYZ, ∠Q = 50° and ∠R = 70°, then ∠X + ∠Y is equal to
(a) 70° (b) 110° (c) 120° (d) 50°
3. Corresponding sides of two similar triangles are in the ratio 9 : 5. Areas of these triangles are in the
ratio
(a) 21 : 85 (b) 81 : 25 (c) 9 : 5 (d) 5 : 9
4. The perimeters of two similar triangles DABC and DPQR are 35 cm and 45 cm respectively, then the
ratio of the areas of the two triangles is
(a) 7 : 9 (b) 28 : 45 (c) 14 : 27 (d) 49 : 81
5. The sides of two similar triangles are in the ratio 2 : 3, then the areas of these triangles are in the ratio
(a) 2 : 5 (b) 4 : 9 (c) 7 : 16 (d) None of these

Triangles  101
D:\EG_Mathematics_Standard-10_(Term-1)_(30-07-2021)--\Open_Files\Ch-5\Ch-5
\ 31-Jul-2021    Praveen Kumar  Proof-3 Reader’s Sign _______________________ Date __________

6. The perimeters of two similar triangles are 25 cm and 15 cm respectively. If one side of the first
triangle is 9 cm, then the corresponding side of second triangle is
(a) 5.4 cm (b) 8 cm (c) 9.5 cm (d) 10 cm
AB 1 ar(∆ABC)
7. Given DABC ~ DPQR, if = , then =
PQ 3 ar(∆PQR)
(a) 1 : 3 (b) 1 : 6 (c) 1 : 9 (d) 1 : 5
8. In the given figures, DPQR ∼ DXYZ. If PQ = 4 cm, QR = 5 cm and XY = 6 cm, then YZ equals.
Z
P R

4 cm 5 cm

Q X Y
6 cm
(a) 4.5 cm (b) 7.5 cm (c) 6.4 cm (d) None of these
9. Let DABC ∼ DDEF, ar (DABC) = 169 cm2 and ar (DDEF) = 121 cm2. If AB = 26 cm, then DE equals
(a) 11 cm (b) 22 cm (c) 33 cm (d) 44 cm
AB 1
= , then ar ( ∆ΑΒ C )
10. Given DABC~DPQR, if is
PQ 3 ar(∆PQR)

(a) (b) 1 (c) 2 (d)


1 5
9 11 5 9
11. In the given figure, DE || BC, AD = 1 cm and BD = 2 cm. What is the ratio of the
ar (∆ABC) to the ar (∆ADE)?
(a) 1 : 9 (b) 1 : 7
(c) 9 : 1 (d) 7 : 1
12. DABC is isosceles with AC = BC. If AB2 = 2AC2, then the measure of ∠C is
(a) 30° (b) 60°
(c) 45° (d) 90°
13. In given figure, ST || RQ, PS = 3 cm and SR = 4 cm. The ratio of the area of
DPST to the area of DPRQ is
(a) 2 : 25 (b) 9 : 49
(c) 11 : 30 (d) 5 : 28
14. In the given figure, DEFG is a square and ∠BAC = 90°.
Then FG2 =
(a) BG × FC (b) BG + FC
BG
(c) (d) None of these
FC
15. In the given figure, DABC and DXYZ are shown. If AB = 3 cm, BC
= 8 cm, AC = 2 3 cm, ∠A = 80°, ∠B = 60°, XY = 4 3 cm, YZ
= 12 cm and XZ = 6 cm, then the value of ∠Y is


(a) 10° (b) 40° (c) 35° (d) 75°

102  Mathematics–10
D:\EG_Mathematics_Standard-10_(Term-1)_(30-07-2021)\Open_Files\Ch-5\Ch-5
\ 13-Sep-2021    Praveen Kumar  Proof-3 Reader’s Sign _______________________ Date __________

16. In the given figure, M is mid-point of side CD of a parallelogram



ABCD. The line BM is drawn intersecting AC at L and AD
produced at E. Then EL =
1
(a) BL (b) BL
2
(c) 2BL (d) None of these

BE
17. In the given figure, DB ⊥ BC, DE ⊥ AB and AC ⊥ BC. Then =
DE
AC AC
(a) (b)
AE DE
BC AC
(c) (d)
DE BC
18. Right angled triangles BAC and BDC are right angled at A and D and they are on same side of BC.
If AC and BD intersect at P, then AP × PC equals
(a) PB × DP (b) BD × DP (c) AP × DP (d) None of these
AB
19. AD and PM are medians of triangles ABC and PQR respectively where ∆ABC ∼ ∆PQR. Then
PQ
equals
AC AD AC AD
(a) (b) (c) (d)
MR MP PM QM
20. ABC is a triangle. PQ is a line segment intersecting AB in P and AC in Q such tht PQ  BC and divides
BP
DABC into two parts equal in area. Then is equal to
AB
(a) 2 + 2 : 2 (b) 2 − 2 : 2 (c) 2 + 3 : 3 (d) 2 − 3 : 3
21. In a trapezium ABCD, AC and BD intersecting at O, AB || DC and AB = 2CD, if area of DAOB = 84 cm2,
the area of DCOD is
(a) 12 cm2 (b) 18 cm2 (c) 21 cm2 (d) 27 cm2
B. Assertion-Reason Type Questions
In the following questions, a statement of assertion (A) is followed by a statement of reason (R).
Choose the correct choice as:
(a) Both assertion (A) and reason (R) are true and reason (R) is the correct explanation of assertion (A).
(b) Both assertion (A) and reason (R) are true but reason (R) is not the correct explanation of
assertion (A).
(c) Assertion (A) is true but reason (R) is false.
(d) Assertion (A) is false but reason (R) is true.
1. Assertion (A): In the given figures, ∆ABC ~ ∆GHI.
Reason (R): If the corresponding sides of two triangles
are proportional, then they are similar.
2. Assertion (A): The sides of two similar triangles are in
the ratio 2 : 5, then the areas of these triangles are in the
ratio 4 : 25.
Reason (R): The ratio of the areas of two similar triangles
is equal to the square of the ratio of their sides.

Triangles  103
D:\EG_Mathematics_Standard-10_(Term-1)_(30-07-2021)--\Open_Files\Ch-5\Ch-5
\ 31-Jul-2021    Praveen Kumar  Proof-3 Reader’s Sign _______________________ Date __________

Case Study Based Questions


I.
A Frame House: A frame-house is a house constructed from a wooden skeleton, typically covered with
timber board. The concept of similar triangles is used to construct it. Look at the following picture:
A
D
P

B C

Q R

House (i) House (ii)

1. The front view of house (i) is


shown along side in which point
P on AB is joined with point Q on
AC.
If PQ || BC, AP = x m, PB = 10 m, AQ = (x – 2) m, QC = 6 m, then
the value of x is
(a) 3 m (b) 4 m
(c) 5 m (d) 8 m
2. The side view of house (i) is shown below in which point F on AC
is joined with point G on DE.
If ACED is a trapezium with AD || CE, F and G are points on non-
parallel sides AC and DE respectively such that FG is parallel to
AF
AD, then =
FC
DG AD
(a) (b)
GE CE
AF DG
(c) (d)
GE FC
3. The front view of house (ii) is shown along side in which point S
on PQ is joined with point T on PR.
PS PT
If = and ∠PST = 70°, ∠QPR = 50°, then angle
QS TR
∠QRP =
(a) 70° (b) 50°
(c) 80° (d) 60°

104  Mathematics–10
D:\EG_Mathematics_Standard-10_(Term-1)_(30-07-2021)--\Open_Files\Ch-5\Ch-5
\ 31-Jul-2021    Praveen Kumar  Proof-3 Reader’s Sign _______________________ Date __________

4. Again consider the front view of house (ii). If S and T are


points on side PQ and PR respectively such that ST || QR and PS
: SQ = 3 : 1. Also TP = 6.6 m, then PR is
(a) 6.9 m
(b) 8.8 m
(c) 10.5 m
(d) 9.4 m

5. Sneha has also a frame house whose front view is shown below
If MN || AB, BC = 7.5 m, AM = 4 m and MC = 2 m, then length
of BN is
(a) 5 m
(b) 4 m
(c) 8 m
(d) 9 m
II. Star of David: The six-pointed Star of David is a common symbol
for Judaism. Known in Hebrew as a Magen David (shield of David),
geometrically it has two equilateral triangles superimposed on each
other.
The diagram representing the Star of David can be shown as below:

AB BC CA
1. If in two triangles ABC and PQR, = = , then
QR PR PQ
(a) DPQR ~ DCAB (b) DPQR ~ DABC (c) DCBA ~ DPQR (d) DBCA ~ DPQR
2. Ayush has drawn a triangle DEF similar to DABC in his notebook such that the perimeters of DABC and
DDEF are representing 27 cm and 15 cm. If one side of the first triangle is 9 cm, then the corresponding
side of second triangle is
(a) 4.5 cm (b) 5 cm (c) 6 cm (d) None of these.
3. In triangle PQR, from S and T, altitudes TQ and SR are drawn. Which of
the following is true?
(a) DRSP ~ DQTP
(b) DRSQ ~ DRTQ
(c) DRSQ ~ DRSP
(d) None of these.
4. If the median of equilateral triangle DABC is 3 cm, then the length of
side of equilateral triangle is
(a) 1 cm (b) 2 cm (c) 3 cm (d) 4 cm
5. If the side of equilateral triangle PQR is 3 3 cm, then the length of the altitude is
(a) 3 cm (b) 4 cm (c) 4.5 cm (d) 5 cm

Triangles  105
D:\EG_Mathematics_Standard-10_(Term-1)_(30-07-2021)--\Open_Files\Ch-5\Ch-5
\ 31-Jul-2021    Praveen Kumar  Proof-3 Reader’s Sign _______________________ Date __________

Answers and Hints


A. Multiple Choice Questions (MCQs) Since corresponding sides of similar triangles are
1. (a) DPQR ~ DCAB 2. (b) 110° proportional,
3. (b) 81 : 25 4. (d) 49 : 81 \ DABC ~ DXZY
5. (b) 4 : 9 6. (a) 5.4 cm \  DABC,
7. (c) 1 : 9  8. (b) 7.5 cm ∠A + ∠B + ∠C = 180°
9. (b) 22 cm ⇒ 80° + 60° + ∠C = 180°
1 ⇒ ∠C = 180° – 140° = 40°
10. (a)
9 ⇒ ∠A = ∠X,
∠B = ∠Z = 60°
=c m = 
ar TABC AB 2 1 2 1
=
ar TPQR PQ 2 3 9  [Corresponding angles of similar triangles]
11. (c) 9 : 1 12. (d) 90° and ∠C = ∠Y = 40°
13. (b) 9 : 49  16. (c) 2BL
14. (a) BG × FC AC
17. (d)
Since, DEFG is a square BC
⇒ DE || BC In Given DB ^ BC,
i.e. ∠1 + ∠2 = 90°
DE ^ AB i.e. ∠E = 90°
AC ^ BC, i.e. ∠C = 90°
In DABC, ∠C = 90°
∠A + ∠2 = 90° ...(i)
Also, ∠1 + ∠2 = 90° (given) ...(ii)
Also, ∠ADE = ∠GBD...(i) From (i) and (ii), we get
 (Corresponding angles) ∠1 + ∠2 = ∠2 + ∠A 
and ∠AED = ∠FCE...(ii) ⇒ ∠1 = ∠A
 (Corresponding angles) Now, in DDEB and DBCA
In DADE and DGBD, ∠E = ∠C (Each 90°)
∠ADE = ∠GBD [From (i)] ∠1 = ∠A (Proved above)
∠DAE = ∠DGB [Both 90°] \ DDEB ~ DBCA
DADE ~ DGBD (AA Similarity) ...(iii) (By AA similarity)
In DADE and DFEC, BE DE BE AC
∠AED = ∠FCE [From (ii)] ⇒ = ⇒ =
AC BC DE BC
∠DAE = ∠EFC [Both 90°]
18. (a) PB × DP
DADE ~ DFEC (AA Similarity) ...(iv)
From (iii) and (iv), In DAPB and DDPC,
DGBD ~ DFEC
GD GB
⇒ =
FC FE
⇒ GD × FE = GB × FC ...(v)
Since DEFG is a square
⇒ GD = FE = FG = DE
So, equation (v) becomes FG × FG = FC × GB
⇒ FG2 = FC × GB ∠BAP = ∠CDP = 90° [Given]
15. (b) 40° ∠APB = ∠DPC
In DABC and DXYZ,  [Vertically opposite angles]
AB BC AC 1 \ DAPB ~ DDPC
= = =   [by AA similarity of triangles]
XZ YZ XY 2

106  Mathematics–10
D:\EG_Mathematics_Standard-10_(Term-1)_(30-07-2021)--\Open_Files\Ch-5\Ch-5
\ 31-Jul-2021    Praveen Kumar  Proof-3 Reader’s Sign _______________________ Date __________

AP PB 21. (c) 21cm2



\ =  In DAOB and DCOD,
DP PC
∠1 = ∠4
AP × PC = PB × DP
∠2 = ∠3 [Alternate angles]
AD \ DAOB ~ DCOD
19. (b)
MP
Given: ∆ABC and ∆PQR.
AD is the median of ∆ABC, PM is the median
of ∆PQR and ∆ABC ∼ ∆PQR.

ar ( ∆AOB) (AB) 2

⇒ = 
1 ar ( ∆COD) (CD) 2
BD = CD =
BC ( AD is median)
2
84 (2CD) 2
and
1
QM = RM = QR ( PM is median)
⇒ =
2 ar (∆COD) (CD) 2
Since ∆ABC ∼ ∆PQR 84 4(CD) 2
AB BC AC
⇒ = 
So, = =  ar (∆COD) (CD) 2
PQ QR PR
84 4
( corresponding sides of similar
⇒ =
triangles are proportional) ar (∆COD) 1
AB BC AB 2BD \ ar DCOD = 21 cm2

Now, =   ⇒  = ...(i)
PQ QR PQ 2QM B. Assertion-Reason Type Questions
Also, since ∆ABC ∼ ∆PQR 1. (a) Both assertion (A) and reason (R) are true
∠B = ∠Q and reason (R) is the correct explanation of
(Corresponding angles of similar assertion (A)
triangles are equal) ...(ii) 2. (a) Both assertion (A) and reason (R) are true
In ∆ABD and ∆PQM, and reason (R) is the correct explanation of
∠B = ∠Q [From (ii)] assertion (A)
AB BD
= [From (i)] Case Study Based Questions
PQ QM
∴ ∆ABD ∼ ∆PQM [SAS similarity] DG
I. 1. (c) 5 m 2. (a)
Since corresponding sides of similar triangles are GE
proportional. 3. (d) 60° 4. (b) 8.8 m
AB AD 5. (a) 5m
So, =
PQ PM II. 1. (a) DPQR ~ DCAB 2. (b) 5 cm
20. (b) 2 − 2 : 2 3. (a) DRSP ~ DQTP 4. (b) 2 cm
5. (c) 4.5 cm

3. Pythagoras Theorem
• Pythagoras Theorem: In a right triangle, the square of the hypotenuse is equal to the sum of the
squares of the other two sides.
• Converse of Pythagoras Theorem: In a triangle, if the square of one side is equal to the sum of the
squares of the other two sides, then the angle opposite to the first side is a right angle.
Example 1. If two sides of a right triangle are 9 cm and 12 cm, then its third side will be
(a) 21 cm (b) 15 cm (c) 3 cm (d) None of these

Triangles  107
D:\EG_Mathematics_Standard-10_(Term-1)_(30-07-2021)--\Open_Files\Ch-5\Ch-5
\ 31-Jul-2021    Praveen Kumar  Proof-3 Reader’s Sign _______________________ Date __________

Solution: The given triangle is a right triangle.


(9 cm)2 + (12 cm)2 = (81 + 144)2 = 225 cm2
and (15 cm)2 = 225 cm2
  Hence, option (b) is the correct answer.
1
BC. Then 9AD2 =
Example 2. In an equilateral triangle ABC, D is a point on the side BC such that BD =
3
(a) 7 AB2 (b) 5 AB2 (c) 8 AB2 (d) 11 AB2
1
Solution:  AL ^ BC  ⇒ BL = LC = BC
2 A
(In an equilateral triangle, the perpendicular from the vertex bisects
the base.)
1 2
Also, BD = BC and DC = BC
3 3
Consider right triangle ALD.
AD2 = AL2 + DL2 ...(i)
and in right triangle ALB, AB2 = AL2 + BL2 ...(ii) B C
D L
Subtracting (i) from (ii), we get
AB2 – AD2 = BL2 – DL2 = (BL + DL) (BL – DL)
1 
⇒ AB2 – AD2 =  BC + BL − BD BD
2 
1 BC BC  BC 2 BC 2
⇒ AB2 – AD2 =  BC + −  = BC × = BC2
2 2 3  3 3 3 9
⇒ 9AB2 – 9AD2 = 2AB2 (BC = AB)
⇒ 7AB2 = 9AD2
  Hence, option (a) is the correct answer.
Important result: In any triangle, the sum of the squares of any two sides is equal
to twice the square of half of the third side together with twice the square of the
median which bisects the third side.
In DABC, AD is a median so that
AB2 + AC2 = 2 (BD2 + AD2)

Exercise 5.3
A. Multiple Choice Questions (MCQs)
Choose the correct answer from the given options:
1. If in ΔABC, AB = 9 cm, BC = 40 cm and AC = 41 cm, then the ΔABC is a/an
(a) Acute angled triangle (b) Right triangle
(c) Obtuse angled triangle (d) Isosceles triangle
2. In ΔABC, ∠B = 90° and BD ⊥ AC. If AC = 9 cm and AD = 3 cm, then BD is equal to
(a) 2 2 cm (b) 3 2 cm (c) 2 3 cm (d) 3 3 cm
3. In DABC, AB = 6 3 cm , AC = 12 cm and BC = 6 cm, then ∠B =
(a) 30° (b) 60° (c) 90° (d) 45°
4. If in an equilateral triangle, the length of the median is 3 cm, then the length of the side of equilateral
triangle is
(a) 1 cm (b) 2 cm (c) 3 cm (d) 4 cm

108  Mathematics–10
D:\EG_Mathematics_Standard-10_(Term-1)_(30-07-2021)--\Open_Files\Ch-5\Ch-5
\ 31-Jul-2021    Praveen Kumar  Proof-3 Reader’s Sign _______________________ Date __________

5. A man goes 12 m due west and then 9 m due north. How far is he from the starting point?
(a) 12 m (b) 15 m (c) 18 m (d) 24 m
6. In an equilateal triangle of side 3 3 cm, the length of the altitude is
(a) 3.5 cm (b) 4 cm (c) 4.5 cm (d) 6 cm
7. A ladder is placed against a wall such that its foot is at distance of 5 m from the wall and its top reaches
a window 5 3 m above the ground. The length of the ladder is
(a) 10 m (b) 15 m (c) 18 m (d) 24 m
8. ABC is an isosceles triangle right-angled at C. The AB2 is equal to
AC
(a) AC2 (b) 2AC (c) AC2 (d)
2
9. In the given figure, ABCD is a rectangle. P is the mid-point of
DC. If QB = 7 cm, AD = 9 cm and DC = 24 cm, then ∠APQ
equals
(a) 30°
(b) 60°
(c) 45°
(d) 90°
10. From airport two aeroplanes start at the same time. If the speed
of first aeroplane due North is 500 km/h and that of other due
East is 650 km/h, then the distance between two aeroplanes
after 2 hours will be
(a) 100 km (b) 100 157 km (c) 100 269 km (d) None of these
11. An aeroplane leaves an airport and flies due north at a speed of 1000 km per hour. At the same
time, another aeroplane leaves the same airport and flies due west at a speed of 1200 km per hour.
How far apart will be the two planes after 1½ hours?
(a) 100 61 km (b) 51 61 km (c) 300 61 km (d) None of these
12. In DABC, ∠ABC = 90°. AD and CE are two medians drawn from A and C, respectively. If AC = 5 cm
3 5
and AD = cm , the length of CE is
2
(a) 2 5 cm (b) 3 5 cm (c) 4 5 cm (d) 5 cm
13. In an isosceles triangle PQR, PQ = QR and PR2 = 2PQ2. Then ∠Q is
(a) 30° (b) 60° (c) 90° (d) None of these
B. Assertion-Reason Type Questions
In the following questions, a statement of assertion (A) is followed by a statement of reason (R).
Choose the correct choice as:
(a) Both assertion (A) and reason (R) are true and reason (R) is the correct explanation of assertion (A)
(b) Both assertion (A) and reason (R) are true but reason (R) is not the correct explanation of assertion (A).
(c) Assertion (A) is true but reason (R) is false.
(d) Assertion (A) is false but reason (R) is true.
1. Assertion (A): If two sides of a right angle are 7 cm and 8 cm, then its third side will be 9 cm.
Reason (R): In a right triangle, the square of the hypotenuse is equal to the sum of the squares of the
other two sides.
2. Assertion (A): In the ∆ABC, AB = 24 cm, BC = 10 cm and AC = 26 cm, then ∆ABC is a right angle
triangle.
Reason (R): If in two triangles, their corresponding angles are equal, then the triangles are similar.

Triangles  109
D:\EG_Mathematics_Standard-10_(Term-1)_(30-07-2021)--\Open_Files\Ch-5\Ch-5
\ 31-Jul-2021    Praveen Kumar  Proof-3 Reader’s Sign _______________________ Date __________

Case Study Based Questions


I.
Rahul is studying in X Standard. He is making a kite to fly it on a Sunday. Few questions came to his
mind while making the kite. Give answers to his questions by looking at the figure.

1. Rahul tied the sticks at what angles to each other?


(a) 30° (b) 45° (c) 90° (d) 60°
2. Which is the correct similarity criteria applicable for smaller triangles at the upper part of this kite?
(a) RHS (b) SAS (c) SSA (d) AAS
3. Sides of two similar triangles are in the ratio 4:9. Corresponding medians of these triangles are in the
ratio
(a) 2:3 (b) 4:9 (c) 81:16 (d) 16:81
4. In a triangle, if square of one side is equal to the sum of the squares of the other two sides, then the angle
opposite to the first side is a right angle. This theorem is called as,
(a) Pythagoras theorem (b) Thales theorem
(c) Converse of Thales theorem (d) Converse of Pythagoras theorem
5. W hat is the area of the kite, formed by two perpendicular sticks of length 6 cm and
8 cm?
(a) 48 cm2 (b) 14 cm2 (c) 24 cm2 (d) 96 cm2
II. Rohan wants to measure the distance of a pond during the visit to his native. He marks points A and
B on the opposite edges of a pond as shown in the figure below. To find the distance between the
points, he makes a right-angled triangle using rope connecting B with another point C at a distance
of 12 m, connecting C to point D at a distance of 40 m from point C and connecting D to the point at
A which is at a distance of 30 m from D such that ADC = 90º
B 12 m
A C
30

m
40
m

D
1. Which property of geometry will be used to find the distance AC?
(a) Similarity of triangles (b) Thales Theorem
(c) Pythagoras Theorem (d) Congruency of triangles

110  Mathematics–10
D:\EG_Mathematics_Standard-10_(Term-1)_(30-07-2021)--\Open_Files\Ch-5\Ch-5
\ 31-Jul-2021    Praveen Kumar  Proof-3 Reader’s Sign _______________________ Date __________

2. What is the distance AC?


(a) 50 m (b) 12 m (c) 100 m (d) 70 m
3. Which is the following does not form a Pythagoras triplet?
(a) (7, 24, 25) (b) (15, 8, 17) (c) (5, 12, 13) (d) (21, 20, 28)
4. The length of AB is
(a) 12 m (b) 38 m (c) 50 m (d) 100 m
5. The length of the rope used the
(a) 120 m (b) 70 m (c) 82 m (d) 22 m

Answers and Hints


A. Multiple Choice Questions (MCQs) ⇒ AC = 15 m
1. (b) Right triangle Hence, the man is 15 m away from the starting
2. (b) 3 2 cm  point.
3. (c) 90° 6. (c) 4.5 cm
4. (b) 2 cm
In DABC, AD is altitude
 Let a be the side of equilateral triangle.
Median is also the altitude in an equilateral
triangle.
2
 a

\ (Altitude) 2 +   = (a)2
 2
a2

⇒ ( 3 )2 + = a2
4
[  Altitude = Median = 3 cm ]
12 + a 2 AD is median also [  DABC is equilateral]

⇒ = a2  ⇒  12 + a2 = 4a2
4 3 3
⇒ 3a2 = 12  ⇒  a = 2 cm So, BD = cm
2
5. (b) 15 m In DABD,
Let A be the initial position of the man. He ⇒ (AD)2 + (BD)2 = (AB)2
goes from A to B and then from B to C such 2
 3 3 2
that AB = 12 m, BC = 9 m and ∠ABC = 90°.
⇒ (AD) 2 +  = (3 3 )

 2 
27

⇒ AD 2 + = 27
4
27

⇒ AD2 = 27 –
4
108 − 27

⇒ AD2 =
4
In right triangle ABC, we have 81

⇒ AD2 =
AC2 = AB2 + BC2 4
 [By Pythagoras Theorem] 9

⇒ AD = = 4.5 cm
⇒ AC2 = (12 m)2 + (9 m)2 2
= (144 + 81) m2 = 225 m2 Altitude = 4.5 cm

Triangles  111
D:\EG_Mathematics_Standard-10_(Term-1)_(30-07-2021)--\Open_Files\Ch-5\Ch-5
\ 31-Jul-2021    Praveen Kumar  Proof-3 Reader’s Sign _______________________ Date __________

7. (a) 10 m
10. (c) 100 269 km
 ABC is a right triangle, right angled at B.
11. (c) 300 61 km
12. (a) 2 5 cm
In right-triangle ABD, ∠B = 90°
\ AD2 = AB2 + BD2
 [By Pythagoras Theorem]
2
 BC 

⇒ AD2 = AB2 +  [ BD = DC]
 2 

So, by Pythagoras theorem,
AC2 = AB2 + BC2
52 + (5 3 ) 2 = 25 + 75
=
AC2 = 100 ⇒ AC = 100
⇒ AC = 10 m
Hence, length of the ladder = 10 m.
8. (c) AC2

9. (d) 90°
In DQCP, ∠C = 90° BC 2

⇒ AD2 = AB2 + ...(i)
PQ2 = PC2 + QC2 4
(By Pythagoras theorem) Now, in right triangle EBC, ∠B = 90°,
= 122 + 162 = 144 + 256 CE2 = BC2 + BE2
= 400  [By Pythagoras Theorem]
⇒ PQ = 20 cm 2
2 1 
⇒ CE2 = BC +  AB  [  BE = AE]
2 
AB2

⇒ CE2 = BC2 + ...(ii)
4
Adding (i) and (ii), we get
BC2 AB2
AD2 + CE2 = AB2 + +BC2 +
4 4
5
AD2 + CE2 = (AB +BC )
2 2
   
4
Now, in DADP, ∠D = 90°
So, AP2 = AD2 + DP2 5
AD2 + CE2 = AC2 

 (By Pythagoras theorem) 4
= 92 + 122 = 225 [  By Pythagoras Theorem in
⇒ AP = 15 cm DABC, AC2 = AB2 + BC2]
Now, in DQBA, ∠B = 90° 2
⇒ AQ2 = AB2 + QB2  3 5 5
 2  + CE = 4 × 25
2

(By Pythagoras theorem)  
= 242 + 72 = 625 125 45
⇒ AQ = 25 cm CE2 = −
4 4
 625 = 400 + 225 2
⇒ AQ2 = PQ2 + AP2 CE = 20
⇒ ∠APQ = 90°
\ CE = 20 = 2 5 cm
 (By converse of Pythagoras theorem)

112  Mathematics–10
D:\EG_Mathematics_Standard-10_(Term-1)_(30-07-2021)--\Open_Files\Ch-5\Ch-5
\ 31-Jul-2021    Praveen Kumar  Proof-3 Reader’s Sign _______________________ Date __________

13. (c) 90° . Assertion-Reason Type Questions


B
PR2 = 2PQ2[Given] 1. (d) Assertion (A) is false but reason (R) is true.
= PQ2 + PQ2 2. (b) Both assertion (A) and reason (R) are true
⇒ PR2 = PQ2 + QR2[  PQ = QR] but reason (R) is not the correct explanation
⇒  DPQR is right angled at Q of assertion (A).
[Using converse of Pythagoras Theorem]
\  ∠Q is a right angle. Case Study Based Questions
I. 1. (c) 90° 2. (b) SAS
3. (b) 4 : 9
4. (d) Converse of Pythagoras theorem
5. (a) 48 cm2
II. 1. (c) Pythagoras Theorem
2. (a) 50 m 3. (d) (21, 20, 28)
4. (b) 38 m 5. (c) 82 m

Experts’ Opinion
Questions based on following types are very important for Exams. So, students are advised to
revise them thoroughly.
1. To apply Basic Proportionality Theorem and its converse in triangles and quadrilaterals.
2. To apply criteria for similar triangles for solving problems.
3. Use the theorem “The ratio of the areas of two similar triangles is equal to the square of the ratio of their
corresponding sides.”
4. To use Pythagoras theorem and its converse.

QUICK REVISION NOTES


•• Two polygons of the same number of sides are similar, if (i) their corresponding angles are equal,
(ii) their corresponding sides are in the same ratio.
•• Similar Triangles: Two triangles are said to be similar if, (i) their corresponding angles are equal, (ii)
their corresponding sides are proportional.
•• Basic Proportionality Theorem: In a triangle, a line drawn parallel to one side to intersect the other
two sides in distinct points, divides the two sides in the same ratio.
•• Converse of Basic Proportionality Theorem: If a line divides any two sides of a triangle in the same
ratio, the line must be parallel to the third side.
•• AAA Similarity: If in two triangles, corresponding angles are equal or the two corresponding angles
are equal, then the triangles are similar.
•• SSS Similarity: If the corresponding sides of two triangles are proportional, then they are similar.
•• SAS Similarity: If in two triangles, one pair of corresponding sides are proportional and the included
angles are equal, then the two triangles are similar.
•• The ratio of the areas of two similar triangles is equal to the square of the ratio of their
corresponding sides.
•• Pythagoras Theorem: In a right triangle, the square of the hypotenuse is equal to the sum of the squares
of the other two sides.
•• Converse of Pythagoras Theorem: In a triangle, if the square of one side is equal to the sum of the
squares of the other two sides, then the angle opposite to the first side is a right angle.

Triangles  113
D:\EG_Mathematics_Standard-10_(Term-1)_(30-07-2021)--\Open_Files\Ch-5\Ch-5
\ 31-Jul-2021    Praveen Kumar  Proof-3 Reader’s Sign _______________________ Date __________

COMMON ERRORS
Errors Corrections

(i) Writing incorrectly the similarity of two triangles (i) Similarity of two triangles should be expressed
in symbolic form. symbolically, using correct correspondence of
their vertices. For example, in triangle ABC and
DEF if A corresponds to D, B corresponds to E
and C corresponds to F, we write similarity of
these two triangles as DABC ∼ DDEF.

(ii) Applying incorrect similarity criterion for (ii) Read and understand questions properly and then
deciding the similarity of two triangles. use correct similarity criterion for deciding the
similarity of two triangles.

(iii) Incorrectly interpreting that similar figures are (iii) Remember that all the congruent figures are
also congruent. similar but the converse is not true.

(iv) Incorrectly taking that similar figures are (iv) Congruent figures have equal areas but not for
congruent in areas. similar figures.

(v) Taking incorrect relation A (v) In a right-angled triangle, the square of hypotenuse
such as a2 + b2 = c2 while is equal to the sum of the squares of the other two
proving a triangle of given c
b sides.
sides to be a right triangle. Thus, b2 = a2 + c2
B a C

114  Mathematics–10
D:\EG_Mathematics_Standard-10_(Term-1)_(30-07-2021)--\Open_Files\Ch-6\Ch-6
\ 31-Jul-2021   Praveen Kumar  Proof-2 Reader’s Sign _______________________ Date __________

6 Introduction to Trigonometry

Topics Covered
1. Trigonometric Ratios 2. Trigonometric Identities

1. Trigonometric Ratios
Understanding Trigonometric Ratios
• Let ABC be a right triangle in which ∠A = 90°, side adjacent to ∠B = AB, side opposite to ∠B = AC
and hypotenuse = BC. With reference to angle B, we define the following ratios known as trigonometric
ratios.
Side opposite to ∠B AC C
(i) sine of ∠B = sin B = =

Side opposite to ∠B
Hypotenuse BC
Side adjacent to ∠B AB

H
yp
(ii) cosine of ∠B = cos B = =

ot
en
Hypotenuse BC

us
e
Side opposite to ∠B AC
(iii) tangent of ∠B = tan B = =
Side adjacent to ∠B AB
(iv) cosecant of ∠B = cosec B = 1 = Hypotenuse
= BC
A B
Side adjacent to ∠B
sin B Side opposite to ∠B AC
(v) secant of ∠B = sec B = 1 = Hypotenuse BC
=
cos B Side adjacent to ∠B AB
(vi) cotangent of ∠B = cot B = 1 = Side adjacent to ∠B = AB
tan B Side opposite to ∠B AC
cos B
(vii) tan B = sin B (viii) cot B =
cos B sin B
The values of the trigonometric ratios of an angle do not vary with the lengths of the sides of the
triangle, if the angle remains the same.
• If one trigonometric ratio of an angle is given, the other trigonometric ratios of the angle can be determined.
4
Example 1. If cos A = , then sin A and sec A respectively are
5
3 4 5 3 B
(a) , (b) ,
4 3 3 4
3 5
(c) , (d) None of these
5 4 5k 3k
4
Solution. Given: cos A =
5
4 CA
Since, cos A = = A 4k C
5 AB
\ CA = 4 k, AB = 5 k, where k is a positive number.

115
D:\EG_Mathematics_Standard-10_(Term-1)_(30-07-2021)--\Open_Files\Ch-6\Ch-6
\ 31-Jul-2021   Praveen Kumar  Proof-2 Reader’s Sign _______________________ Date __________

In right-angled triangle ACB, BC = 25k 2 − 16k 2 = 9k 2 = 3k


BC 3k 3 1 5
So = = , sec A =
sin A = =
AB 5k 5 cos A 4
Hence, option (c) is the correct answer.
4 sin θ − cos θ
Example 2. If 4 tan q = 3, then the value of is
4 sin θ + cos θ
1 1 1 1
(a) (b) (c) (d)
2 3 4 5
3 AB
Solution.  4 tan q = 3  fi tan q =   fi tan q =
4 BC
Let AB = 3k, BC = 4k A

Then, AC = 9k 2 + 16k 2 = 5k
(using Pythagoras theorem)
AB 3k 3 BC 4k 4 3k 5k
\ sin q = = = and cos q = = =
AC 5k 5 AC 5k 5
3 4 12 − 4
4× − θ
4sinθ − cos θ 5 5 5 =8× 5 = 8 =1 B
\ = = 4k C
4 sin θ + cos θ 3 4 12 + 4 5 16 16 2
4× +
5 5 5
Hence, option (a) is the correct answer.
Trigonometric Ratios of Some Specific Angles
• Trigonometric Ratios of 45°: C
1 1
(i) sin 45° = (ii) cos 45° =
2 2
(iii) tan 45° = 1 (iv) cosec 45° = 2
(v) sec 45° = 2 (vi) cot 45° = 1
• Trigonometric Ratios of 30° and 60°: 45°

1 3 A B
(i) sin 30° = (ii) cos 30° =
2 2
1
(iii) tan 30° = (iv) cosec 30° = 2 A
3
2
(v) sec 30° = (vi) cot 30° = 3 30°
3
3 1
(vii) sin 60° = (viii) cos 60° =
2 2
2
(ix) tan 60° = 3 (x) cosec 60° = 60°
3 B C
1
(xi) sec 60° = 2 (xii) cot 60° =
3

116  Mathematics–10
D:\EG_Mathematics_Standard-10_(Term-1)_(30-07-2021)--\Open_Files\Ch-6\Ch-6
\ 31-Jul-2021   Praveen Kumar  Proof-2 Reader’s Sign _______________________ Date __________

• The value of sin q or cos q never exceeds 1, whereas the value of sec q or cosec q is always greater
than or equal to 1.
Example 3. The value of (sin 30° + cos 60°)
(a) 1 (b) 2 (c) 0 (d) None of these
1 1 1+1 2
Solution. sin 30° + cos 60° = + = = =1
2 2 2 2
Hence, option (a) is the correct answer.

Example 4. The value of 5 cos 60° + 4 sec 30° − tan 45° is


2 2 2

sin 2 30° + cos 2 30°


32 14 67 19
(a) (b) (c) (d)
35 55 12 33

Solution. 5 cos 60° + 4 sec 30° − tan 45°


2 2 2

sin 2 30° + cos 2 30°


2 2
1  2  5 16 15 + 64 − 12
5  + 4 ×   − (1)2 + −1
2  3 4 3 12 67 67
= = = = =
2 1 3 4 12 12
1  3
2
+
  +  4 4 4 1
2  2 
Hence, option (c) is the correct answer.

Example 5. The value of 3 sin 30° + 4 cos 45° − cot 30° is


2 2

cos 2 30° + sin 2 30°


1 1 2 3
(a) (b) (c) (d)
2 3 5 8
2
1  1  3 4
3× +4×  − ( 3 )2 + −3
 
Solution. 3 sin 30° + 4 cos 45° − cot 30° =
2 2 2 2
2
= 2 2
cos 2 30° + sin 2 30°  3 2 3 1
 1 +
  +   4 4
2 2
3+4–6
2 1
= =
4 2
4
Hence, option (a) is the correct answer.
3 3
Example 6. Given sin (A – B) = and cos (A + B) = . Then A and B respectively are
2 2
(a) 30°, 45° (b) 45°, –15° (c) 60°, 45° (d) None of these
3 3
Solution. Since, sin (A – B) = and cos (A + B) =
2 2
⇒ sin (A – B) = sin 60° and cos (A + B) = cos 30°

Introduction to Trigonometry  117


D:\EG_Mathematics_Standard-10_(Term-1)_(30-07-2021)\Open_Files\Ch-6\Ch-6
\ 13-Sep-2021   Praveen Kumar  Proof-2 Reader’s Sign _______________________ Date __________

⇒ A – B = 60° ...(i)
and A + B = 30° ...(ii)
Solving (i) and (ii), we get
2A = 90°  ⇒  A = 45°, B = – 15°
Hence, option (b) is the correct answer.

Exercise 6.1
A. Multiple Choice Questions (MCQs)
Choose the correct answer from the given options:
1. If triangle ABC is right angled at C, then the value of sec (A + B) is
 [CBSE Standard SP 2019-20]
2
(a) 0 (b) 1 (c) (d) not defined
3
2. If sin q + cos q = 2 cos q, (q ≠ 90°) then the value of tan q is [CBSE Standard SP 2019-20]
(a) 2 − 1 (b) 2 + 1 (c) 2 (d) − 2
3
3. Given that sin a = and cos b = 0, then the value of b – a is [CBSE Standard SP 2019-20]
2
(a) 0° (b) 90° (c) 60° (d) 30°
4. The value of sin 60° ⋅ cos 30° + sin 30° ⋅ cos 60° is
(a) 0 (b) 1 (c) 2 (d) 8
5. Value of cos 0° ⋅ cos 30° ⋅ cos 45° ⋅ cos 60° ⋅ cos 90° is
(a) 0 (b) 1 (c) 2 (d) 9
 1 
6. The value of  sin 2 θ + 2 
=
 1 + tan θ 
(a) 0 (b) 1 (c) 2 (d) 5
7. The value of (1 + tan2 q)(1 – sin q)(1 + sin q) =
(a) 0 (b) 1 (c) 2 (d) None of these
1
8. If tan(A + B) = 3 and tan(A – B) = , A > B, then the value of A is
3
(a) A = 30° (b) A = 60° (c) A = 90° (d) A = 45°
9. The value of sin 60° cos 30° + sin 30° cos 60° is
(a) 1 (b) 2 (c) 11 (d) 0
10. 2 tan2 45° + cos2 30° – sin2 60° equals
(a) 1 (b) 2 (c) 5 (d) 6
11. If sin q = x and sec q = y, then the value of cot q is
1
(a) xy (b) 2xy (c) (d) x + y
xy
3
12. If (1 + cos A)(1 – cos A) = , the value of sec A is
4
(a) 2 (b) –2 (c) ±2 (d) 0
13. If 15 cot A = 8, then the value of cosec A is
15 13 4 17
(a) (b) (c) (d)
12 15 15 15

118  Mathematics–10
D:\EG_Mathematics_Standard-10_(Term-1)_(30-07-2021)--\Open_Files\Ch-6\Ch-6
\ 31-Jul-2021   Praveen Kumar  Proof-2 Reader’s Sign _______________________ Date __________

 5 sin θ − 3 cos θ 
14. If 5 tan q = 3, then what is the value of  ?
 4 sin θ + 3 cos θ 
(a) 0 (b) 1 (c) 2 (d) 3
2 2
15. Evaluate: 4 sin 60° + 3 tan 30° – 8 sin 45° cos 45°
(a) 0 (b) 1 (c) 2 (d) 5
sin 30° + tan 45° − cosec 60°
16. Evaluate: .
sec 30° + cos 60° + cot 45°
3 3+2 3 3−4 3 3 +8
(a) (b) (c) (d) None of these
3 3−2 3 3+4 3 3 −9
17. If sin q = cos q, then find the value of 2 tan q + cos2 q.
2 3 5
(a) (b) (c) (d) 0
3 4 2
5
18. If tan A = , then (sin A + cos A) ⋅ sec A is equal to
12
11 13 12 17
(a) (b) (c) (d)
12 12 19 12
cos 30° + sin 60°
19. The value of is
1 + cos 60° + sin 30°
3 2 1
(a) (b) (c) (d) 0
2 3 2
20. ABC is a triangle right angled at C and AC = 3 BC. Then ∠ABC =
(a) 30° (b) 60° (c) 90° (d) 0°
21. If tan q + cot q = 2, the value of tan 2 θ + cot 2 θ is
(a) 2 (b) 2 (c) 3 (d) 3
22. ABC is right-angled triangle, right-angled at B. If BC = 7 cm and AC – AB = 1 cm, then cos A + sin A
equals
31 51 17 51
(a) (b) (c) (d)
25 61 39 53
3
23. The value of x such that 2 cosec2 30° + x sin2 60° – tan2 30° = 10, is
4
(a) 1 (b) 2 (c) 3 (d) 5
B. Assertion-Reason Type Questions
In the following questions, a statement of assertion (A) is followed by a statement reason (R). Choose
the correct choice as:
(a) Both assertion (A) and reason (R) are true and reason (R) is the correct explanation of assertion (A).
(b) Both assertion (A) and reason (R) are true but reason (R) is not the correct explanation of assertion (A).
(c) Assertion (A) is true but reason (R) is false.
(d) Assertion (A) is false but reason (R) is true.
3
1. Assertion (A): In a right-angled triangle, if tan q = , the greatest side of the triangle is 5 units.
4
Reason (R): (Greatest side)2 = (Hypotenuse)2 = (Perpendicular)2 + (Base)2.

Introduction to Trigonometry  119


D:\EG_Mathematics_Standard-10_(Term-1)_(30-07-2021)--\Open_Files\Ch-6\Ch-6
\ 31-Jul-2021   Praveen Kumar  Proof-2 Reader’s Sign _______________________ Date __________

1 3
2. Assertion (A): In a right-angled triangle, if cos q = and sin q = , then tan q = 3.
2 2
sin θ

Reason (R):
cos θ

Case Study Based Questions


I. Three children were playing with sticks. As they had one stick each of them, they put all the three
sticks together. Finding all the three sticks equal, they pick up the sticks and put them in a triangular
form in such a way that the ends of each stick touch the other. They were surprised. Now they thought
of a plane. They took another stick and put it as in the adjacent figure. The stick AD is just touching
the stick BC. Somehow, they measured each angle. Finding that each angle. ∠A = ∠B = ∠C = 60°
(equal) and ∠BAD = ∠CAD = 30°. Likewise, they measured BD = CD, and ∠ADB = ∠ADC = 90°.
Taking AB = BC = CA = 2a, you are required to answer the following questions:
1. The length of AD is
(a) a (b) 2a (c) 2a (d) 3a
2. Using the above figure, the value of sin 30° is
1 1 3
(a) (b) (c) (d) 1
2 2 2
3. Using the above figure, the value of cos 60° is
1 3
(a) 0 (b) 1 (c) (d)
2 2
4. Using the above figure, the value of tan 30° is
1
(a) 3 (b) 1 (c) 0 (d)
3
5. Using the above figure, the value of cosec 60° is
2 3 1 1
(a) (b) (c) (d)
3 2 2 2
II. Skysails’ is that genre of engineering
science that uses extensive utilization
of wind energy to move a vessel in the
sea water. The ‘Skysails’ technology
allows the towing kite to gain a height
of anything between 100 metres to 300
metres. The sailing kite is made in such
a way that it can be raised to its proper
elevation and then brought back with
the help of a ‘telescopic mast’ that
enables the kite to be raised properly and
effectively.
1. In the given figures, if sin q = cos (3q – 30°), where q and 3q – 30° are acute angles, then the value
of q is
(a) 30° (b) 60° (c) 45° (d) None of these.
2. What should be the length of the rope of the kite sail in order to pull the ship at the angle q (calculated
in part (a) and be at a vertical height of 200 m?
(a) 300 m (b) 400 m (c) 500 m (d) 600 m

120  Mathematics–10
D:\EG_Mathematics_Standard-10_(Term-1)_(30-07-2021)--\Open_Files\Ch-6\Ch-6
\ 31-Jul-2021   Praveen Kumar  Proof-2 Reader’s Sign _______________________ Date __________

3. If BC = 15 m, q = 30°, then AB is
(a) 2 3 m (b) 15 m (c) 24 m (d) 5 3 m
4. Suppose AB = BC = 12 m, then q =
(a) 0° (b) 30° (c) 45° (d) 60°
5. Given that BC = 6 m and q = 45°. The values of AB and AC are respectively
(a) AB = 4 m, AC = 4 2 m (b) AB = 7 m, AC = 7 5 m
(c) AB = 9 m, AC = 9 3 m (d) AB = 6 m, AC = 6 2 m

Answers and Hints


A. Multiple Choice Questions (MCQs) 3 3 –4
1. (d) not defined 2. (a) 2 − 1 15. (a) 0 16. (b)
3 3 +4
3. (d) 30° 4. (b) 1
5. (a) 0 6. (b) 1 5 17
17. (c) 18. (d)
7. (b) 1 8. (d) A = 45° 2 12
9. (a) 1 10. (b) 2
3
1 19. (a)
11. (c) 2
xy
Given sin q = x and sec q = y 20. (b) 60°
1 Here, in DABC, ∠C = 90° and AC = 3 BC
⇒ cos q = [given]
y
cos θ 1 AC
Now, cot q = = ⇒ = 3 ...(i)
sin θ xy BC
A
1 AC
Hence, cot q = Also, tan B =
xy BC
12. (c) ±2
⇒ tan B = 3
3
(1 + cos A)(1 – cos A) =  [using (i)]
4 C B
3 3 ⇒ B = 60°
2
⇒ 1 – cos  A =   ⇒  1 – = cos2 A
4 4 21. (b) 2

1
= cos2 A  ⇒ sec2 A = 4 Given: tan q + cot q = 2
4 Squaring both sides, we get
⇒ sec A = ±2
tan2q + cot2q + 2 = 4
13. (d) cosec A =
17 ⇒ tan2q + cot2q = 2
15 Taking square root on both sides, we get
14. (a) 0
3 tan 2 θ + cot 2 θ = 2
Given 5 tan q = 3  ⇒ tan q =
5 31
sin θ 3 22. (a)
⇒ = 25
cos θ 5
⇒ sin q = 3x and cos q = 5x 23. (c) 3
3
5 sin θ − 3 cos θ 5 × 3x − 3 × 5 x 2 cosec2 30° + x sin2 60° – tan2 30° = 10
\ = 4
4 sin θ + 3 cos θ 4 × 3x + 3 × 5 x 2
 3
2
15 x − 15 x 3  1 
= ⇒ 2 × (2)2 + x ×   – ×   = 10
12 x + 15 x  
2 4  3
0 3 1
= = 0 ⇒ 8 + x × – = 10
27x 4 4

Introduction to Trigonometry  121


D:\EG_Mathematics_Standard-10_(Term-1)_(30-07-2021)--\Open_Files\Ch-6\Ch-6
\ 31-Jul-2021   Praveen Kumar  Proof-2 Reader’s Sign _______________________ Date __________

3x 1 Case Study Based Questions


⇒ = 10 – 8 + 1
4 4 I. 1. (d) 3a 2. (a)
3x 9 9 4 2
⇒ =   ⇒  x = × 1 1
4 4 4 3
3. (c) 4. (d)
⇒ x = 3 2 3
B. Assertion-Reason Type Questions 2
5. (a)
1. (a) Both assertion (A) and reason (R) are true 3
and reason (R) is the correct explanation of II. 1. (a) 30° 2. (b) 400 m
assertion (A).
2. (a) Both assertion (A) and reason (R) are true 3. (d) 5 3 m
4. (c) 45°
and reason (R) is the correct explanation of
5. (d) AB = 6m, AC = 6 2 m
assertion (A).

2. Trigonometric Identities
An equation involving trigonometric ratios of an angle is called a trigonometric identity, if it is true for
all values of the angle(s) involved.
Some trigonometric ratios are listed below:
(i) sin2 A + cos2 A= 1, 0° £ A £ 90° (ii) 1 + tan2 A = sec2 A, 0° £ A < 90°
(iii) cot2 A + 1 = cosec2 A, 0° < A £ 90°
Example 1. The value of sin­6 q + cos6 q + 3sin2 q cos2 q is
1
(a) 0 (b) 1 (c) 2 (d)
4
Solution. sin6 q + cos6 q + 3sin2 q cos2 q
= (sin2 q + cos2 q)3 – 3 sin4 q cos2 q – 3 sin2 q cos4 q + 3 sin2 q cos2 q
= 1 – 3 sin2 q cos2 q (sin2 q + cos2 q) + 3 sin2 q cos2 q
= 1 – 3 sin2 q cos2 q + 3 sin2 q cos2 q = 1
Hence, option (b) is the correct answer.
Example 2. The value of (sin4q – cos4q + 1) cosec2q is
(a) 0 (b) 1 (c) 2 (d) 5
Solution. (sin4 q – cos4 q + 1) cosec2 q = {(sin2 q + cos2 q) (sin2 q – cos2 q) + 1} cosec2 q
= (sin2 q – cos2 q + 1) cosec2 q = (sin2 q + sin2 q) cosec2 q
1
= (2 sin2 q) cosec2 q = 2 sin 2 θ × =2
sin 2 θ
Hence, option (c) is the correct answer.
Example 3. If tan A = n tan B and sin A = m sin B, then cos2 A =
m2 − 1 m2 + 1 1 − m2 1 + m2
(a) 2 (b) 2 (c) (d)
n −1 n +1 1+ m 2
1 − m2
1 n
Solution. tan A = n tan B  ⇒  tan B = tan A  ⇒  cot B =
n tan A
1 m
and sin A = m sin B  ⇒  sin B = sin A  ⇒  cosec B =
m sin A
Now, cosec2 B – cot2 B = 1

122  Mathematics–10
D:\EG_Mathematics_Standard-10_(Term-1)_(30-07-2021)--\Open_Files\Ch-6\Ch-6
\ 31-Jul-2021   Praveen Kumar  Proof-2 Reader’s Sign _______________________ Date __________

m2 n2 m2 n 2 cos 2 A m 2 − n 2 cos 2 A
⇒ − = 1  ⇒  − = 1  ⇒  =1
sin 2 A tan 2 A sin 2 A sin 2 A sin 2 A
⇒ m2 – n2 cos2 A = sin2 A  ⇒  m2 – 1 = n2 cos2 A – cos2 A
m2 − 1
⇒ m2 – 1 = (n2 – 1) cos2 A  ⇒ cos2 A = 2
n −1
Hence, option (a) is the correct answer.

Exercise 6.2
A. Multiple Choice Questions (MCQs)
Choose the correct answer from the given options:
 4 sin θ − cos θ 
1. If 4 tan q = 3, then   is equal to
 4 sin θ + cos θ 
2 1 1 3
(a) (b) (c) (d)
3 3 2 4
2. If sin q – cos q = 0, then the value of (sin4 q + cos4 q) is
3 1 1
(a) 1 (b) (c) (d)
4 2 4
3. If 2 sin θ = 3 , then θ =
(a) 30° (b) 60° (c) 45° (d) 90°
4. If x = 2 sin2 θ, y = 2 cos2 θ + 1, then the value of x + y is
(a) 1 (b) 2 (c) 3 (d) 12
1 + tan A2
5. Simplest form of is
1 + cot 2 A
(a) sin2 A (b) cos2 A (c) sec2 A (d) tan2 A
 1 
6. The value of  sin 2 θ + equals
 1 + tan θ 
2

(a) 1 (b) 8 (c) 11 (d) 24


2
7. The value of (1 + tan q)(1 – sin q)(1 + sin q) is
(a) 0 (b) 1 (c) 8 (d) 17
3  1
8. If 3x = cosec q and = cot q, the value of 3  x 2 − 2  is
x  x 
1 1 1 1
(a) (b) (c) (d)
2 4 3 5
9. If cos q + sin q = 2 cos q, then cos q – sin q equals
sin θ
(a) sin q (b) 2 sin q (c) 2 sin q (d)
2
2 2
10. The value of (cosec q –1) tan q is
(a) 0 (b) 1 (c) 2 (d) 7
2 1
11. The value of cot q – is
sin 2 θ
(a) 0 (b) –1 (c) 2 (d) –8

Introduction to Trigonometry  123


D:\EG_Mathematics_Standard-10_(Term-1)_(30-07-2021)--\Open_Files\Ch-6\Ch-6
\ 31-Jul-2021   Praveen Kumar  Proof-2 Reader’s Sign _______________________ Date __________

12. If cosec q + cot q = x, the value of cosec q – cot q is


x 1
(a) x (b) 2x (c) (d)
2 x
13. If sin A + sin2 A = 1, then the value of the expression (cos2 A + cos4 A) is
(a) 0 (b) 1 (c) 2 (d) 9
cos 2 θ
14. The magnitude of q in the equation = 3 is
cot 2 θ − cos 2 θ
(a) 0° (b) 30° (c) 60° (d) 90°
tan 2 θ
cot 2 θ
15. The value of + is equal to
1 + tan 2 θ 1 + cot 2 θ
(a) 1 (b) 2 (c) 3 (d) None of these
16. The value of (1 + cot q – cosec q) (1 + tan q + sec q) is equal to
(a) 1 (b) 2 (c) 3 (d) 4
2 2
17. If 7 sin A + 3 cos A = 4, then tan A =
1 1 1 1
(a) (b) (c) (d)
2 3 2 3
1
18. If tan (A + B) = 1 and tan (A – B) = . 0° < A + B < 90°. A > B, then the values of A and B
3
respectively are
(a) A = 30°, B = 4.5° (b) A = 37.5°, B = 7.5° (c) A = 15°, B = 30° (d) None of these
19. If sec q + tan q = p, then the value of cosec q is
p2 − 1 p2 − 1 p2 + 1
(a) 2 , − 1 (b) 2 , 1 (c) 2 , − 1 (d) None of these
p +1 p +1 p +2
20. If x = r sin q cos f, y = r sin q sin f, z = r cos q, then x2 + y2 + z2 =
r
(a) r (b) r2 (c) 2r (d)
2
B. Assertion-Reason Type Questions
In the following questions, a statement of assertion (A) is followed by a statement reason (R). Choose
the correct choice as:
(a) Both assertion (A) and reason (R) are true and reason (R) is the correct explanation of assertion (A).
(b) Both assertion (A) and reason (R) are true but reason (R) is not the correct explanation of assertion
(A).
(c) Assertion (A) is true but reason (R) is false.
(d) Assertion (A) is false but reason (R) is true.
1. Assertion (A): sin2 67° + cos2 67° = 1.
Reason (R): For any value of q, sin2 q + cos2 q = 1.
2. Assertion (A): The value of sec2 10° – cot2 80° is 1.
1
Reason (R): The value of sin 30° = .
2
Answers and Hints
A. Multiple Choice Questions (MCQs)
1 1 5. (d) tan2 A 6. (a) 1
1. (c) 2. (c)
2 2 1
7. (b) 1 8. (c)
3. (b) 60° 4. (c) 3 3

124  Mathematics–10
D:\EG_Mathematics_Standard-10_(Term-1)_(30-07-2021)--\Open_Files\Ch-6\Ch-6
\ 31-Jul-2021   Praveen Kumar  Proof-2 Reader’s Sign _______________________ Date __________

15. (a) 1
9. (c) 2 sin q
tan 2 θ cot 2 θ
Here,
cos q + sin q = 2 cos q +
1 + tan θ 2
1 + cot 2 θ
⇒ sin q = 2 cos q – cos q
⇒ sin q = ( 2 –1) cos q tan 2 θ cot 2 θ
= +
( 2 +1) sin q = ( 2 –1)( 2 +1) cos q sec 2 θ cosec 2 θ
⇒ 2 sin q + sin q = cos q [Q sec2 q = 1 + tan2 q and cosec2 q
⇒ 2 sin q = cos q – sin q = 1 + cot2 q]
10. (b) 1 sin 2 θ cos 2 θ cos 2 θ sin 2 θ
= × + ×
1 cos 2 θ 1 sin 2 θ 1
(cosec2 q – 1)·tan2q = cot2q· =1
cot 2 θ
11. (b) – 1 = sin2 q + cos2 q = 1
1 16. (b) 2
cot 2 q – = cot 2 q – cosec 2 q = – 1
sin 2 q 1
1 17. (d)
12. (d) 3
x Given, 7sin2A + 3cos2A = 4
cosec q + cot q = x
As we know that cosec2 q – cot2 q = 1 Dividing both sides by cos2A, we get
⇒ (cosec q – cot q)(cosec q + cot q) = 1 7 tan2A + 3 = 4 sec2A [Q sec2q = 1 + tan2q]
⇒ (cosec q – cot q)x = 1 ⇒ 7 tan2A + 3 = 4(1 + tan2A)
1 ⇒ 7 tan2A + 3 = 4 + 4 tan2A
⇒ cosec q – cot q =
x ⇒ 3tan2A = 1
13. (b) 1
1
sin A + sin2 A = 1 ⇒ tan2A =
⇒ sin A + 1 – cos2 A = 1 3
⇒ sin A – cos2 A = 0 ⇒ tan A =
1
⇒ sin A = cos2 A 3
⇒ sin2 A = cos4 A 18. (b) A = 37.5°, B = 7.5°
⇒ 1 – cos2 A = cos4 A Given: tan (A + B) = 1 = tan 45°
⇒ cos2 A + cos4 A = 1 [Q  tan 45° = 1]
14. (c) 60° ⇒ tan (A + B) = tan 45°
cos 2 θ ⇒ A + B = 45° ...(i)

= 3
cot 2 θ − cos 2 θ Also, tan (A – B) =
1
= tan 30°
cos 2 θ 3 1
⇒ = 3 [Q  tan 30° = ]
 1  3
cos 2 θ  2 − 1 ⇒ tan (A – B) = tan 30°
 sin θ 
1 ⇒ A – B = 30° ...(ii)
⇒ = 3 Adding (i) and (ii)
cosec 2 θ − 1
1 2A = 75°
⇒ = 3 ⇒ A = 37.5°
cot 2 θ
⇒ tan2q = 3 From equation (i), B = 45° – 37.5°
= 7.5°
⇒ tan q = 3
Hence, A = 37.5° and B = 7.5°
⇒ q = 60°

Introduction to Trigonometry  125


D:\EG_Mathematics_Standard-10_(Term-1)_(30-07-2021)--\Open_Files\Ch-6\Ch-6
\ 31-Jul-2021   Praveen Kumar  Proof-2 Reader’s Sign _______________________ Date __________

p2 + 1 p2 + 1
19. (a) , –1
\ cosec q = , –1
p 2 –1 p 2 –1
sec q + tan q = p
20. (b) r2
1 sin q Hint: Find the sum of squares of
+ = p
cos q cos q
x = r sin q cos f...(i)
1 + sin q = p cos q = p 1 – sin 2 θ
y = r sin q sin f...(ii)
(1 + sin q)2 = p2(1 – sin2 q) and z = r cos q  ...(iii)
1 + sin2 q + 2 sin q = p2 – p2 sin2 q
B. Assertion-Reason Type Questions
(1 + p2)sin2 q + 2sin q + (1 – p2) = 0 1. (a) Both assertion (A) and reason (R) are true
D = 4 – 4(1 + p2)(1 – p2) and reason (R) is the correct explanation of
= 4 – 4(1 – p4) = 4p4 assertion (A).
2. (b) Both assertion (A) and reason (R) are true
sin q = –2 ± 4p2 = –1 ± p2 = p2 –1 , –1
4 2 2
but reason (R) is not the correct explanation
2 (1 + p ) (1 + p ) p +1 of assertion (A).

Experts’ Opinion
Questions based on following types are very important for Exams. So, students are advised to
revise them thoroughly.
1. To use trigonometric ratios of some specific angles.
2. To use trigonometric identities.

COMMON ERRORS
Errors Corrections
(i) Incorrectly interpreting that sin A means the (i) sin A is an abbreviation for ‘the sine of
product of ‘sin’ and ‘A’. angle A’. It is not the product of ‘sin’ and A.
Similar interpretations follow for all the other
trigonometric ratios.
(ii) Interpreting incorrectly that trigonometric ratios (ii) Trigonometric ratios are numerical quantities.
represent lengths. Each one of them represents the ratio of one
length to another. They must themselves never
be considered as lengths.
(iii) Interpreting incorrectly that trigonometric ratios (iii) The trigonometric ratios depend on the magnitude
depend on the lengths of the sides of the triangle. of the angle and not upon the lengths of the sides
of the triangle.
(iv) Drawing other types of triangles instead of right- (iv) Any right-angled triangles are to be used and
angled triangles for calculating trigonometric hypotenuse is related to right triangles only.
ratios.
(v) Writing trigonometric ratios without angle. (v) It is not correct. Must write ratios with correct
angles and do sufficient practice.
2 2
(vi) Considering sin q + cos q = 1 and taking its (vi) Remember that square root is taken only for
square root as sin q + cos q = ± 1. whole term i.e., not with + and – sign.

126  Mathematics–10
D:\EG_Mathematics_Standard-10_(Term-1)_(30-07-2021)--\Open_Files\Ch-6\Ch-6
\ 31-Jul-2021   Praveen Kumar  Proof-2 Reader’s Sign _______________________ Date __________

QUICK REVISION NOTES


•• Trigonometry deals with the relationships between sides and angles of a triangle.
•• Trigonometric ratios of the angle A in a right triangle ABC, right-angled at B are defined as:
C
BC AB
(i) sin A = (ii) cos A =
AC AC
BC 1 AC
(iii) tan A = (iv) cosec A = =
AB sin A BC
1 AC 1 AB
(v) sec A = = (vi) cot A = =
cos A AB tan A BC
•• Trigonometric Ratios of some specific angles: A B
q 0° 30° 45° 60° 90°
1 1 3
sin q 0 1
2 2 2
3 1 1
cos q 1 0
2 2 2
1
tan q 0 1 3 ND*
3
2
cosec q ND* 2 2 1
3
2
sec q 1 2 2 ND*
3
1
cot q ND* 3 1 0
3
* ND stands for ‘Not defined’.
•• Trigonometric Relations/Identities:
sin q cos q
(i) tan q = (ii) cot q =
cos q sin q
(iii) sin2 q + cos2 q = 1, 0° £ q £ 90° (iv) sec2 q = tan2 q + 1, 0° £ q < 90°
(v) cosec2 q = 1 + cot2 q, 0° < q £ 90°

IMPORTANT FORMULAE
•• In a right triangle ABC, right-angled at B,
side opposite to angle A side adjacent to angle A
(i) sin A = (ii) cos A =
hypotenuse hypotenuse
side opposite to angle A
sin A 1 A
(iii) tan A = (iv) cosec A = =
side adjacent to angle A cos A sin A
1 1 cos A
(v) sec A = (vi) cot A = =
cos A tan A sin A
B C
• (i) sin2 A + cos2 A = 1, for 0° ≤ A ≤ 90°. (ii) sec2 A – tan2 A = 1, for 0° ≤ A < 90°.
(iii) cosec2 A – cot2 A = 1, for 0° < A ≤ 90°.

Introduction to Trigonometry  127


D:\EG_Mathematics_Standard-10_(Term-1)_(30-07-2021)--\Open_Files\Ch-7\Ch-7
\ 31-Jul-2021   Anjum Azad  Proof-2 Reader’s Sign _______________________ Date __________

7 Areas Related to Circles

Topics Covered
1. Perimeter and Area of a Circle
2. Areas of Sector and Segment of a Circle

1. Perimeter and Area of a Circle


As we know that a circle is a closed curve consisting of a set of all those points of the plane which are
at a constant distance from a fixed point in the plane. The fixed point is called its centre. The constant
distance is called its radius. It is the line segment joining any point on the boundary (circumference) to
centre. The boundary (or perimeter) of a circle is called its circumference.
• Circumference of a circle = 2pr (unit)
• Area of a circle = pr2 unit2
• Area of the circular path formed by two concentric circles of radii r1 and r2 (r1 > r2)
= pr12 – pr22 = p (r12 – r22) (unit)2,
• The distance travelled (covered) by a wheel in 1 round = its circumference = 2pr (unit)
• Total distance covered by a wheel = its circumference × number of rounds taken by it.
Total distance covered
• Number of rounds made by a wheel =
Its circumference
Total distance covered
• Speed of the wheel =
Time taken
5 18
• If speed = x km/hr, then speed = x × m/s and if speed = x m/s, then speed = x × km/hr.
18 5
22
Note: Unless stated otherwise, the value of p is to be taken as .
7
Example 1. The area of the circle, the circumference of which is equal to the perimeter of a square of
side 11 cm is
(a) 122 cm2 (b) 144 cm2 (c) 154 cm2 (d) 180 cm2
Solution. Since, side of the square = 11 cm
\ Perimeter of the square = 4 × 11 cm = 44 cm
Given: Circumference of circle = perimeter of square
22
⇒ 2pr = 44  ⇒  2 × × r = 44  ⇒  r = 7 cm
7
22
\ Area of the circle = pr2 = × (7) 2 = 154 cm2
7
Hence, option (c) is the correct answer.
Example 2. The area of a ring shaped region enclosed between two concentric circles of radii
20 cm and 15 cm is
(a) 330 cm2 (b) 415 cm2 (c) 520 cm2 (d) 550 cm2

128
D:\EG_Mathematics_Standard-10_(Term-1)_(30-07-2021)--\Open_Files\Ch-7\Ch-7
\ 31-Jul-2021   Anjum Azad  Proof-2 Reader’s Sign _______________________ Date __________

Solution. For the given ring shaped region; R = 20 cm and r = 15 cm


\ Required area = pR2 – pr2
= p (R + r) (R – r)
22
= (20 + 15) (20 – 15) cm2 = 550 cm2
7
Hence, option (d) is the correct answer.
Example 3. If the perimeter and area of a circle are numerically equal; its radius will be
(a) 1 unit (b) 2 units (c) 4 units (d) None of these
2
Solution. Let the radius of the circle be r units; then given that: 2pr = pr
⇒ r = 2 units
Hence, option (b) is the correct answer.
Example 4. A wheel has diameter 84 cm. Number of complete revolutions must it make to cover 792
metres will be
(a) 100 (b) 160 (c) 220 (d) 300
22
Solution. Since distance covered by the wheel in 1 round = p × diameter = × 84 cm = 264 cm
7
And, total distance covered = 792 m = 792 × 100 cm
Total distance covered 792 × 100 cm
\  No. of complete revolutions made = = = 300
Distance covered by wheel in 1 round 264 cm
Hence, option (d) is the correct answer.
Example 5. The wheels of a car are of diameter 80 cm each. How many complete revolutions does each
wheel make in 10 minutes when the car is travelling at a speed of 66 km per hour?
(a) 2275 (b) 2650 (c) 3815 (d) 4375
5 66 × 5 × 100
Solution.  Speed = 66 km per hour = 66 × m/s = cm/s
18 18
and, time taken = 10 minutes = 10 × 60 sec = 600 sec
66 × 5 × 100
\ Distance covered in 10 min = Speed × time = × 600 cm = 11,00,000 cm
18
Given, diameter of each wheel = 80 cm
\  Distance covered by each wheel in 1 revolution = its circumference.
22 1760
= p × diameter = × 80 cm = cm
7 7
Total distance covered
⇒  No. of revolutions made by each wheel in 10 min =
Distance covered in 1 revolution
11,00,000 cm
= = 4375
1760
cm
7
Hence, option (d) is the correct answer.

Exercise 7.1
A. Multiple Choice Questions (MCQs)
. Choose the correct answer from the given options:
1. If the perimeter of a circle is equal to that of a square, then the ratio of their areas is
(a) 22 : 7 (b) 14 : 11 (c) 7 : 22 (d) 11 : 14

Areas Related to Circles  129


D:\EG_Mathematics_Standard-10_(Term-1)_(30-07-2021)--\Open_Files\Ch-7\Ch-7
\ 31-Jul-2021   Anjum Azad  Proof-2 Reader’s Sign _______________________ Date __________

2. The area of the square that can be inscribed in a circle of radius 8 cm is


(a) 256 cm2 (b) 128 cm2 (c) 64 2 cm 2 (d) 64 cm2
3. The diameter of a circle whose area is equal to the sum of the areas of the two circles of radii
24 cm and 7 cm is
(a) 31 cm (b) 25 cm (c) 62 cm (d) 50 cm
4. If the area of circle is numerically equal to twice its circumference, then the diameter of the circle is
(a) 4 units (b) 6 units (c) 8 units (d) 12 units
5. The perimeter (in cm) of a square circumscribing a circle of radius a cm is
(a) 2a (b) 4a (c) 6a (d) 8a
6. What is the diameter of a circle whose area is equal to the sum of the areas of the two circles of radii
24 cm and 7 cm?
(a) 20 cm (b) 30 cm (c) 50 cm (d) 80 cm
7. What is the area of the circle that can be inscribed in a square of side 6 cm?
(a) 9 p cm2 (b) 11 p cm2 (c) 16 p cm2 (d) 15 p cm2
8. The area of a quadrant of a circle whose circumference is 25 cm is [Imp.]
(a) 24 cm2 (b) 28 cm2 (c) 32.5 cm2 (d) 38.5 cm2
9. If area of quadrant of a circle is 38.5 cm2, then its diameter is  22 
(a) 10 cm (b) 14 cm  Use π = 7 
(c) 21 cm (d) None of these
10. The areas of two circles are in the ratio 9 : 4, then what is the ratio of their circumferences?
(a) 1 : 2 (b) 2 : 1 (c) 3 : 2 (d) 2 : 3
11. The cost of fencing a circular field at the rate of ` 24 per metre is ` 5280. The radius of the
field is
(a) 15 m (b) 35 m (c) 25 m (d) 30 m
12. The radii of two circles are 8 cm and 6 cm respectively. The radius of the circle having area equal to
the sum of the areas of the two circles is
(a) 5 cm (b) 10 cm (c) 12 cm (d) 15 cm
13. An athlete runs on a circular track of radius 49 m and covers a distance of 3080 m along its boundary.
How many rounds has he taken to cover this distance?  22 
(a) 5 (b) 8 (c) 10 (d) 15 Take π = 7 
14. The area of the largest triangle that can be inscribed in a semi-circle of radius r units will be
r
(a) r sq. units (b) sq. units (c) r2 sq. units (d) 2r sq. units
2
15. The cost of fencing a circular field at the rate of ` 24 per metre is ` 5280. The field is to be ploughed
at the rate of ` 0.50 per m2­. The cost of ploughing the field is  22 
(a) ` 1925 (b) ` 1650 Take π = 7 
(c) ` 2010 (d) ` 2525
16. The area of circle whose circumference is 22 cm is
32 45 55 77
(a) cm2 (b) cm2 (c) cm2 (d) cm2
2 2 2 2
17. A road which is 7 m wide surrounds a circular park whose circumference is 88 m. The area of the
road is
(a) 220 m2 (b) 340 m2 (c) 550 m2 (d) 770 m2
18. In the given figure, a square of diagonal 8 cm is inscribed in a circle. The area of the shaded
region is
1 2 3 2
(a) 12 cm (b) 18 cm2 (c) 24 cm2 (d) 11 cm2
2 7 4 9

130  Mathematics–10
D:\EG_Mathematics_Standard-10_(Term-1)_(30-07-2021)--\Open_Files\Ch-7\Ch-7
\ 31-Jul-2021   Anjum Azad  Proof-2 Reader’s Sign _______________________ Date __________

The diameter of a circle whose circumference is equal to the sum of the circumference of the two
19.
circles of diameters 36 cm and 20 cm is
(a) 22 cm (b) 32 cm (c) 56 cm (d) 84 cm
It is proposed to build a single circular park equal in area to the sum of areas of two circular parks of
20.
diameters 16 m and 12 m in a locality. The radius of the new park is
(a) 10 m (b) 12 m (c) 15 m (d) 18 m
B. Assertion-Reason Type Questions
In the following questions, a statement of assertion (A) is followed by a statement of reason (R).
Choose the correct choice as:
(a) Both assertion (A) and reason (R) are true and reason (R) is the correct explanation of assertion (A).
(b) Both assertion (A) and reason (R) are true but reason (R) is not the correct explanation of assertion (A).
(c) Assertion (A) is true but reason (R) is false.
(d) Assertion (A) is false but reason (R) is true.
1. Assertion (A): If the circumference of a circle is 176 cm, then its radius is 28 cm.
Reason (R): Circumference = 2p × radius.
2. Assertion (A): If the outer and inner diameter of a circular path is 10 m and 6 m respectively, then area
of the path is 16 p m2.
Reason (R): If R and r be the radius of outer and inner circular path respectively, then area of circular
path = p (R2 – r2).

Case Study Based Questions


I. A brooch is a small piece of jewellery which has a pin at the back so it can be fastened on a dress,
blouse or coat.
Designs of some brooch are shown below. Observe them carefully.

A B
Design A: Brooch A is made with silver wire in the form of a circle with diameter 28 mm. The wire
used for making 4 diameters which divide the circle into 8 equal sectors.
Design B: Brooch B is made with two colours — Gold and Silver. Outer part is made with gold. The
circumference of silver part is 44 mm and the gold part is 3 mm wide everywhere.
Refer to Design A
1. The circumference of the brooch is 
(a) 28 mm (b) 44 mm (c) 56 mm (d) 88 mm
2. The total length of the silver wire required is
(a) 180 mm (b) 200 mm (c) 250 mm (d) 280 mm
Refer to Design B
3. The circumference of outer part (golden) is
(a) 48.49 mm (b) 82.2 mm (c) 72.50 mm (d) 62.86 mm
4. The difference in areas of golden and silver parts is
(a) 18p mm2 (b) 44p mm2 (c) 51p mm2 (d) 64p mm2

Areas Related to Circles  131


D:\EG_Mathematics_Standard-10_(Term-1)_(30-07-2021)--\Open_Files\Ch-7\Ch-7
\ 31-Jul-2021   Anjum Azad  Proof-2 Reader’s Sign _______________________ Date __________

A boy is playing with the brooch B. He makes revolution with it along its edge. How many complete
5.
revolutions must it take to cover 80p mm?
(a) 2 (b) 3 (c) 4 (d) 5

Answers and Hints


A. Multiple Choice Questions (MCQs) be circumferences of two circles.
1. (b) 14 : 11 2. (b) 128 cm2 A1 9
Then, A1 : A2 = 9 : 4  ⇒  =
3. (d) 50 cm 4. (c) 8 units A2 4
5. (d) 8a
πr12 9 r 3
6. (c) 50 cm ⇒ =   ⇒  1 =
According to the question, πr22 4 r2 2
pR2 = πr12 + πr22 Now, C1 : C2 = 2pr1 : 2.pr2
2πr1 3
⇒ p(R2) = π(r12 + r22 ) ⇒ R2 = r12 + r22 = =
2πr2 2
 r1 = 24 cm  So, C1 : C2 = 3 : 2
= (24)2 + (7)2  r = 7 cm 
= 576 + 49 = 625  2  11. (b) 35 m
⇒ R = 625   ⇒ R = 25 Total cost
Length of the fence =
∴ Diameter = 2R = 2 × 25 = 50 cm. Rate
7. (a) 9p cm2 ` 5280

Diameter of the circle inscribed in a square     = = 220 m
` 24/metre
= side of square
So, length of fence = Circumference of
∴ 2r = a
a 6
the field
⇒ r = = = 3 cm 22
2 2
\ 220 m = 2pr = 2 × ×r
7
∴ Area of circle = pr = p(3)2 = 9p cm2
2
220 × 7
8. (d) 38.5 cm2 So, r = m = 35 m
2 × 22
2πr 12. (b) 10 cm
+ 2r = 25 cm
4  pr2 = 64p + 36p
 22  ⇒ pr2 = 100p  ⇒  r2 = 100
⇒ pr + 4r = 50 ⇒ r  + 4 = 50
 7  ⇒ r = 10 cm
7 ∴  Radius of the required circle is 10 cm.
⇒ = 7 cm
r = 50 × 13. (c) 10
50
πr 2 22 × 7 × 7 14. (c) r2 square units.
\ Area of quadrant = =
4 7×4
2
= 38.5 cm
9. (b) 14 cm
1 22
Area of quadrant = × × r2
4 7
Base AB of triangle ABC in semicircle is
1 22
= 38.5 = × × r2 constant, i.e., equal to 2r, and maximum
4 7 altitude may be equal to r.
⇒ r = 7 cm
∴ Diameter = 14 cm 1
∴ Area of triangle = base × altitude
10. (c) 3 : 2 2
Let r1 and r2 be the radii of two circles, A1 1 1
= AB × OC = (2r) × r = r2
and A2 be areas of two circles and C1 and C2 2 2

132  Mathematics–10
D:\EG_Mathematics_Standard-10_(Term-1)_(30-07-2021)--\Open_Files\Ch-7\Ch-7
\ 31-Jul-2021   Anjum Azad  Proof-2 Reader’s Sign _______________________ Date __________

∴  Area of triangle in semicircle


⇒ a2 + a2 = 82  ⇒ 2a2 = 64
= r2 square units. 64
15. (a) ` 1925
⇒ a2 =   ⇒  a2 = 32
2
5280
Circumference of field = = 220 m Area of shaded part
24 = Area of circle – Area of square
220 × 7
∴ 2pr = 220 ⇒ r = = 35 m = pr2 – a2 =
22
× 4 × 4 – 32
2 × 22
Now, 7

area of the field =


22
× (35)2 = 22 × 5 × 35 m2  22 2   22 − 14 
= 16  −  = 16 
7  7 1  7 
The cost of ploughing the field
= ` (22 × 5 × 35 × 0.50) 16 × 8 128
= =
= ` 1925 7 7
77 2
16. (d) cm 2 Area of shaded region = 18 cm2
2 7
2pr = 22 cm
19. (c) 56 cm
22 22 × 7 7
r = = ⇒r= 36 20
2π 2 × 22 2 2pr = 2p × + 2p ×
2 2
2
22  7  77 = 36p + 20p = 56p
Area of circle = pr2 = ×  = cm 2
7  2  2 ⇒ r = 28 cm
17. (d) 770 m2 ∴  Diameter of the required circle = 28 cm × 2

Circumference of circular = 56 cm
park = 88 m 20. (a) 10 m
2pr1 = 88 m Let the radius of the new park be R.
88 88 × 7 ∴  Area of new park
⇒ r1 = = = 14 m = Area of old park I + Area of park II
2×π 2 × 22
⇒ pR2 = πr12 + πr22
Width of road = 7 m
So, r2 = r1 + 7 m = 14 + 7 = 21 m
⇒ p(R2) = π(r12 + r22 )
22
So, area of the road = πr22 − πr12 = (212 – 142)
7  r1 = 28 cm 

⇒ p(R2) = p[82 + 62]  r = 6 cm 
22 2 
= (21 + 14)(21 – 14)
⇒ R2 = 64 + 36
7

=
22
× 35 × 7 = 770 m2
⇒ R = 100 = 10 m
7
2 B. Assertion-Reason Type Questions
18. (b) 18 cm2 a 1. (a) Both assertion (A) and reason (R) are true
7 D C

Let the side of the and reason (R) is the correct explanation of
square be a cm. assertion (A).
cm

So, radius of the circle, a a 2. (a) Both assertion (A) and reason (R) are true
8

O
AC and reason (R) is the correct explanation of
r = OA = assertion (A).
2
A a B
Case Study Based Questions
8
⇒ r = = 4 cm I. 1. (d) 88 mm 2. (b) 200 mm
2
So, in right angled DABC, 3. (d) 62.86 mm 4. (c) 51p
AB2 + BC2 = AC2 5. (c) 4

Areas Related to Circles  133


D:\EG_Mathematics_Standard-10_(Term-1)_(30-07-2021)--\Open_Files\Ch-7\Ch-7
\ 31-Jul-2021   Anjum Azad  Proof-2 Reader’s Sign _______________________ Date __________

2. Areas of Sector and Segment of a Circle


A sector is a part of the circular region which is enclosed by two radii and the corresponding arc. Hence,
OABC is a minor sector and OCDA is a major sector. ∠AOC is called the angle of sector.
θ
• Area of the sector of a circle of radius r with central angle θ = × πr 2 , where
q is measured in degrees. 360°
OR
1 1
Area of the sector = × length of arc × radius = lr
2 2
• Length of the arc of the sector of a circle of radius r with central angle
θ
θ= × 2πr , where q is measured in degrees.
360°
• Area of the minor segment APB of the circle in the given figure
= area of sector OAPB – area of DOAB
i 1
= × rr 2 – r2 × sin q
360° 2
• Area of the major sector of a circle of radius r
= pr2 – area of the corresponding minor sector.
Example 1. The area of a sector of a circle with radius 6 cm if angle of the sector is 60° is
2 1 6 3
(a) 15 cm2 (b) 16 cm2 (c) 18 cm2 (d) 19 cm2
3 2 7 8
Solution. Given: r = 6 cm and q = 60°
θ 22 60° 132 6
\ Area of sector = πr 2 × = ×6×6× cm 2 = cm 2 = 18 cm 2
360° 7 360° 7 7
Hence, option (c) is the correct answer.
Example 2. A chord AB of a circle of radius 10 cm subtends a right angle at the centre. The area of the
minor-sector is [Take p = 3.14] [Imp.]
2 2 2
(a) 38.5 cm (b) 42 cm (c) 78.5 cm (d) 82 cm2
Solution:  q = 90° and r = 10 cm
θ
\  Area of the minor-sector OACB = πr ×
2
360°
90° O1
0
= 3.14 × 10 × 10 × cm 2 cm cm
360° 1 0
= 78.5 cm2 A B
Hence, option (c) is the correct answer. C
Areas of Combinations of Plane Figures
Example 3. A square ABCD is inscribed in a circle of radius 10 units. The area of the circle, not included
in the square is (Take p = 3.14)
(a) 84 cm2 (b) 108 cm2 (c) 114 cm2 (d) 122 cm2
Solution. When a square or a rectangle is inscribed in a circle then diagonal of the square is the diameter
of the circle.
\  The diagonal AC of the square ABCD = Diameter of the circle = 2 × 10 cm
= 20 cm
Since, the diagonals of a square are equal and bisect each other at 90°; therefore; AC = BD = 20 cm

134  Mathematics–10
D:\EG_Mathematics_Standard-10_(Term-1)_(30-07-2021)--\Open_Files\Ch-7\Ch-7
\ 31-Jul-2021   Anjum Azad  Proof-2 Reader’s Sign _______________________ Date __________

1
and area of the square = × AC × BD
2
1
= × 20 × 20 cm = 200 cm
2 2
2
Also, area of the circle = pr2 = 3.14 × (10)2 cm2 = 314 cm2
\ The required area = Area of circle – Area of the square
= 314 cm2 – 200 cm2 = 114 cm2.
Hence, option (c) is the correct answer.
Example 4. The area of an equilateral triangle is 17320.5 cm2. With each vertex as centre, a circle is
described with radius equal to half the length of the side of the triangle. The area of the triangle not
included in the circles is (Use p = 3.14 and 3 = 1.73205).
(a) 1620.51 cm2 (b) 1810.25 cm2 (c) 2430.60 cm2 (d) None of these
3
Solution. We know that area of an equilateral triangle = × (side)2
4
3 A
\ × (side)2 = 17320.5
4
60°
2 4 4
⇒ (side) = 17320 . 5 × = 17320 . 5 ×
3 1.73205
⇒ Side = 200 cm. B C
200
⇒  Radius of each circle drawn = cm = 100 cm
2
For each minor sector; q = 60°
θ
\  Area of each sector inside the triangle = πr 2 ×
360°
60°
= 3.14 × (100) 2 × cm 2 = 5233.33 cm2
360°
\  Area of the triangle not included in the circles
= Area of triangle – 3 × area of each sector inside the triangle
= (17320.5 – 3 × 5233.33) cm2 = 1620.51 cm2.
Hence, option (a) is the correct answer.
Example 5. In the given figure, PQ = 24 cm, PR = 7 cm and O is the centre
of the circle. The area of the shaded portion is
(a) 132.58 cm2 (b) 148.20 cm2
2
(c) 154.36 cm (d) 161.54 cm2
Solution. We know that the angle subtended by semi-circle is 90°
⇒ ∠RPQ = 90°,
i.e., DRPQ is a right-angled triangle with hypotenuse = RQ
\ RQ2 = PR2 + PQ2
⇒ RQ2 = (7)2 + (24)2 = 49 + 576 = 625
⇒ RQ = 25 cm = diameter of the circle.
RQ 25
\ Radius of the circle = = cm = 12.5 cm
2 2
Area of given shaded portion = Area of semicircle – Area of right DRPQ
1 2 1 1 22 1 1718.75
= πr − × RP × PQ = × × (12.5) − × 7 × 24 = – 84
2
2 2 2 7 2 7

Areas Related to Circles  135


D:\EG_Mathematics_Standard-10_(Term-1)_(30-07-2021)--\Open_Files\Ch-7\Ch-7
\ 31-Jul-2021   Anjum Azad  Proof-2 Reader’s Sign _______________________ Date __________

1718.75 - 588 1130.75


= = cm 2 = 161.54 cm 2
7 7
Hence, option (d) is the correct answer.
Example 6. In a circular table cover of radius 32 cm, a design is formed
having an equilateral triangle ABC in the middle, as shown below. The area
of the design is  [Imp.]
(a) 777.36 cm2 (b) 1888.11 cm2

(c) 2010.54 cm2 (d) None of these
Solution. Let O be the centre of the circle. Join OB and OC, also draw OD
⊥ BC.
Since, ABC is an equilateral triangle, ∠BAC = 60°.
Also, angle subtended by an arc at the centre of the circle is twice the angle
at remaining circumference.
\ ∠BOC = 2 ∠BAC = 2 × 60° = 120°
Now, by R.H.S., DOBD ≅ DOCD  ⇒  ∠BOD = ∠COD = 60°
In DOBD, OB = radius of the circle = 32 cm
BD 3 BD
⇒ sin 60° = ⇒ =   ⇒  BD = 16 3 = CD
OB 2 32
∴ BC = 2 BD = 2 × 16 3 cm = 32 3 cm
Area of the design = Area of the circle – Area of equilateral DABC
3  22 3 
= πr 2 − × (side) 2 =  × (32) − × (32 3 ) 2  cm 2
2
4 7 4 
 22528 
=  − 768 3  cm2 = 1888.11 cm2
7 
Hence, option (b) is the correct answer.
Example 7. The area of the shaded portion in the figure, given below, where a circular arc of radius 6
cm has been drawn with vertex O of an equilateral triangle OAB of side 12 cm as centre is [Imp.]
(a) 156.64 cm2 (b) 188.46 cm2 (c) 256.64 cm2 (d) 310.25 cm2
Solution. Since, DOAB is equilateral, ∠AOB = 60°
Required area = Area of circle + Area of DOAB
– Area of sector with angle 60°
3 θ
= πr 2 + × (side) 2 − πr 2 ×
4 360°
22 2 3 22 2 60
= ×6 + × (12) 2 − ×6 × cm 2
7 4 7 360
22 3
= (36 − 6) + × 144 cm 2
7 4
 660 
=  + 36 3  = 156.64 cm 2
7 
Hence, option (a) is the correct answer.

136  Mathematics–10
D:\EG_Mathematics_Standard-10_(Term-1)_(30-07-2021)--\Open_Files\Ch-7\Ch-7
\ 31-Jul-2021   Anjum Azad  Proof-2 Reader’s Sign _______________________ Date __________

Example 8. ABCD is a square of side 4 cm. At each corner of the square, a quarter circle of radius 1 cm,
and at the centre, a circle of radius 1 cm, are drawn, as shown in the given figure. The area of the shaded
region is
(a) 8.46 cm2 (b) 7.25 cm2 (c) 9.71 cm2 (d) 10.43 cm2
Solution. Area of the shaded portion A B
= Area of given square ABCD
– 4 × area of each quarter circle
– area of the circle at the centre.
2 cm
 2
= 4 × 4 − 4  π(1)  − π(1) 2 sq. cm
 4 

1 cm
 22 22  5
= 16 − −  cm = 9 cm = 9.71 cm
2 2 2
D C
 7 7 7
Hence, option (c) is the correct answer. 4 cm

Exercise 7.2
A. Multiple Choice Questions (MCQs)
Choose the correct answer from the given options.
1. In a circle of radius 21 cm, an arc subtends an angle of 60° at the centre. The length of the arc is
(a) 11 cm (b) 22 cm (c) 27 cm (d) 44 cm
2. An arc of length 15.7 cm subtends a right angle at the centre of the circle. Then the radius of the circle
is
(a) 20 cm (b) 10 cm (c) 15 cm (d) 12 cm
3. The angle described by a minute hand in 5 minutes is
(a) 30° (b) 60° (c) 90° (d) None of these
4. The area of the sector in the following figure showing a chord AB of [Imp.]
a circle of radius 18 cm subtending an angle of 60° at the centre O is
 [Take p = 3.14] O
(a) 151.31 cm2 (b) 169.56 cm2
(c) 173.33 cm2
60°
(d) None of these
A B
The given figure is a sector of circle of radius 10.5 cm. The perimeter
5. [Imp.]
22
of the sector is [Take p = ]
7
(a) 32 cm (b) 44 cm
(c) 54 cm (d) None of these

22
6. In a circle of diameter 42 cm,if an arc subtends an angle of 60° at the centre where p = , then what
will be the length of arc? 7
(a) 11 cm (b) 20 cm (c) 22 cm (d) 28 cm
7. A horse is tied to a pole with 28 m long rope. The perimeter of the field where the horse can
graze is (Take p = 22/7)
(a) 60 cm (b) 85 cm (c) 124 cm (d) 176 cm
8. A car has two wipers which do not overlap. Each wiper has a blade of length 21 cm sweeping through
22
an angle 120°. The total area cleaned at each sweep of the blades is [Take p = ]
2 2 2 2 7
(a) 360 cm (b) 448 cm (c) 556 cm (d) 924 cm

Areas Related to Circles  137


D:\EG_Mathematics_Standard-10_(Term-1)_(30-07-2021)--\Open_Files\Ch-7\Ch-7
\ 31-Jul-2021   Anjum Azad  Proof-2 Reader’s Sign _______________________ Date __________

The area of the shaded region in the given figure, if AC = 24 cm, BC = 10


9. A
cm and O is the centre of the circle is [Take p = 3.14]
(a) 128.56 cm2
(b) 145.33 cm2 O

(c) 248.16 cm2


(d) None of these
B C
In the given figure, a square OABC is inscribed in a quadrant OPBQ of a
10.
circle. If OA = 20 cm, the area of the shaded region is [Use p = 3.14]
(a) 126 cm2
(b) 200 cm2
(c) 228 cm2
(d) None of these
In the given figure, the shape of the top of a table is that of a sector of a circle
11.
with centre O and ∠AOB = 90°. If AO = OB = 42 cm, then the perimeter
22
of the top of the table is[Take p = ] O
7
(a) 282 cm
(b) 266 cm
(c) 320 cm
A
(d) 480 cm B

In the given figure, APB and CQD are semicircles of diameter


12.
7 cm each, while ARC and BSD are semicircles of diameter
14 cm each. The perimeter of the shaded region is
22
[Take p = ]
7
(a) 22 cm
(b) 33 cm
(c) 44 cm
(d) 66 cm
A piece of wire 22 cm long is bent into the form of an arc of a circle subtending an angle of 60° at its
13.
22
centre. The radius of the circle is [Take p = ]
7
(a) 7 cm (b) 14 cm (c) 21 cm (d) 28 cm
In the given figure, the boundary of shaded region consists of four semicircular
14.
arcs, two smallest being equal. If diameter of the largest is 14 cm and that of
the smallest is 3.5 cm, the area of the shaded region is
22
[Take p = ]
7
2 2
(a) 72.225 cm (b) 86.625 cm
(c) 92.330 cm2 (d) None of these
The area of the major segment APB in figure of a circle of radius 35 cm
15. P
22
and ∠AOB = 90° is [Take p = ]
2 7
(a) 1200 cm O
(b) 2400 cm2
(c) 3500 cm2 90°

(d) None of these A B

138  Mathematics–10
D:\EG_Mathematics_Standard-10_(Term-1)_(30-07-2021)--\Open_Files\Ch-7\Ch-7
\ 31-Jul-2021   Anjum Azad  Proof-2 Reader’s Sign _______________________ Date __________

In the given figure, ABCD is a square of side 14 cm. Semicircles are drawn
16. A B
with each side of square as diameter. The area of the shaded region is
X
22
 [Take p = ]
2 7 O
(a) 84 cm W Y
(b) 96 cm2
(c) 110 cm2 Z

(d) 220 cm2 D C

The short and long hands of a clock are 4 cm and 6 cm long respectively. The sum of distances travelled
17.
by their tips in 2 days is
(a) 1148 cm (b) 1426.35 cm (c) 1910.85 cm (d) None of these
The area of the sector of a circle of radius 5 cm, if the corresponding arc length is 3.5 cm is
18.
(a) 3.25 cm2 (b) 8.75 cm2 (c) 4.60 cm2 (d) 5.50 cm2
The area of the largest circle that can be drawn inside the given rectangle of length ‘a’ cm and breadth
19.
‘b’ cm (a > b) is
1 1 1
(a) πb 2 cm2 (b) πb 2 cm2 (c) πb 2 cm2 (d) pb2 cm2
2 3 4
All the vertices of a rhombus lie on a circle. The area of the rhombus, if the area of the circle is
20.
1256 cm2 is [Use p = 3.14]
(a) 300 cm2 (b) 600 cm2 (c) 800 cm2 (d) 900 cm2
The difference of the areas of two segments of a circle formed by a chord of radius 5 cm subtending
21.
an angle of 90° at the centre is [Imp.]
 25π 25   15π 7   7π 3 
(a)  −  cm2 (b)  −  cm2 (c)  − 2
4 2   4 2   4 2  cm (d) None of these
In the given figure, PSR, RTQ and PAQ are three semicircles of
22.
diameters 10 cm, 3 cm and 7 cm respectively. The perimeter of the
shaded region is [Use p = 3.14]
(a) 22.5 cm
(b) 31.4 cm
(c) 36.6 cm
(d) None of these
There are three semi-circles A, B and C having diameter 3 cm each
23.
and another semicircle E having a circle D with diameter 4.5 cm as
shown in the figure. The area of the shaded region is E D
15 18
(a) p cm2 (b) p cm2
16 19 A C
3 cm B 3 cm
37 63
(c) p cm2 (d) p cm2
54 16
24. The area of the shaded region given in the figure is
(a) (180 – 2p) cm2
(b) (90 – 8p) cm2
(c) (180 – 8p) cm2
(d) (90 – 2p) cm2

Areas Related to Circles  139


D:\EG_Mathematics_Standard-10_(Term-1)_(30-07-2021)--\Open_Files\Ch-7\Ch-7
\ 31-Jul-2021   Anjum Azad  Proof-2 Reader’s Sign _______________________ Date __________

The area of the shaded region in the given figure, if PR = 12 cm, PQ


25. P Q
22
= 5 cm and O is the centre of the circle is [Take p = ]
2 7
(a) 36.39 cm
(b) 48.24 cm2 O

(c) 28.76 cm2


(d) 62.62 cm2 R
B. Assertion-Reason Type Questions
In the following questions, a statement of assertion (A) is followed by a statement of reason (R).
Choose the correct choice as:
(a) Both assertion (A) and reason (R) are true and reason (R) is the correct explanation of assertion (A).
(b) Both assertion (A) and reason (R) are true but reason (R) is not the correct explanation of assertion (A).
(c) Assertion (A) is true but reason (R) is false.
(d) Assertion (A) is false but reason (R) is true.
1. Assertion (A): In a circle of radius 6 cm, the angle of a sector is 60°. Then the area of the sector is
6 2
18 cm .
7
Reason (R): Area of the circle with radius r is pr2.
2. Assertion (A): The length of the minute hand of a clock is 7 cm. Then the area swept by the minute
5
cm2.
hand in 5 minute is 12
6
θ
Reason (R): The length of an arc of a sector of angle q and radius r is given by l =
× 2πr .
360°
Case Study Based Questions
Pookalam is the flower bed or flower pattern designed during Onam in Kerala. It is similar as Rangoli
I.
in North India and Kolam in Tamil Nadu.
During the festival of Onam, your school is planning to conduct a Pookalam competition. Your friend
who is a partner in competition, suggests two designs given below.
Observe these carefully.
A A B

B C
D C
I II
Design I: This design is made with a circle of radius 32 cm leaving equilateral triangle ABC in the
middle as shown in the given figure.
Design II: This Pookalam is made with 9 circular design each of radius 7 cm.
Refer Design I:
1. The side of equilateral triangle is
(a) 12 3 cm (b) 32 3 cm (c) 48 cm (d) 64 cm
2. The altitude of the equilateral triangle is
(a) 8 cm (b) 12 cm (c) 48 cm (d) 52 cm

140  Mathematics–10
D:\EG_Mathematics_Standard-10_(Term-1)_(30-07-2021)--\Open_Files\Ch-7\Ch-7
\ 31-Jul-2021   Anjum Azad  Proof-2 Reader’s Sign _______________________ Date __________

Refer Design II:


3. The area of square is
(a) 1264 cm2 (b) 1764 cm2 (c) 1830 cm2 (d) 1944 cm2
4. Area of each circular design is
(a) 124 cm2 (b) 132 cm2 (c) 144 cm2 (d) 154 cm2
5. Area of the remaining portion of the square ABCD is
(a) 378 cm2 (b) 260 cm2 (c) 340 cm2 (d) 278 cm2
II. Energy conservation is the effect made to reduce the consumption of energy by using less of an energy
service.
Some children of a school prepared posters on ‘Save Energy’ which are shown below.

Save Think Turn the


Energy Green Lights off
Turn off when you
Don’t
Lights leave
Waste it

Poster I Poster II Poster III


Refer to Poster I
1. If radius of the circle is 21 cm and angle subtended by arc at centre is 60°, the length of the arc is
(a) 22 cm (b) 28 cm (c) 33 cm (d) None of these.
2. The area of sector formed by the arc (radius = 21 cm and angle of sector = 60°) is
(a) 124 cm2 (b) 231 cm2 (c) 285 cm2 (d) 310 cm2
Refer to Poster II
3. What is the radius of circular region if length of poster is 18 cm and breadth is 14 cm?
(a) 14 cm (b) 18 cm (c) 7 cm (d) 9 cm
4. What is the area of circular region if length of poster is 18 cm and breadth is 14 cm?
(a) 154 cm2 (b) 172 cm2 (c) 196 cm2 (d) 216 cm2
Refer to Poster III
5. The area of poster if the length and breadth of it are 18 cm and 14 cm respectively, is
(a) 155 cm2 (b) 175 cm2 (c) 195 cm2 (d) 205 cm­2

Answers and Hints

A. Multiple Choice Questions (MCQs) 60° 22


1. (b) 22 cm 2. (b) 10 cm 3. (a) 30° = ×2× × 10.5 + 2 × 10.5
360° 7
4. (b) 169.56 cm2
= 11 + 21 = 32 cm
60°
Area of sector = × 3.14 × 18 × 18 6. (c) 22 cm
360°
= 169.56 cm2 θ
5. (a) 32 cm Length of arc = ( 2πr )
360°
Radius (r) = 10.5 cm 60°  22 
Angle (q) = 60° = 360°  2 × 7 × 21
θ
Perimeter = × 2πr + 2r = 22 cm
360°

Areas Related to Circles  141


D:\EG_Mathematics_Standard-10_(Term-1)_(30-07-2021)--\Open_Files\Ch-7\Ch-7
\ 31-Jul-2021   Anjum Azad  Proof-2 Reader’s Sign _______________________ Date __________

7. (d) 176 cm 1
Horse can graze in the field which is a = πr 2 − (20) 2
4
circle of radius 28 cm. 1
So, required perimeter = × 3.14 × 20 2 × 20 2 − 400
4
= 2pr = 2p (28) cm  3.14 
= 400  − 1 = 400 × (1.57 – 1)
22 2 
= 2 × × 28 cm = 176 cm
7 = 400 × 0.57 = 228 cm2
8. (d) 924 cm2 11. (a) 282 cm

Here, r = 21 cm, q = 120° Perimeter = length of major arc + 2r
270°
= × 2 × πr + 2r
360°
3 22
= × × 42 + 2 × 42
2 7
θ
Area of a sector = ×πr 2 = 198 + 84 = 282 cm
360°
120° 22 12. (d) 66 cm
= × ×21×21 Perimeter of shaded region
360° 7
= 462 cm2 = Perimeter of semicircles (ARC + APB
∴  Total area cleaned by two wipers + BSD + CQD)
= 2 × 462 = 924 cm2 = p[r1 + r2 + r3 + r4]
2
9. (b) 145.33 cm 22  7 7  22
=  7+ +7+  = × 21
Here, AB is diameter, 7  2 2 7
AC = 24 cm, BC = 10 cm and –ACB = 90° = 66 cm
 [Angle in a semicircle is 90°] 13. (c) 21 cm
\ AB2 = AC2 + BC2 Length of wire = Length of Arc
[By Pythagoras theorem] θ 22
⇒ 22 cm = ×2× ×r

⇒ AB = (24) 2 + (10) 2 cm 360° 7
60° 22
= 576 + 100 cm ⇒ 22 = ×2× ×r
360° 7
= 676 cm = 26 cm
22 × 360 × 7
AB ⇒ r = = 21 cm
⇒ OB = OA = = 13 cm 60 × 2 × 22
2
\  Area of shaded region = Area of semicircle Thus, the radius of the circle = 21 cm
 – Area of DACB 14. (b) 86.625 cm2
Area of shaded region
1 1  2
=  π(13) − × 24 × 10 cm = area of big semicircle – area of 2 small
2

2 2 
semicircles + area of middle semicircle
1  2
=  × 3.14 × 169 − 120 cm 1 1  3.5  1
2
 7
2
2  = π × (7 ) 2 − 2 × × π ×   + π ×  
 2   2
= [265.33 – 120] cm2 = 145.33 cm2 2 2 2
10. (c) 228 cm2
2
49  7 1  49   49 49 49 
= π−  π+   π =  − + π

OA = 20 cm 2  4  
2 4   2 16 8 
OB2 = OA2 + AB2 = 400 + 400 = 2 × 400  1 1 1 22  8 − 1 + 2 
=  − +  × 49 × =  × 7 × 22
fi OB = 20 2   fi  OB = r = 20 2  2 16 8  7  16 
\  Shaded area = Area of quadrant 9
– Area of square = × 7 × 22 = 86.625 cm 2 .
16

142  Mathematics–10
D:\EG_Mathematics_Standard-10_(Term-1)_(30-07-2021)--\Open_Files\Ch-7\Ch-7
\ 31-Jul-2021   Anjum Azad  Proof-2 Reader’s Sign _______________________ Date __________

Therefore, area of four shaded parts (i.e. X,


Y, W, Z) = (2 × 42) cm2 = 84 cm2
17. (c) 1910.85 cm
18. (b) 8.75 cm2

Here, l = 3.5 cm, r = 5 cm
Length of arc
2r r i
l =
360°
15. (c) 3500 cm2 2 × r ×5×i Q

Radius of circle = 35 cm
⇒ 3.5 =
360° r θ r
∠AOB = 90° ri
πr 2 θ ⇒ = 3.5
36
Area of sector OAB = 3.5 × 36
360° ⇒ q = l
22 90° 1925 2 r
= × 35 × 35 × = cm Now, Area of sector
7 360° 2
Area of minor segment rr 2 i r × 5 × 5 × 35 × 36
= =
360° 360° × r × 10
= area of sector OAB – area of DOAB
25 × 35 875
=
1925 1 1925 1225 = =
− × 35 × 35 = − 100 100
2 2 2 2 2
= 8.75 cm
700
= = 350 cm 2 ∴  Area of sector = 8.75 cm2
2 1 2
Area of major segment APB = area of circle 19. (c) πb cm
2
4
– area of minor segment
The diameter of circle that can be drawn
22 inside the rectangle is equal to the breadth of
= × 35 × 35 – 350 = 3500 cm2
7 rectangle.
The length of the rectangle = a cm
The breadth of the rectangle = b cm
∴ Diameter of circle = b cm
b
⇒ r = cm
2
∴ Area of circle A = pr2
2
 b 1 2
= π   = πb cm
2
 2 4

16. (a) 84 cm2 20. (c) 800 cm2
Area of the square Diagonal of a rhombus are perpendicular bisector
ABCD = 14 × 14 = 196 cm2 of each other.
1
Area of semicircle AOB = × pr2
2
1 22 2 14
= × × 7 × 7 = 77 cm ( r = )
2 7 2
2
Similarly area of semicircle DOC = 77 cm
Hence, the area of shaded region (Part W and
Part Y)
= Area of square – Area of two semicircles
AOB and COD
= 196 – 154 = 42 cm2
\  Each diagonal is diameter of the circle.

Areas Related to Circles  143


D:\EG_Mathematics_Standard-10_(Term-1)_(30-07-2021)--\Open_Files\Ch-7\Ch-7
\ 31-Jul-2021   Anjum Azad  Proof-2 Reader’s Sign _______________________ Date __________

Now, From figure,


area of circle = 1256 cm2 AB = 3 + 3 + r + 2r + r
1256 ⇒ 14 = 6 + 4r  ⇒ 4r = 14 – 6
fi pr2 = 1256  fi  r2 =

π ⇒ 4r = 8  ⇒  r = = 2 cm
8
1256 4

fi r2 = = 400 So, the area of shaded region
3.14
fi r = 20 cm = Area of square – Area of 4 semi circles
\  Diameter of the circle = 40 cm – Area of square (JKLM)
= Each diagonal of the rhombus One square One square Four
Area of rhombus ABCD JKLM semicircles
1 1 a1 = 14 cm a2 = 2r r = 2 cm
= (d1 × d2) = × 40 × 40  a2 = 2 × 2
2 2
2  a2 = 4 cm
= 800 cm
∴ Required area
21. (a)  25π − 25  cm 2 = a1 − 4 ×
2 πr 2
− a22
 4 2  2
22. (b) 31.4 cm 4×π×2×2
Perimeter of shaded region = length of arc = 14 × 14 − −4×4
2
PSR + length of arc PAQ + length of arc QTR
= 5p + 3.5p + 1.5p = 10p = 10 × 3.14 = 31.4 cm = 196 – 16 – 8p = (180 – 8p) cm2
23. (d)
63
π cm 2 Hence, the shaded area = (180 – 8p) cm2.
16 25. 36.39 cm2
24. (c) (180 – 8p) cm2.
Identification of shapes of figures: B. Assertion-Reason Type Questions
(i) 4 semi circles of radius r 1. (b) Both assertion (A) and reason (R) are true but
(ii) square ABCD of side 14 cm reason (R) is not the correct explanation of
(iii) square JKLM of side 2r assertion (A).
2. (b) Both assertion (A) and reason (R) are true but
reason (R) is not the correct explanation of
assertion (A).
Case Study Based Questions
I. 1. (b) 32 3 cm 2. (c) 48 cm
3. (b) 1764 cm2 4. (d) 154 cm2
2
5. (a) 378 cm
II. 1. (a) 22 cm 2. (b) 231 cm2
3. (c) 7 cm 4. (a) 154 cm2
2
5. (b) 175 cm

144  Mathematics–10
D:\EG_Mathematics_Standard-10_(Term-1)_(30-07-2021)--\Open_Files\Ch-7\Ch-7
\ 31-Jul-2021   Anjum Azad  Proof-2 Reader’s Sign _______________________ Date __________

Experts’ Opinion
Questions based on following types are very important for Exams. So, students are advised to
revise them thoroughly.
1. To find the area of sector of a circle.
2. To find the area of segment.
3. To find area of combination of plane figures.

IMPORTANT FORMULAE
•• Circumference of a circle = 2pr • Circumference of a semicircle = pr + 2r
2 1
•• Area of a circle = pr • Area of semicircle = pr2
2
1
•• Area of quadrant = pr2
4
•• Area of the ring = p (R2 – r2), R = outer radius and r = inner radius.
θ
•• Length of an arc of a sector of a circle with radius r and angle with degree measure q = × 2πr
360°
θ
•• Area of a sector of circle with radius r and angle with degrees measure q = × πr 2
360°
•• Area of the major sector = pr2 – Area of minor sector
•• Area of segment of a circle = Area of the corresponding sector – Area of the corresponding triangle
•• Area of major segment = pr2 – Area of minor segment

COMMON ERRORS
Errors Corrections
(i) Interpreting the concept of sector and segment (i) Make concept of both term clear by understanding
incorrectly. them.
(ii) Finding perimeter of sector, without taking the (ii) Draw the figure and note the data given to get
radius into consideration. the answer correctly.
(iii) Using formula for area of sector and length of (iii) Remember all the formulae and use them
arc of sector incorrectly. properly.
(iv) While finding the area of shaded region students (iv) Be careful in such problems. Do enough practice
forget to subtract the common area overlapped. to avoid the errors.

Areas Related to Circles  145


D:\EG_Mathematics_Standard-10_(Term-1)_(30-07-2021)--\Open_Files\Ch-7\Ch-7
\ 31-Jul-2021   Anjum Azad  Proof-2 Reader’s Sign _______________________ Date __________

QUICK REVISION NOTES


•• A circle is the locus of a point which moves in a plane in such a way that its distance from a fixed point
always remains the same. The fixed point is called as centre and constant distance is known as radius.
•• Area and perimeter of a circle with radius r
•• Circumference (perimeter) = 2pr unit or pd unit, where d (diameter) = 2r unit
πd 2
 Area = pr2 or (unit)2
4
πr 2 πd 2
 Area of semicircle = or (unit)2
2 8
πr 2
 Area of a of a quadrant = (unit)2
4
•• Area of a sector of circle with radius r and an angle q at the centre
θ
= pr2 × (unit)2
360°
OR
1 1
= × length of arc × radius = lr
2 2
•• Area of segment of a circle with radius r and angle with degree measure q
= Area of the sector – Area of the triangle
θ 1
= × πr 2 – r 2 × sin θ (unit)2
360° 2
•• Area enclosed by two concentric circles:
If R and r are radii of two concentric circles, then area enclosed by the two circles
= pR2 – pr2 = p (R2 – r2) = p (R – r) (R + r) (unit)2

146  Mathematics–10
D:\EG_Mathematics_Standard-10_(Term-1)_(30-07-2021)--\Open_Files\Ch-8\Ch-8
\ 31-Jul-2021    Amit  Proof-2 Reader’s Sign _______________________ Date __________

8 Probability

Topics Covered
1. Probability–A Theoretical Approach

1. Probability–A Theoretical Approach


In theoretical approach of probability, predictions about the happenings, on the basis of certain
assumptions, are made without actually performing the experiment.
Probability of an event E is defined as P(E) and is given by the formula:
Number of trials in which the event happened
P(E) =
Total number of trials
Example 1. A card is drawn from a deck of 52 cards. The event E is that the card is not an ace of hearts.
The number of outcomes favourable to E is
(a) 52 (b) 53 (c) 51 (d) 31
Solution. There is only one ace of hearts. Therefore, number of outcomes favourable to E = 52 – 1 = 51.
Hence, option (c) is the correct answer.
Example 2. A card is selected from a deck of 52 cards. The probability of it being a red face card is
5 7 3 5
(a) (b) (c) (d)
52 52 26 26
Solution. There are 6 red face cards in a deck of 52 cards.
\  No. of favourable outcomes = 6 out of 52 possible outcomes
No. of favourable outcomes 6 3
Thus, P(E) = = =
No. of possible outcomes 52 26
ence, option (c) is the correct answer.
H
Example 3. A card is drawn at random from a well-shuffled pack of 52 playing cards. The probability
of getting neither a red card nor a queen is
6 7 11 9
(a) (b) (c) (d)
13 13 13 13
Solution. Total number of possible outcomes = 52
Number of red cards = 26
Number of queens = 2
So, number of red cards and queens = 28
Number of cards which are neither red card nor queen = 52 – 28 = 24
24 6
\  P (getting neither a red card nor a queen) = =
52 13
Hence, option (a) is the correct answer.

147
D:\EG_Mathematics_Standard-10_(Term-1)_(30-07-2021)--\Open_Files\Ch-8\Ch-8
\ 31-Jul-2021    Amit  Proof-2 Reader’s Sign _______________________ Date __________

Example 4. Two dice are thrown at the same time and the product of numbers appearing on them is
noted. The probability that the product is a prime number is
1 1 1 5
(a) (b) (c) (d)
3 6 5 6
Solution. Now for the product of the numbers on the dice is prime number can be have in these possible
ways—(1, 2), (2, 1), (1, 3), (3, 1), (5, 1), (1, 5)
So, number of possible ways = 6
6 1
\ required probability = =
36 6
Hence, option (b) is the correct answer.
Example 5. Rahim tosses two different coins simultaneously. The probability of getting at least
one tail is
1 3 3 1
(a) (b) (c) (d)
4 4 5 6
Solution. Number of possible outcomes = 4 as possible outcomes are HH, HT, TH, TT.
Favourable outcomes for getting at least one tail are HT, TH, TT
No. of favourable outcomes = 3
3
\ P (getting at least one tail) =
4
Hence, option (b) is the correct answer.
Example 6. A jar contains 24 marbles, some are green and other are blue. If a marble is drawn at
2
random from the jar, the probability that it is green is . The number of blue marbles in the jar is
3
(a) 5 (b) 6 (c) 4 (d) 8
Solution. Let E be the probability of getting green marbles.
2
They, P(E) =
3
No. of green marbles 2 No. of green marbles
⇒ P(E) =   ⇒  =
No. of total marbles in jar 3 24
2 × 24
⇒ = No. of green marbles
3
⇒ Number of green marbles = 16
\ Number of blue marbles = 24 – 16 = 8
Hence, option (d) is the correct answer.
Example 7. A game consists of tossing a 10 rupee coin 3 times and noting its outcome each time. Sudhir
wins if all the tosses give the same result, i.e., three heads or three tails and loses otherwise. The probability
that Sudhir will not win the game is
1 1 3
(a) (b) (c) (d) 10
4 6 4
Solution. The possible outcomes are HHH, HHT, HTH, THH, TTH, THT, HTT, TTT
Number of possible outcomes = 8
Number of favourable outcomes when he will not win = 8 – 2 = 6
6 3
\ P (Sudhir will not win) = =
8 4
Hence, option (c) is the correct answer.

148  Mathematics–10
D:\EG_Mathematics_Standard-10_(Term-1)_(30-07-2021)--\Open_Files\Ch-8\Ch-8
\ 31-Jul-2021    Amit  Proof-2 Reader’s Sign _______________________ Date __________

Example 8. In the given figure, a disc is shown on which a player spins arrow
a
twice. The fraction is formed, where ‘a’ is the number of sector on which
b 2
arrow stops in the first spin and ‘b’ is the number of the sector in which the arrow 3
stops in the second spin. On each spin, each sector has equal chance of selection 1
a
by the arrow. The probability that the fraction > 1 is 4
b 6
1 5 5
(a) (b)
12 6
5
(c) (d) 1
12
a
Solution. For > 1, when a = 1, b cannot take any value.
b
a = 2, b can take 1 value
a = 3, b can take 2 values
a = 4, b can take 3 values
a = 5, b can take 4 values
a = 6, b can take 5 values
Thus, the favourable outcomes are:
(2,1), (3,1), (3,2), (4,1), (4,2), (4,3), (5,1), (5,2), (5,3), (5,4), (6,1), (6,2), (6,3), (6,4), (6,5)
Total no. of possible outcomes = 36
JK a NO 15 5
\ P KK > 1OO = or
L b P 36 12
Hence, option (c) is the correct answer.

Exercise 8.1
A. Multiple Choice Questions (MCQs)
Choose the correct answer from the given options:
1. One card is drawn from a well shuffled deck of 52 cards. The probability that it is black queen is
1 1 1 2
(a) (b) (c) (d)
26 13 52 13
2. The probability of an impossible event is
1
(a) 1 (b) (c) not defined (d) 0
2
3. If P(A) denotes the probability of an event A, then
(a) P(A) < 0 (b) P(A) > 1 (c) 0 ≤ P(A) ≤ 1 (d) –1 ≤ P(A) ≤ 1
4. If the probability of an event is p, the probability of its complementary event will be
1
(a) p – 1 (b) p (c) 1 – p (d) 1 −
p
5. Someone is asked to take a number from 1 to 100. The probability that it is a prime is
1 6 1 13
(a) (b) (c) (d)
5 25 4 15
6. A number is chosen at random from the numbers –5, –4, –3, –2, –1, 0, 1, 2, 3, 4, 5. Then the probability
that square of this number is less than or equal to 1 is
1 2 3 3
(a) (b) (c) (d)
11 11 11 26
7. If the probability of an event E happening is 0.023, then P(E) =
(a) 0.245 (b) 0.977 (c) 0.678 (d) 0.5

Probability  149
D:\EG_Mathematics_Standard-10_(Term-1)_(30-07-2021)--\Open_Files\Ch-8\Ch-8
\ 31-Jul-2021    Amit  Proof-2 Reader’s Sign _______________________ Date __________

8. A card is drawn at random from a well shuffled pack of 52 playing cards. The probability of getting
a red face card is
3 5 2 1
(a) (b) (c) (d)
26 26 13 26
9. A die is thrown once. What is the probability of getting a number greater than 4?
1 1 1 1
(a) (b) (c) (d)
2 3 4 5
Two players, Sangeeta and Reshma, play a tennis match. It is known that the probability of winning
10.
the match by Sangeeta is 0.62. What is the probability of winning the match by Reshma?
(a) 0.22 (b) 0.24 (c) 0.38 (d) 0.35
Cards marked with number 3, 4, 5, …, 50 are placed in a box and mixed thoroughly. A card is drawn
11.
at random from the box. The probability that the selected card bears a perfect square number is
1 1 1 1
(a) (b) (c) (d)
4 6 5 8
20 tickets, on which numbers 1 to 20 are written, are mixed thoroughly and then a ticket is drawn at
12.
random out of them. The probability that the number on the drawn ticket is a multiple of 3 or 7 is
1 2 3
(a) (b) (c) (d) 1
5 5 5
Two different dice are tossed together. The probability that the product of the two numbers on the top
13.
of the dice is 6 is
4 5 8 1
(a) (b) (c) (d)
9 9 9 9
A number is chosen at random from the numbers −3, −2, −1, 0, 1, 2, 3. What will be the probability
14.
that square of this number is less than or equal to 1?
3 4 5 6
(a) (b) (c) (d)
7 7 7 7
The probability of selecting a rotten apple randomly from a heap of 900 apples is 0·18. What is the
15.
number of rotten apples in the heap?
(a) 122 (b) 144 (c) 162 (d) 184
A letter of English alphabet is chosen at random. The probability that the chosen letter is a consonant is
16.
7 5 11 21
(a) (b) (c) (d)
26 26 26 26
A die is thrown once. What is the probability of getting a number less than 3?
17.
1 1 1 1
(a) (b) (c) (d)
2 3 5 9
If the probability of winning a game is 0.07, what is the probability of losing it?
18.
(a) 0.33 (b) 0.63 (c) 0.93 (d) 0.57
The probability of getting a doublet in a throw of a pair of dice is
19.
1 1 1 1
(a) (b) (c) (d)
2 4 5 6
The probability of getting a black queen when a card is drawn at random from a well-shuffled pack
20.
of 52 cards is
1 3 8
(a) (b) (c) (d) 1
26 26 13
Cards numbered 7 to 40 were put in a box. Poonam selects a card at random. What is the probability
21.
that Poonam selects a card which is a multiple of 7?
3 5 7 9
(a) (b) (c) (d)
28 34 34 34

150  Mathematics–10
D:\EG_Mathematics_Standard-10_(Term-1)_(30-07-2021)--\Open_Files\Ch-8\Ch-8
\ 31-Jul-2021    Amit  Proof-2 Reader’s Sign _______________________ Date __________

A game of chance consists of spinning an arrow which comes to rest pointing at one of the numbers
22.
1, 2, 3, 4, 5, 6, 7, 8 and these are equally likely outcomes. The probability that the arrow will point at
any factor of 8 is
1 1
(a) 0 (b) (c) (d) 1
2 4
A game consists of tossing a coin 3 times and noting the outcomes each time. If getting the same result
23.
in all the tosses is a success, the probability of losing the game is
3 1 3
(a) (b) (c) (d) 1
4 4 8
A ticket is drawn at random from a bag containing tickets numbered from 1 to 40. The probability
24.
that the selected ticket has a number which is a multiple of 5 is
1 3 4
(a) (b) (c) (d) 1
5 5 5
Three cards of spades are lost from a pack of 52 playing cards and the remaining cards are shuffled
25.
and then a card was drawn at random from them. The probability that the drawn card is of black colour
is
11 19 23 27
(a) (b) (c) (d)
52 52 52 52
The king, queen and jack of clubs are removed from a deck of 52 playing cards and the remaining
26.
cards are shuffled. A card is drawn from the remaining cards. The probability of getting a
card of queen is
1 2 3 5
(a) (b) (c) (d)
49 49 49 49
A coin is tossed two times. Find the probability of getting at least one head is
27.
1 3 1 3
(a) (b) (c) (d)
4 4 8 8
A ticket is drawn at random from a bag containing tickets numbered from 1 to 40. The probability
28.
that the selected ticket has a number which is a multiple of 10 is
7 3 1 9
(a) (b) (c) (d)
10 10 10 10
1000 tickets of a lottery were sold and there are 5 prizes on these tickets. If John has purchased one
29.
lottery ticket, what is the probability of winning a prize?
1 7 3 1
(a) (b) (c) (d)
100 200 200 200
p
The probability of guessing the correct answer to a certain test is
30. . If the probability of not guessing
1 12
the correct answer to this question is , then the value of p is
3
(a) 2 (b) 4 (c) 6 (d) 8
A lot of 25 bulbs contain 5 defective ones. One bulb is drawn at random from the lot. What is the
31.
probability that the bulb is good?
1 2 3 4
(a) (b) (c) (d)
5 5 5 5
If a number x is chosen at random from the numbers –3, –2, –1, 0, 1, 2, 3. What is probability that
32.
x2 ≤ 4?
5 3 4 6
(a) (b) (c) (d)
7 7 7 7

Probability  151
D:\EG_Mathematics_Standard-10_(Term-1)_(30-07-2021)--\Open_Files\Ch-8\Ch-8
\ 31-Jul-2021    Amit  Proof-2 Reader’s Sign _______________________ Date __________

33. A letter is selected at random from the set of English alphabets. What is the probability that it is a
vowel?
1 3 5 7
(a) (b) (c) (d)
26 26 26 26
34. A bag contains 15 white and some black balls. If the probability of drawing a black ball from the bag
is thrice that of drawing a white ball, the number of black balls in the bag is
(a) 15 (b) 30 (c) 30 (d) 45
35. The probability of selecting a red ball at random from a jar that contains only red, blue and orange
1 1
balls is . The probability of selecting a blue ball at random from the same jar is . The jar contains
4 3
10 orange balls, the total number of balls in the jar is
(a) 16 (b) 20 (c) 24 (d) 26
36. A bag contains, white, black and red balls only. A ball is drawn at random from the bag. If the probability
3 2
of getting a white ball is and that of a black ball is , then find the probability of getting a red ball.
10 5
If the bag contains 20 black balls, then total number of balls in the bag is
(a) 30 (b) 50 (c) 38 (d) 54
37. A number x is selected at random from the numbers 1, 4, 9, 16 and another number y is selected at
random from the numbers 1, 2, 3, 4. The probability that the value of xy is more than 16 is
(a) 3 (b) 5 (c) 7 (d) 1
8 8 8
B. Assertion-Reason Type Questions
In the following questions, a statement of assertion (A) is followed by a statement of reason (R).
Choose the correct choice as:
(a) Both assertion (A) and reason (R) are true and reason (R) is the correct explanation of assertion (A).
(b) Both assertion (A) and reason (R) are true but reason (R) is not the correct explanation of assertion (A).
(c) Assertion (A) is true but reason (R) is false.
(d) Assertion (A) is false but reason (R) is true.
5
1. Assertion (A): If a pair of dice is thrown once, then the probability of getting a sum of 8 is .
36
1
Reason (R): In a simultaneous toss of two coins, the probability of getting exactly one head is .
2
2. Assertion (A): The probability of a sure event is 1.
Reason (R): Let E be an event. Then 0 ≤ P (E) ≤ 1.

Case Study Based Questions


Garima has two children, Tapan and Maya. Every Sunday is a game
I.
night in the family. Tonight Garima has planned for a game with
three cubes, one purple and two yellow. She placed the three cubes
in a bag and called for her children.
Garima: Do you want to play a game of probability?
Maya: What is probability?
Garima: Let me ask you something before I answer you. Can you
predict what is in this bag?
Tapan: I cannot guess that!
Maya: I am 100% sure it is a toy!
Garima: I am glad you think that Maya. Just now you used the
concept of probability.

152  Mathematics–10
D:\EG_Mathematics_Standard-10_(Term-1)_(30-07-2021)--\Open_Files\Ch-8\Ch-8
\ 31-Jul-2021    Amit  Proof-2 Reader’s Sign _______________________ Date __________

Whether an event can happen or not, can’t be predicted with total certainty. But we can always predict
how likely or unlikely it is for an event to happen.
And for predicting that, we use a concept called probability.
Number of ways event can happen
Probability (an event to happen) =
Total number of ways all eveents can happen
Placing the bag of cubes in the centre, Garima explained the rules of the game to the children.
Garima: Without looking, the first player will pick out a cube from the bag and then the second player
will also pick out one cube without looking. If the two cubes picked out were the same colour, then
the first person will win the game. If the boxes picked out are of two differently coloured cubes, then
the second player will be the winner.
1. In the first round, Maya pulled out a cube, which was yellow. What is the probability that Tapan will
win the game?
1 1 2
(a) (b) (c) (d) 0
2 3 3
2. In the second round, Tapan started by picking out a purple cube. What is the probability for Tapan to
win the round?
1 2
(a) 1 (b) (c) 0 (d)
3 3
3. In the third round, Maya pulled out a cube. The probability that the pulled out cube is not of yellow
colour is
1 1 2
(a) 1 (b) (c) (d)
2 3 3
4. In the fourth round, Tapan pulled out a cube. The probability that the pulled out cube is either purple
or yellow is
1 1 2
(a) 1 (b) (c) (d)
2 3 3
5. In the last round, Maya pulled out a cube. The probability that the pulled out cube is of green colour
is
1 1
(a) 1 (b) (c) (d) 0
2 3
Rahul and Ravi planned to play Business (board game) in which they were supposed to use two dice.
II.

    
Ravi got first chance to roll the dice. What is the probability that he got the sum of the two numbers
1.
appearing on the top face of the dice is 8?
1 5 1
(a) (b) (c) (d) 0
26 36 18
Rahul got next chance. What is the probability that he got the sum of the two numbers appearing on
2.
the top face of the dice is 13?
5 1
(a) 1 (b) (c) (d) 0
36 18

Probability  153
D:\EG_Mathematics_Standard-10_(Term-1)_(30-07-2021)--\Open_Files\Ch-8\Ch-8
\ 31-Jul-2021    Amit  Proof-2 Reader’s Sign _______________________ Date __________

Now it was Ravi’s turn. He rolled the dice. What is the probability that he got the sum of the two
3.
numbers appearing on the top face of the dice is less than or equal to 12?
5 1
(a) 1 (b) (c) (d) 0
36 18
Rahul got next chance. What is the probability that he got the sum of the two numbers appearing on
4.
the top face of the dice is equal to 7?
5 5 1
(a) (b) (c) (d) 0
9 36 6
Now it was Ravi’s turn. He rolled the dice. What is the probability that he got the sum of the two
5.
numbers appearing on the top face of the dice is greater than 8?
5 1 5
(a) 1 (b) (c) (d)
36 18 18

Answers and Hints


1 2
1. (a) 2. (d) 0 12 (b)
26 5
3. (c) 0 ≤ P(A) ≤ 1 4. (c) 1 – p When one ticket is drawn, total possible cases
1 3 are 20.
5. (c) 6. (c)
4 11 Favourable cases when the number is a
7. (b) 0.977 multiple of 3 or 7 are 3, 6, 9, 12, 18, 7, 14,
3 i.e., 8 cases
8. (a)
26 Required probability
6 3
Required probability = = Number of favourable cases
52 26 =
1 Number of total possible cases
9. (b) 8 2
3 = =
Total number of outcomes = 6, 20 5
i.e.,{1, 2, 3, 4, 5, 6} 1
13. (d)
No. of fabourable outcomes = 2, i.e.{5, 6} 9
2 1 Total number of possible outcomes = 36
\  Required probability = =
6 3 A = Product of the numbers on the top of the
10. (c) 0.38 dice is 6.
P(winning the match by Sangeeta) \  Favourable outcomes (1,6), (2,3), (3,2),
+ P(winning the match by Reshma) = 1 (6,1), i.e. 4.
⇒  P(winning the match by Reshma)
Favourable outcomes
= 1 – 0.62 = 0.38 P(A) =
1 Total number of possible outcomes
11. (d) 4 1
8 = =
Total possible outcomes when one card is 36 9
drawn = 48 14. (a)
3
When the number on drawn card is a perfect 7
square, total favourable cases are 4, 9, 16, 25, Favourable outcomes are –1, 0, 1
36, 49, i.e. = 6 3
\  Required Probability =
P(perfect square number) 7
Number of total possible outcomes 15. (c) 162
= Let the number of rotten apples in the heap be n.
Number of favourable outcoomes
6 1 n
= = \ = 0.18  fi  n = 162
48 8 900

154  Mathematics–10
D:\EG_Mathematics_Standard-10_(Term-1)_(30-07-2021)--\Open_Files\Ch-8\Ch-8
\ 31-Jul-2021    Amit  Proof-2 Reader’s Sign _______________________ Date __________

21 3 3
16. (d) 26. (c) 27. (b)
26 49 4
Total number of possible outcomes = 26 1 1
No. of favourable outcomes = 21 28. (c) 29. (d)
10 200
21
\  Required probability = 30. (d) 8
26 p 1 p 1 2
17. (b)
1 + = 1  ⇒  = 1− = , \ p = 8
3 12 3 12 3 3
Total number of outcomes = 6, i.e., {1, 2, 3, 4
31. (d)
4, 5, 6} 5
Number of favourable outcomes Good bulbs = 25 – 5 = 20
20 4
= 2, i.e., {1, 2} P(good bulb) = =
25 5
2 1
\  Required probability = = 32. (a)
5
6 3 7
18. (c) 0.93 Total number of possible outcomes = 7
We know that for any event E, No. of favourable outcomes to x2 ≤ 4 are (–1)2
P(E) + P(E) = 1  ⇒  0.07 + P(E) = 1 = 1, 02 = 0, 12 = 1, (–2)2 = 4 and (2)2 = 4
⇒ P(E) = 1 – 0.07 = 0.93 \  Favourable outcomes = 5
1 1 5
19. (d) 20. (a) \  Required probability =
6 26 7
5
5 33. (c)
21. (b) 26
34 34. (d) 45
Number of cards = 40 – 6 = 34 Let the number of black balls in the bag
Multiples of 7 from 7 to 40 = 7, 14, 21, 28, 35 be n.
∴  Number of favourable outcomes = 5 \  Total number of balls are 15 + n
So, probability of getting a card with a Prob(Black ball) = 3 × Prob(White ball)
5
multiple of 7 = n 15
34 ⇒   = 3×   ⇒  n = 45
1 15 + n 15 + n
22. (b) 35. (c) 24
2
Let number of red balls in the jar = x
3
23. (a)  number of blue balls in the jar = y
4 \ Total number of balls in the jar = x + y + 10
Possible outcomes = {HHH, HHT, HTH,
1
THH, TTH, THT, HTT, TTT} Probability of selecting red ball =
Total possible outcomes = 8, 4
x 1
There are two cases of the same result, i.e. ⇒ =   ⇒ 4x = x + y + 10
{HHH, TTT} x + y +10 4
Number of outcomes in which the game is lost ⇒ 3x – y = 10 ...(i)
=8–2=6 1 y
Also =
∴  Probability of losing the game = =
6 3 3 x + y +10
8 4 ⇒ x + y + 10 = 3y
24. (a)
1 ⇒      x = 2y – 10 ...(ii)
5 From (i) and (ii), we have
25. (c)
23 3(2y – 10) – y = 10  ⇒ 6y – 30 – y = 10
52 ⇒ 5y = 40  ⇒  y = 8
Total number of possible outcomes = 52 – 3 Putting y = 8 in (ii), we get
= 49 x = 2 × 8 – 10 = 16 – 10 = 6
No. of favourable outcomes = 26 – 3 = 23 ⇒ x = 6
\  Required probability =
23 Hence, total number of balls in the jar
52 = 6 + 8 + 10 = 24

Probability  155
D:\EG_Mathematics_Standard-10_(Term-1)_(30-07-2021)--\Open_Files\Ch-8\Ch-8
\ 31-Jul-2021    Amit  Proof-2 Reader’s Sign _______________________ Date __________

36. (b) 50 3
Let R = getting a red ball 37. (a)
8
B = getting a black ball x can be 1, 4, 9 or 16 and y can be 1, 2, 3 or 4
W = getting a white ball Total number of cases of xy are 16.
Now, P(R) + P(B) + P(W) = 1 Number of cases when xy is more than 16 are
3 2 (9 × 2), (9 × 3), (9 × 4), (16 × 2), (16 × 3), (16 × 4),
  P(R ) + + = 1 i.e., 6 cases.
10 5
 3 2 6 3
⇒ P(R) = 1 −  +  P(value of xy more than 16) = = .
 10 5  16 8
= 1−
7
=
3 B. Assertion-Reason Type Questions
10 10 1. (b) Both assertion (A) and reason (R) are true
Let x and y be the white and red balls but reason (R) is not the correct explanation
respectively. of assertion (A).
x 3 2. (b) Both assertion (A) and reason (R) are true
Then, = but reason (R) is not the correct explanation
x + y + 20 10
of assertion (A).
⇒ 10x – 3x – 3y = 60
⇒ 7x – 3y = 60 ...(i) Case Study Based Questions
y 3 1 1
Again, = I. 1. (a) 2. (c) 0 3. (c)
x + y + 20 10 2 3
⇒ 10y – 3x – 3y = 60 4. (a) 1 5. (d)
⇒ 7y – 3x = 60 ...(ii) 5
II. 1. (b) 2. (d) 0 3. (a) 1
On solving eqn (i) and (ii), x = 15 and y = 15. 36
So, total number of balls in the bag 1 5
4. (c) 5. (d)
= 15 + 15 + 20 = 50 6 18

Experts’ Opinion
Questions based on following types are very important for Exams. So, students are advised to
revise them thoroughly.
1. Problems based on tossing a coin. 2. Problems based on throwing a die.
3. Problems based on playing cards. 4. Problems based on selecting of an object from bag/box.

IMPORTANT FORMULAE
No. of outcomes favourable to E
• P(E) =
No. of all possible outcomes of the expeeriment
• For any event E, P (E) + P (E) = 1, where E stands for ‘not E’.
COMMON ERRORS
Errors Corrections
(i) Interpreting incorrectly that all the experiments (i) An experiment whose outcome is known is not
are random experiment. a random experiment.
(ii) Writing incorrectly the possible outcomes when a (ii) Take care of the order of occurrence. Here,
coin is tossed one by one or two coins are tossed (HH, HT, TH, TT).
together, say (HH, HT, TT).
(iii) Using negative values and a number greater than (iii) The probability of an event lies between 0 and 1
one for probability. (both 0 and 1 inclusive). So, a negative value and a
number greater than 1 cannot be used for probability.

156  Mathematics–10
D:\EG_Mathematics_Standard-10_(Term-1)_(30-07-2021)--\Open_Files\Ch-8\Ch-8
\ 31-Jul-2021    Amit  Proof-2 Reader’s Sign _______________________ Date __________

QUICK REVISION NOTES


• An experiment which has a number of possible outcomes is known as a random experiment.
• The theoretical probability or classical probability of an event E, written as P(E) is defined as
No. of outcomes favourable to E
P(E) =
No. of all possible outcomess of the experiment
where, we assume that the outcomes of the experiment are equally likely.
• The probability of a sure event (or a certain event) is 1.
• The probability of an event E is a number P(E) such that 0 ≤ P(E) ≤ 1
• An event having only one outcome is called an elementary event. The sum of the probabilities of all the
elementary events of an experiment is 1.
• For any event E, P(E) + P(E) = 1, where E stands for ‘not E’. E and E are called complementary events.
• When each outcome of a random experiment is likely to occur as the other, then they are termed as
equally likely outcomes.
A deck of Playing Cards (52)
Red Cards (26) Black Cards (26)

Hearts (13) Diamonds (13) Spades (13) Clubs (13)

Ace (1 × 4) Number Cards (9 × 4) Face Cards (3 × 4)

2, 3, 4, 5, 6, 7, 8, 9, 10 Jack, Queen, king

Probability  157
D:\EG_Mathematics_Standard-10_(Term-1)_(30-07-2021)\Open_Files\Sample_Paper_1-3_(Stand)\Sample_Paper_1-3_(Stand)
\ 13-Sep-2021   Bhandari  Proof-4 Reader’s Sign _______________________ Date __________

Sample Paper - 1
(Issued by CBSE on 2nd September, 2021)

Time Allowed: 90 Minutes Maximum Marks: 40

General Instructions:
1. The question paper contains three parts A, B and C.
2. Section-A consists of 20 questions of 1 mark each. Any 16 questions are to be attempted.
3. Section-B consists of 20 questions of 1 mark each. Any 16 questions are to be attempted.
4. Section-C consists of 10 questions based on two Case Studies. Attempt any 8 questions.
5. There is no negative marking.

SECTION-A
Section-A consists of 20 questions of 1 mark each. Any 16 questions are to be attempted.
1. The ratio of LCM and HCF of the least composite and the least prime numbers is 1
(a) 1 : 2 (b) 2 : 1 (c) 1 : 1 (d) 1 : 3
2. The value of k for which the lines 5x + 7y = 3 and 15x + 21y = k coincide is 1
(a) 9 (b) 5 (c) 7 (d) 18
3. A girl walks 200 m towards East and then 150m towards North. The distance of the girl from the starting
point is 1
(a) 350 m (b) 250 m (c) 300 m (d) 225 m
4. The lengths of the diagonals of a rhombus are 24 cm and 32 cm, then the length of the altitude of the
rhombus is 1
(a) 12 cm (b) 12.8 cm (c) 19 cm (d) 19.2 cm
5. Two fair coins are tossed. What is the probability of getting at the most one head? 1

3 1 1 3
(a) (b) 4 (c) (d)
4 2 8
6. ΔABC ~ ΔPQR. If AM and PN are altitudes of ΔABC and ΔPQR respectively and AB2 :
PQ2 = 4 : 9, then AM : PN = 1
(a) 16 : 81 (b) 4 : 9 (c) 3 : 2 (d) 2 : 3
7. If 2 sin2 β – cos2 β = 2, then β is 1
(a) 0° (b) 90° (c) 45° (d) 30°
8. Prime factors of the denominator of a rational number with the decimal expansion 44.123 are 1
(a) 2, 3 (b) 2, 3, 5 (c) 2, 5 (d) 3, 5
9. The lines x = a and y = b, are 1
(a) intersecting (b) parallel (c) overlapping (d) None of these
10. The distance of point A (–5, 6) from the origin is 1
(a) 11 units (b) 61 units (c) 11 units (d) 61 units
23
11. If a2 = , then a is 1
25
(a) rational (b) irrational (c) whole number (d) integer

158
D:\EG_Mathematics_Standard-10_(Term-1)_(30-07-2021)\Open_Files\Sample_Paper_1-3_(Stand)\Sample_Paper_1-3_(Stand)
\ 13-Sep-2021   Bhandari  Proof-4 Reader’s Sign _______________________ Date __________

12. If LCM (x, 18) = 36 and HCF (x, 18) = 2, then x is 1
(a) 2 (b) 3 (c) 4 (d) 5
13. In ΔABC right angled at B, if tan A= 3 , then cos A cos C – sin A sin C = 1
3
(a) – 1 (b) 0 (c) 1 (d)
2
14. If the angles of ΔABC are in ratio 1 : 1 : 2, respectively (the largest angle being angle C), then the
sec A tan A
value of − is 1
cos ec B cot B
1 3
(a) 0 (b) (c) 1 (d)
2 2
15. The number of revolutions made by a circular wheel of radius 0.7 m in rolling a distance of 176 m is
1
(a) 22 (b) 24 (c) 75 (d) 40
16. ΔABC is such that AB = 3 cm, BC = 2cm, CA = 2.5 cm. If ΔABC ~ ΔDEF and EF = 4 cm, then
perimeter of ΔDEF is 1
(a) 7.5 cm (b) 15 cm (c) 22.5 cm (d) 30 cm
17. In the figure, if DE || BC, AD = 3 cm, BD = 4 cm and BC = 14 cm, then DE equals 1

(a) 7 cm (b) 6 cm (c) 4 cm (d) 3 cm


4 sin β − 3 cos β
18. If 4 tan β = 3, then =  1
4 sin β + 3 cos β
1 2 3
(a) 0 (b)
(c) (d)
3 3 4
19. One equation of a pair of dependent linear equations is – 5x + 7y = 2. The second equation can be 1
(a) 10x +14y +4 = 0 (b) –10x –14y + 4 = 0 (c) –10x + 14y + 4 = 0 (d) 10x – 14y = – 4
20. A letter of English alphabets is chosen at random. What is the probability that it is a letter of the word
‘MATHEMATICS’? 1
4 9 5 11
(a) (b) (c) (d)
13 26 13 26
SECTION-B
Section-B consists of 20 questions of 1 mark each. Any 16 questions are to be attempted.
21. If sum of two numbers is 1215 and their HCF is 81, then the possible number of pairs of such numbers
are 1
(a) 2 (b) 3 (c) 4 (d) 5

Sample Paper - 1  159


D:\EG_Mathematics_Standard-10_(Term-1)_(30-07-2021)\Open_Files\Sample_Paper_1-3_(Stand)\Sample_Paper_1-3_(Stand)
\ 13-Sep-2021   Bhandari  Proof-4 Reader’s Sign _______________________ Date __________

22.
Given below is the graph representing two linear equations
by lines AB and CD respectively. What is the area of the
triangle formed by these two lines and the line x = 0? 1
(a) 3 sq. units
(b) 4 sq. units
(c) 6 sq. units
(d) 8 sq. units
If tan α + cot α = 2, then tan20 α + cot20 α =
23. 1
(a) 0 (b) 2
(c) 20 (d) 220

24.
If 217x + 131y = 913, 131x + 217y = 827, then x + y is  1
(a) 5 (b) 6 (c) 7 (d) 8
25.
The LCM of two prime numbers p and q (p > q) is 221. The value of 3p – q is equal to 1
(a) 4 (b) 28 (c) 38 (d) 48
26.
A card is drawn from a well shuffled deck of cards. What is the probability that the card drawn is
neither a king nor a queen? 1
11 12 11 11
(a) (b) (c) (d)
13 13 26 52
27.
Two fair dice are rolled simultaneously. The probability that 5 will come up at least once is 1
5 11 12 23
(a) (b) (c) (d)
36 36 36 36
If 1 + sin2 α = 3 sin α cos α, then values of cot α are
28. 1
(a) –1, 1 (b) 0, 1 (c) 1, 2 (d) –1, – 1
29.
The vertices of a parallelogram in order are A (1, 2), B (4, y), C (x, 6) and D (3, 5). Then (x, y) is 1
(a) (6, 3) (b) (3, 6) (c) (5, 6) (d) (1, 4)
30.
In the given figure, ∠ACB = ∠CDA, AC = 8 cm, AD = 3 cm, then BD is 1

22 26 55 64
(a) cm (b) cm (c) cm (d) cm
3 3 3 3
31.
The equation of the perpendicular bisector of line segment joining points A (4, 5) and B (–2, 3) is 1
(a) 2x – y + 7 = 0 (b) 3x +2y – 7 = 0 (c) 3x – y – 7 = 0 (d) 3x + y – 7 = 0

160  Mathematics–10
D:\EG_Mathematics_Standard-10_(Term-1)_(30-07-2021)\Open_Files\Sample_Paper_1-3_(Stand)\Sample_Paper_1-3_(Stand)
\ 13-Sep-2021   Bhandari  Proof-4 Reader’s Sign _______________________ Date __________

cot y°
32.
In the given figure, D is the mid-point of BC, then the value of is 1
cot x°

1 1 1
(a) 2 (c) (d) (b)
2 3 4
1
33.
The smallest number by which should be multiplied so that its decimal expansion terminates after
two decimal places is 13 1
13 13 10 100
(a) (b) (c) (d)
100 10 13 13
34.
Sides AB and BE of a right triangle, right angled at B are of lengths 16 cm and 8 cm respectively.
The length of the side of largest square FDGB that can be inscribed in the triangle ABE is 1

32 16 8 4
(a) cm (b) cm (c) cm (d) cm
3 3 3 3
35.
Point P divides the line segment joining R (– 1, 3) and S (9, 8) in ratio k :1. If P lies on the line
x – y + 2 = 0, then value of k is 1
2 1 1 1
(a) (b) (c) (d)
3 2 3 4
36.
In the figure given below, ABCD is a square of side 14 cm with E, F, G and H as the mid points of
sides AB, BC, CD and DA respectively. The area of the shaded portion is 1

49π
(a) 44 cm² (b) 49 cm² (c) 98 cm² (d) cm²
2

Sample Paper - 1  161


D:\EG_Mathematics_Standard-10_(Term-1)_(30-07-2021)\Open_Files\Sample_Paper_1-3_(Stand)\Sample_Paper_1-3_(Stand)
\ 13-Sep-2021   Bhandari  Proof-4 Reader’s Sign _______________________ Date __________

37.
Given below is the picture of the Olympic rings made by taking five congruent circles of radius 1cm
each, intersecting in such a way that the chord formed by joining the point of intersection of two
circles is also of length 1cm. Total area of all the dotted regions assuming the thickness of the rings
to be negligible is 1

π 3 π 3 π 3 π 3
(a) 4  −  cm² (b)  −  cm² (c) 4  −  cm² (d) 8  −  cm²
 12 4  6 4  6 4  6 4 
1
If 2 and are the zeroes of px2 + 5x + r, then
38. 1
2
(a) p = r = 2 (b) p = r = – 2 (c) p = 2, r = – 2 (d) p = – 2, r = 2
39.
The circumference of a circle is 100 cm. The side of a square inscribed in the circle is 1
100 50 2 100 2
(a) 50 2 cm (b) cm (c) cm (d) cm
π π π
The number of solutions of 3x + y = 243 and 243x – y = 3 is
40. 1
(a) 0 (b) 1 (c) 2 (d) infinite
SECTION-C
Section-C consists of 10 questions of 1 mark each. Any 8 questions are to be attempted.
Q41 – Q45 are based on Case Study-1
Case Study-1
The figure given here shows the path of a diver, when she takes a jump from the diving board. Clearly it
is a parabola.
Annie was standing on a diving board, 48 feet above the water level. She took a dive into the pool. Her
height (in feet) above the water level at any time ‘t’ in seconds is given by the polynomial h(t) such that
h(t) = – 16t² + 8t + k

41.
What is the value of k? 48 1
(a) 0 (b) – 48 (c) 48 (d)
−16
42.
At what time will she touch the water in the pool? 1
(a) 30 seconds (b) 2 seconds (c) 1.5 seconds (d) 0.5 seconds

162  Mathematics–10
D:\EG_Mathematics_Standard-10_(Term-1)_(30-07-2021)\Open_Files\Sample_Paper_1-3_(Stand)\Sample_Paper_1-3_(Stand)
\ 13-Sep-2021   Bhandari  Proof-4 Reader’s Sign _______________________ Date __________

43.
Rita’s height (in feet) above the water level is given by another polynomial p(t) with zeroes – 1 and
2. Then p(t) is given by 1
(a) t² + t – 2 (b) t² + 2t – 1 (c) 24t² – 24t + 48 (d) – 24t² + 24t + 48
44.
A polynomial q(t) with sum of zeroes as 1 and the product as – 6 is modelling Anu’s height in feet
above the water at any time t (in seconds). Then q(t) is given by 1
(a) t² + t + 6 (b) t² + t – 6 (c) – 8t² + 8t + 48 (d) 8t² – 8t + 48
45.
The zeroes of the polynomial r(t) = – 12t² + (k – 3)t + 48 are negative of each other. Then k is 1
(a) 3 (b) 0 (c) – 1.5 (d) – 3
Q46 – Q50 are based on Case Study-2
Case Study-2
A hockey field is the playing surface for the game of hockey. Historically, the game was played on natural
turf (grass) but nowadays it is predominantly played on an artificial turf. It is rectangular in shape – 100
yards by 60 yards. Goals consist of two upright posts placed equidistant from the centre of the backline,
joined at the top by a horizontal crossbar. The inner edges of the posts must be 3.66 metres (4 yards) apart,
and the lower edge of the crossbar must be 2.14 metres (7 feet) above the ground.
Each team plays with 11 players on the field during the game including the goalie. Positions you might
play include:
• Forward: As shown by players A, B, C and D.
• Midfielders: As shown by players E, F and G.
• Fullbacks: As shown by players H, I and J.
• Goalie: As shown by player K.
Using the picture of a hockey field below, answer the questions that follow:

46. The coordinates of the centroid of ΔEHJ are


1
 2   −2  2   2 
(a)  − , 1 (b) 1,
  (c)  , 1 (d)  − , − 1
 3  3 3  3 

Sample Paper - 1  163


D:\EG_Mathematics_Standard-10_(Term-1)_(30-07-2021)\Open_Files\Sample_Paper_1-3_(Stand)\Sample_Paper_1-3_(Stand)
\ 13-Sep-2021   Bhandari  Proof-4 Reader’s Sign _______________________ Date __________

47. If a player P needs to be at equal distances from A and G, such that A, P and G are in straight line,

then position of P will be given by 1
 3   3   3
(a)  − , 2 (b)  2, − 2  (c)  2,  (d) (– 2, – 3)
2  2
48. The point on x-axis equidistant from I and E is 1
1   1  1   1
(a)  , 0 (b)  0, −  (c)  − , 0 (d)  0, 
2   2  2   2
49. What are the coordinates of the position of a player Q such that his distance from K is twice his distance
from E and K, Q and E are collinear? 1
(a) (1, 0) (b) (0, 1) (c) (– 2, 1) (d) (– 1, 0)
50. The point on y-axis equidistant from B and C is 1
(a) (– 1, 0) (b) (0, – 1) (c) (1, 0) (d) (0, 1)

164  Mathematics–10
D:\EG_Mathematics_Standard-10_(Term-1)_(30-07-2021)\Open_Files\Sample_Paper_1-3_(Stand)\Sample_Paper_1-3_(Stand)
\ 13-Sep-2021   Bhandari  Proof-4 Reader’s Sign _______________________ Date __________

Marking Scheme
1. (b) Least composite number is 4 and the least prime number is 2.
LCM (4, 2) : HCF (4, 2) = 4 : 2 = 2 : 1
a1 b1 c1
2. (a) For lines to coincide: = =
a2 b2 c2

5 7 −3
So, = =
15 21 − k
i.e. k = 9
3. (b) By Pythagoras theorem

(200 )
2
The required distance = 2
+ 150

= ( 4000 + 22500) = (62500) = 250 m.


So the distance of the girl from the starting point is 250 m.
1 1
4. (d) Area of the Rhombus = × d1 × d2 = × 24 × 32 = 384 cm²
2 2
Using Pythagoras theorem
2 2
1  1 
Side² =  d1  +  d 2  = 12² +16² = 144 + 256 = 400
2  2 

Side = 20 cm
Area of the Rhombus = base × altitude
384 = 20 × altitude
384
So altitude = = 19.2 cm
20
5. (a) Possible outcomes are (HH), (HT), (TH), (TT).
Favorable outcomes (at the most one head) are (HT), (TH), (TT).
3
So probability of getting at the most one head = .
4
6. (d) Ratio of altitudes = Ratio of sides for similar triangles
So, AM : PN = AB : PQ = 2 : 3
7. (b) 2 sin2 β – cos2 β = 2
Then, 2 sin2 β – (1 – sin2 β) = 2
3 sin2 β = 3  or  sin2 β =1
β is 90°
8. (c) Since it has a terminating decimal expansion, so prime factors of the denominator will be 2, 5.
9. (a) Lines x = a is a line parallel to y axis and y = b is a line parallel to x axis. So they will intersect.
10. (d) Distance of point A (– 5, 6) from the origin (0, 0) is
( 0 + 5 ) 2 + ( 0 − 6 )2 = 25 + 36 = 61 units

165
D:\EG_Mathematics_Standard-10_(Term-1)_(30-07-2021)\Open_Files\Sample_Paper_1-3_(Stand)\Sample_Paper_1-3_(Stand)
\ 13-Sep-2021   Bhandari  Proof-4 Reader’s Sign _______________________ Date __________

23
11. (b) a² = 23/25,  then  a = , which is irrational
5
12. (c) LCM × HCF = Product of two numbers
36 × 2 = 18 × x
x = 4
13. (b) tan A = 3 = tan 60°
So, ∠A = 60°,
Hence, ∠C = 30°.
 1  3  3  1
So cos A cos C – sin A sin C =   ×   –   ×   = 0
2  2   2   2
14. (a) 1x + 1x + 2x = 180°,  x = 45°
∠A , ∠B and ∠C are 45°, 45° and 90° respectively
sec A tan A sec 45° tan 45° 2 1
− − = −
cos ec B cot B = cos ec 45° cot 45° 2 1
=1–1=0

total distance 176


15. (d) Number of revolutions = = = 40
circumference 22
2× × 0.7
7
perimeter of ∆ABC BC
16. (b) =
perimeter of ∆DEF EF
7.5 2
=
perimeter of ∆DEF 4
So perimeter of ΔDEF = 15 cm
17. (b) Since DE || BC, ΔABC ~ ΔADE (By AA rule of similarity)
AD DE 3 DE
So = i.e. = . So DE = 6 cm
AB BC 7 14
18. (a) Dividing both numerator and denominator by cos β,
4 sin β − 3 cos β 4 tan β − 3 3 − 3
= = =0
4 sin β + 3 cos β 4 tan β + 3 3 + 3
19. (d) – 2(– 5x + 7y = 2) gives 10x – 14y = – 4.
a1 b1 c1
Now = = =–2
a2 b2 c2
20. (a) Number of Possible outcomes are 26
Favorable outcomes are M, A, T, H, E, I, C, S
8 4
Probability = =
26 13
21. (c) Since HCF = 81, two numbers can be taken as 81x and 81y,
ATQ 81x + 81y = 1215
or x + y = 15
which gives four co-prime pairs
1, 14
2, 13

166  Mathematics–10
D:\EG_Mathematics_Standard-10_(Term-1)_(30-07-2021)\Open_Files\Sample_Paper_1-3_(Stand)\Sample_Paper_1-3_(Stand)
\ 13-Sep-2021   Bhandari  Proof-4 Reader’s Sign _______________________ Date __________

4, 11
7, 8
1
22. (c) Required Area is area of triangle ACD = × 6 × 2 = 6 sq units
2
23. (b) tan α + cot α = 2 gives α = 45°. So tan α = cot α = 1
tan20 α + cot20 α = 120 + 120 = 1 + 1 = 2
24. (a) Adding the two given equations we get: 348x + 348y = 1740.
So, x + y = 5
25. (c) LCM of two prime numbers = product of the numbers
221 = 13 × 17.
So, p = 17 and q = 13
∴ 3p – q = 51 – 13 =38
26. (a) Probability that the card drawn is neither a king nor a queen
52 − 8
=
52
44 11
= =
52 13
27. (b) Outcomes when 5 will come up at least once are
(1, 5), (2, 5), (3, 5), (4, 5), (5, 5), (6, 5), (5, 1), (5, 2), (5, 3), (5, 4) and (5, 6)
11
Probability that 5 will come up at least once =
36
28. (c) 1+ sin2 α = 3 sin α cos α
sin2 α + cos2 α + sin2 α = 3 sin α cos α
2 sin2 α – 3 sin α cos α + cos2 α = 0
(2 sin α – cos α) (sin α – cos α) = 0
∴ cot α = 2  or  cot α = 1
29. (a) Since ABCD is a parallelogram, diagonals AC and BD bisect each other.
∴ mid point of AC = mid point of BD
 x + 1 6 + 2  3 + 4 5 + y
 ,  =  , 
2 2 2 2 
Comparing the co-ordinates, we get,
x +1 3+ 4
=  So, x = 6
2 2
6+2 5+ y
Similarly, =  So, y = 3
2 2
∴ (x, y) = (6, 3)
30. (c) ΔACD ~ Δ ABC (AA)
AC AD

∴ = (CPST)
AB AC
8 3
=
AB 8

Marking Scheme  167


D:\EG_Mathematics_Standard-10_(Term-1)_(30-07-2021)\Open_Files\Sample_Paper_1-3_(Stand)\Sample_Paper_1-3_(Stand)
\ 13-Sep-2021   Bhandari  Proof-4 Reader’s Sign _______________________ Date __________

64
This gives cm AB =
3
64 55
So, BD = AB – AD = −3 = cm
3 3
31. (d) Any point (x, y) of perpendicular bisector will be equidistant from A and B.


∴ ( x − 4 )2 + ( y − 5 ) 2 = ( x + 2)2 + ( y − 3)2
Solving we get – 12x – 4y + 28 = 0  or  3x + y – 7 = 0
AC
cot y° BC CD CD 1
32. (b) =
cot x° = AC = BC 2CD 2
=
CD
1
33. (a) The smallest number by which should be multiplied so that its decimal expansion terminates
13
13 1 13 1
after two decimal points is as × = = 0.01
100 13 100 100
34. (b) ΔABE is a right triangle & FDGB is a square of side x cm
ΔAFD ~ Δ DGE (AA-similarity)
AF FD

∴ = (CPST)
DG GE
16 − x x
=
x 8− x
16
128 = 24x or x =
cm
3
35. (a) Since P divides the line segment joining R (– 1, 3) and S (9, 8) in ratio k : 1
 1 3 
∴ coordinates of P are  9k − k + 1, 8k + k + 1  .

9k − 1 8k + 3
Since P lies on the line x – y + 2 = 0, then k + 1 − k + 1 + 2 = 0
9k – 1 – 8k – 3 + 2k + 2 = 0
2
which gives k = .
3
36. (c)

168  Mathematics–10
D:\EG_Mathematics_Standard-10_(Term-1)_(30-07-2021)\Open_Files\Sample_Paper_1-3_(Stand)\Sample_Paper_1-3_(Stand)
\ 13-Sep-2021   Bhandari  Proof-4 Reader’s Sign _______________________ Date __________

Shaded area = Area of semicircle + (Area of half square – Area of two quadrants)
= Area of semicircle + (Area of half square – Area of semicircle)
= Area of half square
1
= × 14 × 14 = 98 cm²
2
37. (d) Let O be the center of the circle. OA = OB = AB = 1 cm

So, ΔOAB is an equilateral triangle and ∴ ∠AOB = 60°


Required Area = 8 × Area of one segment with r = 1 cm, q = 60°
 60 3 2
= 8 ×  360 × π × 1 − 4 × 1 
2

 
π 3
= 8  6 − 4  cm²
 
1 5
38. (b) Sum of zeroes = 2 + =−
2 p
5 5
i.e.,
= − . So, p = – 2
2 p
1 r
Product of zeroes = 2 × =
2 p
r
i.e.
= 1  or  r = p = – 2
p
100
39. (c) 2πr = 100.  So, Diameter = 2r = = diagonal of the square.
π
100
side 2 = diagonal of square =
π
100 50 2

∴ side = =
2π π
40. (b) 3x + y = 243 = 35
So, x + y = 5 ...(1)
243x – y = 3
(35)x – y = 31
So, 5x – 5y = 1 ...(2)
a1 b1
Since, ≠ , so unique solution.
a2 b2
41. (c) Initially, at t = 0, Annie’s height is 48ft
So, at t = 0, h should be equal to 48

Marking Scheme  169


D:\EG_Mathematics_Standard-10_(Term-1)_(30-07-2021)\Open_Files\Sample_Paper_1-3_(Stand)\Sample_Paper_1-3_(Stand)
\ 13-Sep-2021   Bhandari  Proof-4 Reader’s Sign _______________________ Date __________

h(0) = – 16(0)² + 8(0) + k = 48


So, k = 48
42. (b) When Annie touches the pool, her height = 0 feet
i.e., – 16t² + 8t + 48 = 0 above water level
2t² – t – 6 = 0
2t² – 4t + 3t – 6 = 0
2t (t – 2) +3 (t – 2) = 0
(2t + 3) (t – 2) = 0
3
i.e.
t = 2  or  t = –
2
Since time cannot be negative, so t = 2 seconds
43. (d) t = – 1 and t = 2 are the two zeroes of the polynomial p(t)
Then, p(t) = k {t – (–1)} (t – 2)
= k(t + 1) (t – 2)
When t = 0 (initially) h1 = 48 ft
p(0) = k (0² – 0 – 2) = 48
i.e. – 2k = 48
So the polynomial is – 24 (t² – t – 2) = – 24t² + 24t + 48
44. (c) A polynomial q(t) with sum of zeroes as 1 and the product as – 6 is given by
q(t) = k [t² – (sum of zeroes)t + product of zeroes]
= k [t² – 1t + (–6)] ...(1)
When t = 0 (initially) q(0) = 48 ft
q(0) = k (0² – 1(0) – 6) = 48
i.e. – 6k = 48  or  k = – 8
Putting k = – 8 in equation (1), reqd. polynomial is – 8 [t² – 1t + (–6)]
= – 8t² + 8t + 48
45. (a) When the zeroes are negative of each other,
sum of the zeroes = 0
−b
So, = 0
a
−(k − 3)
= 0
−12
k −3
= 0
12
k – 3 = 0,
i.e. k = 3
46. (a) Centroid of ΔEHJ with E (2, 1), H (– 2, 4) and J (– 2, – 2) is

 2 + ( − 2) + ( − 2) 1 + 4 + ( − 2)   2 

,  =  − , 1

3 3 3

170  Mathematics–10
D:\EG_Mathematics_Standard-10_(Term-1)_(30-07-2021)\Open_Files\Sample_Paper_1-3_(Stand)\Sample_Paper_1-3_(Stand)
\ 13-Sep-2021   Bhandari  Proof-4 Reader’s Sign _______________________ Date __________

47. (c) If P needs to be at equal distance from A (3, 6) and G (1, – 3), such that A, P and G are collinear,
then P will be the mid-point of AG.
 3 + 1 6 + ( − 3)   3
So coordinates of P will be  2 ,  =  2, 2 
2
48. (a) Let the point on x-axis equidistant from I (– 1, 1) and E (2, 1) be (x, 0)
Then ( x + 1)2 + (0 − 1)2 = ( x − 2)2 + (0 − 1)2
x2 + 1 + 2x +1 = x2 + 4 – 4x + 1
6x = 3
1
So, x = .
2
1 

∴ the required point is  , 0
2 
49. (b) Let the coordinates of the position of a player Q such that his distance from K(– 4, 1) is twice his
distance from E(2, 1) be Q(x, y)
Then KQ : QE = 2 : 1
 2 × 2 + 1 × − 4 2 × 1 + 1 × 1
Q(x, y) =  ,  = (0, 1)
3 3
50. (d) Let the point on y-axis equidistant from B(4, 3) and C(4, – 1) be (0, y)

Then ( 4 − 0 ) 2 + ( 3 − y )2 = ( 4 − 0)2 + ( y + 1)2


16 + y2 + 9 – 6y = 16 + y2 + 1 + 2y
– 8y = – 8
So, y = 1.

∴ the required point is (0, 1)

Marking Scheme  171


D:\EG_Mathematics_Standard-10_(Term-1)_(30-07-2021)\Open_Files\Sample_Paper_1-3_(Stand)\Sample_Paper_1-3_(Stand)
\ 13-Sep-2021   Bhandari  Proof-4 Reader’s Sign _______________________ Date __________

Sample Paper - 2
Time Allowed: 90 Minutes Maximum Marks: 40
General Instructions: Same as Sample Paper 1

SECTION-A
Section-A consists of 20 questions of 1 mark each. Any 16 questions are to be attempted.
1. The least number which is a perfect square and is divisible by each of 16, 20 and 24 is  1
(a) 240 (b) 1600 (c) 2400 (d) 3600
2. The value of x, for which the polynomial x2 – 1 and x2 – 2x + 1 vanish simultaneously, is  1
(a) 2 (b) –2 (c) –1 (d) 1
1 1
3. If the points (a, 0), (0, b) and (1, 1) are collinear, then + equals  1
a b
(a) 1 (b) 2 (c) 0 (d) –1
4. If x = p sec θ and y = q tan θ, then  1
1
(a) x2 – y2 = p2 – q2 (b) x2q2 – y2p2 = pq (c) x2q2 – y2p2 = (d) x2q2 – y2p2 = p2q2
p2q2
a sin θ − b cos θ
5. If b tan θ = a, the value of
is 1
a sin θ + b cos θ
a+b a 2 + b2 a 2 − b2
(a) a − b (b) 2 (c) (d)
a 2 + b2 a + b2 a 2 − b2 a 2 + b2

6. (cos4 A – sin4 A) is equal to 1


(a) 1 – 2 cos2 A (b) 2 sin2 A – 1 (c) sin2 A – cos2 A (d) 2 cos2 A – 1
7. The probability of getting a number greater than 2 in throwing a die is 1
2 1 4 1
(a) (b) (c) (d)
3 3 3 4
8. Out of one-digit prime numbers, one number is selected at random. The probability of selecting an
even number is 1
1 1 4 2
(a) (b) (c) (d)
2 4 9 5
9. A boat travels with a speed of 15 km/h in still water. In a river flowing at 5 km/hr, the boat travels some
distance downstream and then returns. The ratio of average speed to the speed in still water is  1
(a) 8 : 3 (b) 3 : 8 (c) 8 : 9 (d) 9 : 8
10. The point on the x-axis which if equidistant from the points A(– 2, 3) and B(5, 4) is 1
(a) (0, 2) (b) (2, 0) (c) (3, 0) (d) (– 2, 0)
11. If sin θ + sin2 θ + sin3 θ = 1, then cos6 θ – 4 cos4 θ + 8 cos2 θ is equal to 1
(a) 1 (b) 2 (c) 3 (d) 4
12. A race track is in the form of a ring whose inner and outer circumference are 437 m and 503 m respectively.
The area of the track is 1
(a) 66 sq. cm (b) 4935 sq. cm (c) 9870 sq. cm (d) None of these

172
D:\EG_Mathematics_Standard-10_(Term-1)_(30-07-2021)\Open_Files\Sample_Paper_1-3_(Stand)\Sample_Paper_1-3_(Stand)
\ 13-Sep-2021   Bhandari  Proof-4 Reader’s Sign _______________________ Date __________

125
13. After how many places, the decimal form of will terminate? 1
24 .53
(a) Three places (b) Four places (c) Two places (d) None of these
14. If the graph of a polynomial intersects the X-axis at only one point, it can be a  1
(a) linear (b) quadratic (c) cubic (d) None of these
15. The number of common solutions for the system of linear equations 1

5x + 4y + 6 = 0 and 10x + 8y = 12 is
(a) 0 (b) 1 (c) 2 (d) None of these
16. If ∆ ABC ∼ ∆ PQR, AB = 6.5 cm, PQ = 10.4 cm and perimeter of ∆ ABC = 60 cm, the perimeter of
∆ PQR is  1
(a) 65 cm (b) 96 cm (c) 60 cm (d) 104 cm
17. In the given figure, O is the intersecting point of OA and OC and OABC is a square
of side 4 units, then the position of A, B and C is  1
(a) (4, 0) (4, 4) (0, 4) (b) (4, 0) (0, 4) (4, 4)
(c) (0, 4) (4, 4) (4, 0) (d) None of these
18. If x cos A = 8 and 15 cosec A = 8 sec A, then the value of x is 1

(a) 20 (b) 16 (c) 17 (d) 13


19. If a pair of linear equations is consistent, then the lines will be 1
(a) parallel (b) always coincident
(c) intersecting or coincident (d) always intersecting
20. The radius of the circles whose centre is at (0, 0) and which passes through the points (– 6, 8) is 1
(a) 10 units (b) 11 units (c) 9 units (d) 8 units
SECTION-B
Section-B consists of 20 questions of 1 mark each. Any 16 questions are to be attempted.
4 1 − sin A
21. If sin A = , then the value of is 1
5 1 + cos A
1 1 3
(a) (b) (c) 2 (d)
8 2 5
22. A coin is tossed twice. The probability of getting both heads is  1

1 1 1
(a) (b) (c) (d) 1
2 3 4
23. The quadratic polynomial, whose zeroes are 3 + 2 and 3 − 2 , is 1
(a) x2 – 3x + 5 (b) x2 – 6x + 7 (c) x2 – 7x + 6 (d) x2 – 8x + 12
24. If the area of a circle is 2464 cm2, then the diameter of circle is 1
(a) 57 cm (b) 56 cm (c) 54 cm (d) None of these

Sample Paper - 2  173


D:\EG_Mathematics_Standard-10_(Term-1)_(30-07-2021)\Open_Files\Sample_Paper_1-3_(Stand)\Sample_Paper_1-3_(Stand)
\ 13-Sep-2021   Bhandari  Proof-4 Reader’s Sign _______________________ Date __________

25. The coordinates of the point which is equidistant from the three vertices of the ∆AOB as shown in the
figure is

 x y  y x
(a) (x, y) (b) (y, x) (c)  ,  (d)  , 
 2 2  2 2

26. In the given figure, DE || BC, If AD = 3 cm, DB = 4 cm and AE = 6 cm, then EC is 1

(a) 8 cm (b) 12 cm (c) 6 cm (d) 4 cm


27. The number of polynomials having zeroes as – 2 and 5 is 1
(a) 1 (b) 2 (c) 3 (d) more than 3
28. The perimeter of a triangle with vertices (0, 4), (0, 0) and (3, 0) is 1

(a) 5 units (b) 12 units (c) 11 units (d) (7 + 5) units


29. Which of the following pair of equations are inconsistent? 1
y
(a) 3x–y = 9, x − = 3 (b) 4x + 3y = 24, – 2x + 3y = 6
3
(c) 5x – y = 10, 10x – 2y = 20 (d) – 2x + y = 3, – 4x + 2y = 10
30. A coin and a die is tossed simultaneously. The probability of the event that ‘tail’ and a prime number
turns up 1
1 1 1 2
(a) (b) (c) (d)
2 4 3 3
31. A car has two wipers which do not overlap each other. Each wiper has a blade of length r cm sweeping
through an angle of 60°. The area cleaned at each sweep of blades is 1
1 2 1 2
(a) πr cm 2 (b) πr cm 2 (c) πr 2 cm 2 (d) πr 2 cm 2
3 3 3 3
32. The pair of equations x + 2y + 5 = 0 and –3x – 6y + 1 = 0 has  1
(a) a unique solution (b) exactly two solutions
(c) infinitely many solutions (d) no solution

174  Mathematics–10
D:\EG_Mathematics_Standard-10_(Term-1)_(30-07-2021)\Open_Files\Sample_Paper_1-3_(Stand)\Sample_Paper_1-3_(Stand)
\ 13-Sep-2021   Bhandari  Proof-4 Reader’s Sign _______________________ Date __________

33. The length of minute hand of a clock is 14 cm. Then, the area swept by the minute hand in one minute
 22 
is  take π =  1
 7 
(a) 28 cm2 (b) 10.27 cm2 (c) 14.7 cm2 (d) 12.28 cm2
34. If 2x + 3y = 5 and 3x + 2y = 10, then x – y =  1
(a) 3 (b) 4 (c) 5 (d) 6
35. If (a, b) is the mid-point of the line segment joining the points A(10, –6) and B (k, 4) and a – 2b = 18,
then the value of k is 1
(a) 30 (b) 22 (c) 4 (d) 40
36. If cos A + cos2 A = 1, then the value of sin2 A + sin4 A is 1
(a) 0 (b) 1 (c) –1 (d) 2
37. The value of k for which 3 is a zero of polynomial 2x2 + x + k is  1
(a) 21 (b) 20 (c) –21 (d) 18
38. The ratio in which the point P (m, 6) divides the line segment joining A (–4, 3) and B (2, 8) is  1
(a) 2 : 3 (b) 1 : 2 (c) 3 : 2 (d) 2 : 1
4
39. A fraction becomes , if 2 is added to both numerator and denominator, if however 4 is subtracted
5 1
from both numerator and denominator, then the fraction becomes . The algebraical representation on
situation is 2 1
(a) 5x – 4y + 2 = 0, x – y = 0 (b) 5x – 4y + 2 = 0, 2x – y – 4 = 0
(c) x – 4y = 0, y + 2x = 0 (d) None of these
40. Which of the following is not the graph of a quadratic polynomial? 1

(a) (b) (c) (d)

SECTION-C
Section-C consists of 10 questions of 1 mark each. Any 8 questions are to be attempted.
Q41 – Q45 are based on Case Study-1
Case Study-1
Indian Army Day: The Army Day is celebrated on 15th January every year in India. The day is celebrated in
the form of parades and other military shows in the national capital New Delhi as well as in all headquarters.

Sample Paper - 2  175


D:\EG_Mathematics_Standard-10_(Term-1)_(30-07-2021)\Open_Files\Sample_Paper_1-3_(Stand)\Sample_Paper_1-3_(Stand)
\ 13-Sep-2021   Bhandari  Proof-4 Reader’s Sign _______________________ Date __________

Parade I: An Army contingent of 616 members is to march behind an army band of 32 members in

parade. The two groups are to march in the same number of columns.
Parade II: An Army contingent of 1000 members is to march behind an army band of 56 members

in parade. The two groups are to march in the same number of columns.
Refer to Parade I
41.
The expression of 616 as a product of its prime numbers is 1
(a) 22 × 141 × 111 (b) 23 × 71 × 111 (c) 24 × 71 × 111 (d) None of these.
42.
The LCM of 32 and 616 is 1
(a) 1248 (b) 1836 (c) 2464 (d) 3016
43.
What is the maximum number of columns in which they can march? 1
(a) 6 (b) 8 (c) 10 (d) 12
Refer to Parade II
44.
The HCF of numbers 1000 and 56 is 1
(a) 2 (b) 4 (c) 6 (d) 8
45.
Which of the following is the correct expression of the number 1000 as a product of its prime numbers?
 1
(a) 23 × 53 (b) 103 (c) 8 × 125 (d) 8

Q46 – Q50 are based on Case Study-2


Case Study-2
A Frame House: A frame-house is a house constructed from a wooden skeleton, typically covered with
timber board. The concept of similar triangles is used to construct it. Look at the following picture:
A
D
P

B C

Q R

House (i) House (ii)

46.
The front view of house (i) is shown below in which point P on AB is joined with point Q on AC.
If PQ || BC, AP = x m, PB = 10 m, AQ = (x – 2) m, QC = 6 m, then the value of x is 1
(a) 3 m (b) 4 m (c) 5 m (d) 8 m

176  Mathematics–10
D:\EG_Mathematics_Standard-10_(Term-1)_(30-07-2021)\Open_Files\Sample_Paper_1-3_(Stand)\Sample_Paper_1-3_(Stand)
\ 13-Sep-2021   Bhandari  Proof-4 Reader’s Sign _______________________ Date __________

47.
The side view of house (i) is shown below in which point F on AC is joined with point G on DE.

If ACED is a trapezium with AD || CE, F and G are points on non-parallel sides AC and DE respectively
AF
such that FG is parallel to AD, then = 1
FC
DG AD AF DG
(a) (b) (c) (d)
GE CE GE FC
48.
The front view of house (ii) is shown below in which point S on PQ is joined with point T on PR.

PS PT

If = and ∠PST = 70°, ∠QPR = 50°, then angle ∠QRP =  1
QS TR
(a) 70° (b) 50° (c) 80° (d) 60°
49.
Again consider the front view of house (ii). If S and T are points on side
PQ and PR respectively such that ST || QR and PS : SQ = 3 : 1. Also
TP = 6.6 m, then PR is 1
(a) 6.9 m
(b) 8.8 m
(c) 10.5 m
(d) 9.4 m
50.
Sneha has also a frame house whose front view is shown below.

If MN || AB, BC = 7.5 m, AM = 4 m and MC = 2 m, then length
of BN is 1
(a) 5 m
(b) 4 m
(c) 8 m
(d) 9 m

Sample Paper - 2  177


D:\EG_Mathematics_Standard-10_(Term-1)_(30-07-2021)\Open_Files\Sample_Paper_1-3_(Stand)\Sample_Paper_1-3_(Stand)
\ 13-Sep-2021   Bhandari  Proof-4 Reader’s Sign _______________________ Date __________

Answers
1. (d) 2. (d) 3. (d) 4. (d) 5. (d)
6. (d) 7. (a) 8. (b) 9. (c) 10. (b)
11. (d) 12. (b) 13. (b) 14. (a) 15. (a)
16. (b) 17. (a) 18. (c) 19. (c) 20. (a)
21. (a) 22. (c) 23. (b) 24. (b) 25. (a)
26. (a) 27. (d) 28. (b) 29. (d) 30. (b)
31. (c) 32. (d) 33. (b) 34. (c) 35. (b)
36. (b) 37. (c) 38. (c) 39. (b) 40. (d)
41. (b) 42. (c) 43. (b) 44. (d) 45. (a)
46. (c) 47. (a) 48. (d) 49. (b) 50. (a)

178  Mathematics–10
D:\EG_Mathematics_Standard-10_(Term-1)_(30-07-2021)\Open_Files\Sample_Paper_1-3_(Stand)\Sample_Paper_1-3_(Stand)
\ 13-Sep-2021   Bhandari  Proof-4 Reader’s Sign _______________________ Date __________

Sample Paper - 3
Time: Allowed: 90 Minutes Maximum Marks: 40
General Instructions: Same as Sample Paper 1

SECTION-A
Section-A consists of 20 questions of 1 mark each. Any 16 questions are to be attempted.
1. If a = 23 × 3, b = 2 × 3 × 5, c = 3n × 5 and LCM (a, b, c) = 23 × 32 × 5, then n =  1
(a) 1 (b) 2 (c) 3 (d) 4
p
2. The rational number of the form , q ≠ 0 , p and q are positive integers, which represents 0.134
q
i.e., (0.1343434 .......) is  1
134 134 133 133
(a) (b) (c) (d)
999 990 999 990
1 1
3. If a and b are zeroes and the quadratic polynomial f(x) = x2 – x – 4, then the value of + − αβ is 1
α β
15 −15
(a) (b) (c) 4 (d) 15
4 4
4. If sec 5A = cosec (A + 30°), where 5A is an acute angle, then the value of A is  1
(a) 15° (b) 5° (c) 20° (d) 10°
5. If f(x) = cos2 x + sec2 x, then f(x)  1
(a) ≥ 1 (b) ≤ 1 (c) ≥ 2 (d) ≤ 2
6. If tan 2A = cot (A – 18°), where 2A is an acute angle, then the value of A is 1
(a) 12° (b) 18° (c) 36° (d) 48°
7. Two coins are tossed simultaneously. The probability of getting at most one head is 1
1 1 3
(a) (b) (c) (d) 1
4 2 4
8. A three-digit number is to be formed using the digits 3, 4, 7, 8 and 2 without repetition. The probability
that it is an odd number is 1
2 1 4 3
(a) (b) (c) (d)
5 5 5 5
9. If X = 28 + (1 × 2 × 3 × 4 × .... × 16 × 28) and Y = 17 + (1 × 2 × 3 × ... × 17), then which of the following
is/are true?  1
(i) X is a composite number (ii) Y is a prime number
(iii) X – Y is a prime number (iv) X + Y is a composite number
(a) Both (i) and (ii) (b) Both (ii) and (iii) (c) Both (ii) and (iv) (d) Both (i) and (ii)

179
D:\EG_Mathematics_Standard-10_(Term-1)_(30-07-2021)\Open_Files\Sample_Paper_1-3_(Stand)\Sample_Paper_1-3_(Stand)
\ 13-Sep-2021   Bhandari  Proof-4 Reader’s Sign _______________________ Date __________

10. It is given that DABC ∼ DEDF such that AB = 5 cm, AC = 7 cm, DF = 15 cm and DE = 12 cm, then the
sum of the remaining sides of the triangles is 1
(a) 23.05 cm (b) 16.8 cm (c) 6.25 cm (d) 24 cm
a sin φ b sin θ a
11. If tan q = and tan φ = , then = 1
1 − a cos φ 1 − b cos θ b
sin θ sin θ sin φ sin θ
(a) (b) (c) (d)
1 − cos θ 1 − cos φ sin θ sin φ
12. Area of the largest triangle that can be inscribed in a semi-circle of radius r units is: 1
1 2
(a) r2 sq. units (b) r sq. units (c) 2r2 sq. units (d) 2
2 r sq. units
2
13. If the HCF of 65 and 117 is 13, LCM of 65 and 117 is 45 × a, then the value of a is 1
(a) 9 (b) 11 (c) 13 (d) 17

14. The coordinates of the point P as shown in the diagram will be 1

(a) (2, –3) (b) (–3, 2)


(c) (2, 3) (d) (3, 2)

PS 3
15. In ∆ PQR, ST||QR, = and PR = 28 cm, then the value of PT is  1
SQ 5

(a) 9.5 cm (b) 9 cm (c) 10 cm (d) 10.5 cm

16. If 2 is a zero of polynomial f (x) = ax2 – 3(a – 1) x –1, then the value of a is 1
5 1
(a) 0 (b) 2 (c) (d)
2 2
17. Which of the following cannot be the probability of an event? 1
3
(a) 1.5 (b) (c) 25% (d) 0.3
5
18. The line joining the mid-points of two sides of a triangle is 1
(a) bisector of the third side (b) perpendicular to the third side
(c) parallel to the third side (d) None of these
19. The probability that an ordinary year contains 53 Sundays is 1
2 1 7 7
(a) (b) (c) (d)
7 7 53 52
20. P is the LCM of 2, 4, 6, 8, 10; Q is the LCM of 1, 3, 5, 7, 9 and L is the LCM of P and Q. Then, which
of the following is true? 1
(a) L = 21 P (b) L = 40 Q (c) L = 63 P (d) L = 16 P

180  Mathematics–10
D:\EG_Mathematics_Standard-10_(Term-1)_(30-07-2021)\Open_Files\Sample_Paper_1-3_(Stand)\Sample_Paper_1-3_(Stand)
\ 13-Sep-2021   Bhandari  Proof-4 Reader’s Sign _______________________ Date __________

SECTION-B
Section-B consists of 20 questions of 1 mark each. Any 16 questions are to be attempted.
21. The value of a for which the lines x = 1, y = 2 and a2x + 2y – 20 = 0 are concurrent, is 1

(a) 1 (b) 8 (c) –4 (d) –3

22. The perimeter of a square circumscribing a circle of radius a cm is equal to ....... cm. 1
a
(a) 8a (b) 4a (c) 2a (d)
2
23. If the point P (x, y) is equidistant from the points A (5, 1) and B (1, 5), then 1
(a) y = 3x (b) x = y (c) x = –8y (d) –8x = y

24. In the given figure, ∠ABC = 90° and BD ⊥ AC. If BD = 8 cm and AD = 4 cm,
then the value of CD is 1
(a) 8 cm (b) 12 cm
(c) 14 cm (d) 16 cm

25. If HCF (a, b) = 12 and a × b = 1800, then LCM (a, b) = 1


(a) 3600 (b) 900 (c) 150 (d) 90

26. The area of a sector of a circle with radius 6 cm, if angle of the sector 60° is  1
132 2 36 2 132 2
(a) cm (b) cm (c) cm (d) None of these
14 7 7
27. The zeroes of the quadratic polynomial (x2 + 5x + 6) are 1
(a) –2 and –3 (b) 3 and 4 (c) 3 and 2 (d) 2 and 1

4
28. If tan θ − = 3, then sin2 θ is 1
tan θ
4 3 16 5
(a) (b) (c) (d)
17 17 17 17
29. A vertical stick 20 m long casts a shadow 10 m long on the ground. At the same time, a tower casts a
shadow 50 m long on the ground. The height of the tower is 1
(a) 100 m (b) 120 m (c) 25 m (d) 200 m
30. The graph of a quadratic polynomial is  1
(a) straight line (b) parabola (c) hyperbola (d) None of these
31. ∆ ABC ∼ ∆ DEF and the perimeters of ∆ ABC and ∆ DEF are 30 cm and 18 cm respectively. If BC
= 9 cm, then EF =  1
(a) 6.3 cm (b) 5.4 cm (c) 7.2 cm (d) 4.5 cm
32. 7 cos 30° + 5 tan 30° + 6 cot 60° is  1
43 41 3 47 49 3
(a) (b) (c) (d)
2 3 2 2 3 2

Sample Paper - 3  181


D:\EG_Mathematics_Standard-10_(Term-1)_(30-07-2021)\Open_Files\Sample_Paper_1-3_(Stand)\Sample_Paper_1-3_(Stand)
\ 13-Sep-2021   Bhandari  Proof-4 Reader’s Sign _______________________ Date __________

33. If one of the zeroes of the quadratic polynomial (k – 1) x2 + kx + 1 is –3, then the value of k is 1
4 2
(b) − (c) (d) −2
4
(a)
3 3 3 3
34. The value of y, if the distance between the points (2, y) and (–4, 3) is 10 is given by 1
(a) 6 (b) –11 (c) 5 (d) 11
35. A pendulum swings through an angle 60° and describes an arc 8.8 cm in length.
 22 
Then, the length of pendulum is  use π = 7   1
(a) 8.40 cm (b) 16.8 cm (c) 4.20 cm (d) 8.90 cm
36. If one zero of the polynomial 3x2 – 8x + 2k + 1 is seven times the other, then the value of k is 1
2 2 2 3
(a) (b) (c) (d)
5 3 7 2
37. A girl of height 9 cm is walking away from the base of a lamp-post at a speed of 1.2 m/s. If the lamp
is 3.6 m above the ground, then the value of length of her shadow after 4s is 1
(a) 3.2 m (b) 4.8 m (c) 1.6 m (d) 3.6 m
38. The ratio, in which the Y-axis divides the line segment joining the points (5, –6) and (–1, –4) is 1
(a) 1 : 5 (b) 5 : 1 (c) 2 : 4 (d) None of these
39. If x tan 45° cos 60° = sin 60° cot 60°, then the value of x will be 1
1 1 3
(a) 1 (b) (c) (d)
2 2 2
40. Six years hence, a man’s age will be three times the age of his son and three years ago he was nine
times as old as his son. The present age of the man is  1
(a) 28 (b) 30 (c) 32 (d) 34

SECTION-C
Section-C consists of 10 questions of 1 mark each. Any 8 questions are to be attempted.
Q41 – Q45 are based on Case Study-1
Case Study-1
Due to ongoing Corona virus outbreak, Wellness Medical store has started selling masks of decent quality.
The store is selling two types of masks currently type A and type B.
The cost of type A mask is ` 15 and of type B mask is ` 20. In the month of April, 2020, the store sold
100 masks for total sales of ` 1650.

` 15 ` 20

182  Mathematics–10
D:\EG_Mathematics_Standard-10_(Term-1)_(30-07-2021)\Open_Files\Sample_Paper_1-3_(Stand)\Sample_Paper_1-3_(Stand)
\ 13-Sep-2021   Bhandari  Proof-4 Reader’s Sign _______________________ Date __________

41. How many masks of each type were sold in the month of April? 1
(a) 40 masks of type A, and 60 masks of type B (b) 60 masks of type A, and 40 masks of type B
(c) 70 masks of type A, and 30 masks of type B (d) 30 masks of type A, and 70 masks of type B
42. If the store had sold 50 masks of each type, what would be its sales in the month of April? 1
(a) ` 550 (b) ` 560 (c) ` 1050 (d) ` 1750
43. Due to great demand and short supply, the store has increased the price of each type by ` 5 from May
1, 2020. In the month of May, 2020, the store sold 310 masks for total sales of ` 6875. How many
masks of each type were sold in the month of May? 1
(a) 175 masks of type A, and 135 masks of type B
(b) 200 masks of type A, and 110 masks of type B
(c) 110 masks of type A, and 200 masks of type B
(d) 135 masks of type A, and 175 masks of type B
44. What percent of masks of each type sale was increased in the month of May, compared with the sale
of month April? 1
(a) 110 % in type A and 180 % in type B (b) 180 % in type A and 110 % in type B
(c) 350 % in type A and 150 % in type B (d) 150 % in type A and 350 % in type B
45. What extra profit did store earn by increasing price in May month? 1
(a) ` 1550 (b) ` 3100 (c) ` 1650 (d) ` 1825

Q46 – Q50 are based on Case Study-2


Case Study-2

Resident Welfare Association (RWA) of a Gulmohar Society in Delhi have installed three electric
poles A, B and C in a society’s common park. Despite these three poles, some parts of the park are
still in dark. So, RWA decides to have one more electric pole D in the park.

The park can be modelled as a coordinate systems given below.
y
10
9
8
A
7
B
6
5
C
4
3
2
1
x
O 1 2 3 4 5 6 7 8 9 10
46.
What is the position of the pole C ? 1
(a) (4, 5) (b) (5, 4) (c) (6, 5) (d) (5, 6)
47.
What is the distance of the pole B from the corner O of the park ? 1
(a) 6 2 units (b) 3 2 units (c) 6 3 units (d) 3 3 units

Sample Paper - 3  183


D:\EG_Mathematics_Standard-10_(Term-1)_(30-07-2021)\Open_Files\Sample_Paper_1-3_(Stand)\Sample_Paper_1-3_(Stand)
\ 13-Sep-2021   Bhandari  Proof-4 Reader’s Sign _______________________ Date __________

48.
The position of the fourth pole D so that four points A, B C and D form a parallelogram is 1
(a) (5, 2) (b) (1, 5) (c) (1, 4) (d) (2, 5)
49.
What is the distance between poles A and C? 1
(a) 6 2 units (b) 3 2 units (c) 6 3 units (d) 3 3 units
50.
What is the distance between poles B and D? 1
(a) 2 3 units (b) 28 units (c) 6 3 units (d) 26 units

Answers
1. (b) 2. (d) 3. (a) 4. (d) 5. (c)
6. (c) 7. (c) 8. (a) 9. (a) 10. (a)
11. (d) 12. (a) 13. (c) 14. (a) 15. (d)
16. (c) 17. (a) 18. (c) 19. (b) 20. (a)
21. (c) 22. (a) 23. (b) 24. (d) 25. (c)
26. (c) 27. (a) 28. (c) 29. (a) 30. (b)
31. (b) 32. (a) 33. (a) 34. (d) 35. (a)
36. (b) 37. (c) 38. (b) 39. (a) 40. (b)
41. (c) 42. (d) 43. (a) 44. (d) 45. (a)
46. (b) 47. (a) 48. (b) 49. (b) 50. (d)

184  Mathematics–10
D:\EG_Mathematics_Standard-10_(Term-1)_(30-07-2021)\Open_Files\Sample_Paper_1-3_(Stand)\Notes
\ 14-Sep-2021   Bhandari Proof-1 Reader’s Sign _______________________ Date __________

NOTES
...........................................................................................................................................................

...........................................................................................................................................................

...........................................................................................................................................................

...........................................................................................................................................................

...........................................................................................................................................................

...........................................................................................................................................................

...........................................................................................................................................................

...........................................................................................................................................................

...........................................................................................................................................................

...........................................................................................................................................................

...........................................................................................................................................................

...........................................................................................................................................................

...........................................................................................................................................................

...........................................................................................................................................................

...........................................................................................................................................................

...........................................................................................................................................................

...........................................................................................................................................................

...........................................................................................................................................................

...........................................................................................................................................................

...........................................................................................................................................................

...........................................................................................................................................................

...........................................................................................................................................................

...........................................................................................................................................................

...........................................................................................................................................................

...........................................................................................................................................................

...........................................................................................................................................................

191
D:\EG_Mathematics_Standard-10_(Term-1)_(30-07-2021)\Open_Files\Sample_Paper_1-3_(Stand)\Notes
\ 14-Sep-2021   Bhandari Proof-1 Reader’s Sign _______________________ Date __________

NOTES
...........................................................................................................................................................

...........................................................................................................................................................

...........................................................................................................................................................

...........................................................................................................................................................

...........................................................................................................................................................

...........................................................................................................................................................

...........................................................................................................................................................

...........................................................................................................................................................

...........................................................................................................................................................

...........................................................................................................................................................

...........................................................................................................................................................

...........................................................................................................................................................

...........................................................................................................................................................

...........................................................................................................................................................

...........................................................................................................................................................

...........................................................................................................................................................

...........................................................................................................................................................

...........................................................................................................................................................

...........................................................................................................................................................

...........................................................................................................................................................

...........................................................................................................................................................

...........................................................................................................................................................

...........................................................................................................................................................

...........................................................................................................................................................

...........................................................................................................................................................

...........................................................................................................................................................

192

You might also like